NTA CUET (UG) 10 Mock Test Sample Question Papers Economics (2024) 9789357288125, 9357288120

Description of the product: • 100% Exam Ready With 2023 CUET(UG) Exam Papers (2 Slots) – Fully Solved with Explanations

112 25 21MB

English Pages 123 [122] Year 2023

Report DMCA / Copyright

DOWNLOAD PDF FILE

Table of contents :
Cover page
Copyright
Contents
Oswaal Books Expert Tips to Crack CUET (UG) in the first Attempt
Latest Syllabus
CUET Solved Paper, 2023 (29th May 2023)
Latest 2022 Solved Paper 18th August 2022 Slot-1
Sample Question Papers
Sample Question Paper -1
Sample Question Paper -2
Sample Question Paper -3
Sample Question Paper -4
Sample Question Paper -5
Sample Question Paper -6
Sample Question Paper -7
Sample Question Paper -8
Sample Question Paper -9
Sample Question Paper -10
Solutions
Solutions of Question Paper - 1
Solutions of Question Paper - 2
Solutions of Question Paper - 3
Solutions of Question Paper - 4
Solutions of Question Paper - 5
Solutions of Question Paper - 6
Solutions of Question Paper - 7
Solutions of Question Paper - 8
Solutions of Question Paper - 9
Solutions of Question Paper - 10
Recommend Papers

NTA CUET (UG) 10 Mock Test Sample Question Papers Economics (2024)
 9789357288125, 9357288120

  • 0 0 0
  • Like this paper and download? You can publish your own PDF file online for free in a few minutes! Sign Up
File loading please wait...
Citation preview

For 2024 Exam

BEST SELLER

ECONOMICS Section II (Domain Specific Subject) Strictly ar per the Latest Examination Pattern issued by NTA

The ONLY book you need to Ace CUET (UG)

1

2

3

4

100% Exam Readiness

Extensive Practice

Concept Clarity

Valuable Exam Insights

With 10 Solved Sample Question Papers

With 550+ NCERT - based MCQs

With 350+ Explanations & Smart Answer Keys

With Latest Solved Paper 2023

(i)

3rd EDITION

ISBN SYLLABUS COVERED

YEAR 2023-24 “9789357288125”

CUET (UG) CERTIFICATE OF COMMON UNIVERSITY ENTRANCE TEST

PUBLISHED BY

COPYRIG HT

RESERVED BY THE PUBLISHERS

All rights reserved. No part of this book may be reproduced, stored in a retrieval system, or transmitted, in any form or by any means, without written permission from the publishers. The author and publisher will gladly receive information enabling them to rectify any error or omission in subsequent editions.

OSWAAL BOOKS & LEARNING PVT. LTD. 1/11, Sahitya Kunj, M.G. Road, Agra - 282002, (UP) India

1010, Cambourne Business Centre Cambridge, Cambridgeshire CB 236DP, United kingdom

0562-2857671

[email protected]

www.OswaalBooks.com

DI SCL A IMER

This book is published by Oswaal Books and Learning Pvt Ltd (“Publisher”) and is intended solely for educational use, to enable students to practice for examinations/tests and reference. The contents of this book primarily comprise a collection of questions that have been sourced from previous examination papers. Any practice questions and/or notes included by the Publisher are formulated by placing reliance on previous question papers and are in keeping with the format/pattern/ guidelines applicable to such papers. The Publisher expressly disclaims any liability for the use of, or references to, any terms or terminology in the book, which may not be considered appropriate or may be considered offensive, in light of societal changes. Further, the contents of this book, including references to any persons, corporations, brands, political parties, incidents, historical events and/or terminology within the book, if any, are not intended to be offensive, and/or to hurt, insult or defame any person (whether living or dead), entity, gender, caste, religion, race, etc. and any interpretation to this effect is unintended and purely incidental. While we try to keep our publications as updated and accurate as possible, human error may creep in. We expressly disclaim liability for errors and/or omissions in the content, if any, and further disclaim any liability for any loss or damages in connection with the use of the book and reference to its contents”.

Kindle Edition ( ii )

Preface National Testing Agency (NTA) has been established in November 2017 under the Societies Registration Act (1860) by the Ministry of Education as a premier, specialist, autonomous, and self-sustained testing organization to conduct entrance examinations for admission/fellowship in higher educational institutions. The Common University Entrance Test (CUET (UG) - 2022) is being introduced for admission into all UG Programmes in all Central Universities for the academic session 2023 under the Ministry of Education, (MoE). The Common University Entrance Test (CUET) will provide a common platform and equal opportunities to candidates across the country, especially those from rural and other remote areas, and help establish a better connection with the Universities. A single examination will enable the Candidates to cover a wide outreach and be part of the admissions process to various Central Universities. CUET – UG Computer Based Test (CBT) for the Central Universities is to be conducted by the National Testing Agency (NTA). The curriculum for CUET is based on the National Council of Educational Research and Training (NCERT) syllabus for class 12 only. CUET scores are mandatorily required while admitting students to undergraduate courses in 44 central universities. A merit list will be prepared by participating Universities/organizations. Universities may conduct their individual counselling on the basis of the scorecard of CUET (UG)-2023 provided by NTA.

A few benefits of studying from Oswaal Sample Question Papers • • • •

100% Exam Readiness With 10 Solved Sample Question Papers Extensive Practice With 550+ NCERT - based MCQs Concept Clarity With 350+ Explanations & Smart Answer Keys Valuable Exam Insights With Latest Solved Papers 2023

Our Heartfelt Gratitude! Finally, we would like to thank our authors, editors, and reviewers. Special thanks to our students who send us suggestions and constantly help improve our books. We promise to always strive towards ‘Making Learning Simple’ for all of you. Wish you all Happy Learning!

( iii )

-Team Oswaal Books

Contents

l l l l

Oswaal Books Expert Tips to Crack CUET (UG) in the first Attempt Latest Syllabus CUET Solved Paper, 2023 (29th May 2023) Latest 2022 Solved Paper 18th August 2022 Slot-1

v - vi - 1 - 15 -

v vi 14 24

Sample Question Papers l Sample Question Paper - 1 l Sample Question Paper - 2 l Sample Question Paper - 3 l Sample Question Paper - 4 l Sample Question Paper - 5 l Sample Question Paper - 6 l Sample Question Paper - 7 l Sample Question Paper - 8 l Sample Question Paper - 9 l Sample Question Paper - 10

1 - 5 6 - 10 11 - 15 16 - 21 22 - 26 27 - 31 32 - 36 37 - 41 42 - 46 47 - 51

Solutions l Sample Question Paper - 1 l Sample Question Paper - 2 l Sample Question Paper - 3 l Sample Question Paper - 4 l Sample Question Paper - 5 l Sample Question Paper - 6 l Sample Question Paper - 7 l Sample Question Paper - 8 l Sample Question Paper - 9 l Sample Question Paper - 10 

52 - 55 56 - 59 60 - 62 63 - 65 66 - 69 70 - 72 73 - 76 77 - 80 81 - 84 85 - 88 

Books Expert Tips to Crack Oswaal BooksOswaal Expert Tips to Crack CUET (UG) in the First A empt CUET (UG) in the First Attempt

Excited about your UG but unsure if you will get admission to your preferred university? In a major announcement by the chairman of the University Grants Commission, the Naonal Tesng Agency will be conducng the Common Universies Entrance Test (CUET (UG) 2022) for undergraduate programs in Central Universies for the upcoming academic session. However, the UGC Chairperson also stated that CUET (UG) will not just be limited to admissions to Central Universies. Many prominent private universies have indicated that they would also like to adopt a common entrance exam for undergraduate admissions and take admissions on the basis of CUET (UG) scores. This makes CUET (UG) a very important examinaon in itself and hence it becomes mandatory to be aware of the ps & tricks that could help you ace the exam on the first a† empt.

The first step is to understand The pa† ern of the examinaon. CUET includes three secons, secon 1 includes queson based on languages, secon 2 includes 27 domain-based subjects and secon 3 includes General Test. The syllabus of the upcoming Common University Entrance Test, CUET 2022, will be completely based on the syllabus of class 12 th . No queson will be asked from class 11th syllabus.

While preparing for the exam, it is i m p o r t a n t t o i d e n f y t h e important topics and pracce important quesons from those t o p i c s . P r a c c e i m p o r t a n t q u e s o n s t h r o u g h O s w a a l Q u e so n B a n k a n d S a m p l e Q u e s o n P a p e r s , L i s n g topics also helps in idenfying the weak areas that need special effort and me. The aspirants can start preparing to focus on the areas that they consider to be tough, followed by the ones that are their strengths.

Make a habit of preparing notes f ro m t h e b e g i n n i n g o f t h e preparaon. It will not only help in making the study systemac but also make the revision of the syllabus easy even when you might have limited me to revise.

Collecng and preparing from the appropriate study material cannot be ignored as irrelevant. The books chosen by the aspirants to study from should be on the lines of the current syllabus and the ones that could help you with swi• revision before the examinaon.

Make sure to revise as much as possible. The revision will help the aspirants in keeping the concepts fresh in their minds unl the day of the final examinaons. They may refer to a few good pracce quesons and concise revision notes to achieve their desired results.

Devote a sufficient amount of me to all the secons of the examinaons. This requires a wellmade plan and an honest adherence to the said plan. Priorize the most important topics or the topics that the aspirants are not familiar with to be able to master them in me.

With this said, an important queson that is gaining ground amongst students who will be appearing for this exam is if they should take coaching to get themselves ready for the exams. The answer is a simple no, the exam will simply not require any coaching as it is completely based on the Class 12th syllabus which will be quite fresh in students' minds as they will be just out of school. All they need is a good revision and pracce of quesons from Oswaal Queson Bank and Sample Queson Papers for CUET (UG) preparaons.

(v)

Latest Syllabus ECONOMICS - 309 Note: There will be one Question Paper which will have 50 questions out of which 40 questions need to be attempted. Unit I: Introduction to Microeconomics • What is microeconomics? • Central problems Unit II: Consumer Behaviour and Demand • Consumer’s Equilibrium: meaning and attainment of equilibrium through Utility Approach: One and two commodity cases. • Demand: market demand, determinants of demand, demand schedule, demand curve, movement along and shifts in the demand curve, price elasticity of demand, measurement of price elasticity of demand – percentage, total expenditure, and geometric methods

Introductory Macroeconomics

Unit III: National Income and Related Aggregates — Basic Concepts and Measurement • Macroeconomics: meaning. • Circular flow of income, concepts of GDP, GNP, NDP, NNP (at market price and factor cost). • Measurement of National Income –Value Added method, Income method, and Expenditure method. Unit IV: Determination of Income and Employment • Aggregate demand, aggregate supply, and their components • Propensity to consume and propensity to save (average and marginal) • Meaning of involuntary unemployment and full employment • Determination of income and employment: two-sector model • Concept of investment multiplier and its working • Problems of excess and deficient demand • Measures to correct excess and deficient demand – availability of credit, change in government spending Unit V: Money and Banking • Money: meaning, evolution, and functions • Central bank: meaning and functions • Commercial banks: meaning and functions Unit VI: Government Budget and the Economy • Government budget – meaning and its components • Objectives of government budget • Classification of receipts – revenue and capital; classification of expenditure – revenue and capital, plan and nonplan, and developmental and non-developmental • Balanced budget, surplus budget, and deficit budget: meaning and implications • Revenue deficit, fiscal deficit, and primary deficit: meaning and implications; measures to contain different deficits. Unit VII: Balance of Payments • Foreign exchange rate – meaning (fixed and flexible), merits and demerits; determination through demand and supply • Balance of payments accounts – meaning and components • A brief analysis of recent exchange rate issues

Indian Economic Development

Unit VIII: Development Experience (1947-90) and Economic Reforms since 1991 A brief introduction of the state of the Indian economy on the eve of independence. Indian economic system and common goals of Five year Plans. Main features, problems and policies of agriculture (institutional aspects and new agricultural strategy), industry (IPR 1956; SSI – role & importance) and foreign trade. Unit IX: Current challenges facing the Indian Economy Poverty: absolute and relative; Main programmes for poverty alleviation: A critical assessment; Human Capital Formation: How many people become resource; Role of human capital in economic development; Rural development: Key issues: credit and marketing – role of cooperatives; agricultural diversification; Employment: Growth and changes in work force participation rate in formal and informal sectors; problems and policies Infrastructure: Meaning and Types: Cases Studies: Health: Problems and Policies – A critical assessment; Sustainable Economic Development: Meaning, Effects of Economic Development on Resources and Environment, including global warming Unit X: Development Experience of India • A comparison with neighbours • India and Pakistan • India and China • Issues: economic growth, population, sectoral development and other Human Development Indicators 

( vi )

Exclusive School Books Suppliers Exclusive School Books Suppliers

VIJAYAWADA WEST KAMENG BANGLORE RAJKOT

MAHARASHTRA

ANDHRA PRADESH

Sri Vikas Book Centre, 9848571114, 9440715700,

PUNE

ASSAM

JALNA

Dutta Book Stall, 8729948473

KARNATAKA

CHENNAI

Satish Agencies, 8861630123

GUJRAT

HYDERABAD

Royal Stationers, 9824207514

KOLKATA

ANDHRA PRADESH

GUWAHATI

PATNA

INDORE

Akshaya Books Corner, 9666155555

Bookmark-IT, 7305151653

TELANGANA

Sri Balaji Book Depot , 9676996199, (040) 27613300

WEST BENGAL

United Book House, 9831344622

Bhaiya Industries, 9893326853, Sushil Prakashan,(0731) 2503333, 2535892, 9425322330, Bhaiya Store, 9425318103, Arun Prakashan, 9424890785, Bhaiya Book Centre, 9424081874, Seva Suppliers, 9826451052

ANDAMAN & NICOBAR ISLAND PORTBLAIR

Anil Paper Mart, 9422722522, (02482) 230733

TAMIL NADU

Our Distributors

VISAKHAPATHAM JBD Educational, 9246632691, 9246633693, Sri Rajeshwari Book Link, 9848036014 VIJAYAWADA

Madhusheela Books & Stationery, 7875899892

Krishna Book Centre, 9474205570, Kumar Book Depot, 9932082455, Kumar Book Depot, 9932082455, Sree aditya Book Centre, 8332972720, 7013300914

ASSAM

Book Emporium, 9675972993, 6000763186, Ashok Publication, 7896141127, Kayaan Enterprises, (0361) 2630443, Orchid Book house, 9864624209, Newco, 9864178188

JABALPUR

Vinay Pustak Sadan, 8962362667, Anand Books and Stationers, 9425323508

SAGAR

Princi Book Depot, Sagar, 9977277011

KATNI

Shri Mahavir Agency, 9425363412

UJJAIN BHOPAL

BIHAR

Nova Publisher & Distributors, (0612) 2666404, Shri Durga Pustak Mandir, 9334477386, Sharda Pustak Bhandar, 9334259293, Vikas Book Depot, 9504780402, Alka Book Agency, 9835655005, Metro Book(E&C), Ishu Pustak Bhandar, 8294576789, Gyan Ganga Limited, 6203900312, Ishu Pustak Bhandar, ( E & C ), 9334186300/8294576789

PUNE

Shreenath Book Depot, 9827544045 Gupta Brother, 9644482444

MAHARASHTRA

Natraj Book Depot, (020) 24485054, 9890054092, Vikas Book House, 9921331187, Pravin Sales, 9890683475, New Saraswati Granth Bhandar, 9422323859, Akshar Books & Stationary, 7385089789, Vardhaman Educational, 9860574354, Yash Book Centre, 9890156763, Pragati Book Centre, (ISC), 9850039311, Praveen Sales, Pragati Book Centre, Pune ( E & C ), 9850039311 Shree Sainath Agencies, 7350294089, Maya Book Centre, (ISC), 9372360150 Vidyarthi Sales Agencies, 9819776110, New Student Agencies, 7045065799, Shivam Books & Stationery, 8619805332

Pustak Bhandar, 7870834225

CHATTISGARH

AURANGABAD MUMBAI

AMBIKAPUR

Saini Brothers, 9425582561, M.P Department Stores, 9425254264

JALGAON

BOKARO BHILAI

Bokaro Student Friends Pvt. Ltd, Bokaro, 7277931285 Anil Book Depot, 9425234260

LATUR KOLHAPUR

Yash Book House, 9637936999, Shri Ganesh Pustakalay, 9730172188 Granth the Book World, 9922295522

KORBA

Kitab Ghar, Korba ( E & C ), 9425226528

NANDED

MUZAFFARPUR

DURG

RAIPUR RAIGARH DELHI

Bhagwati Bhawani Book Depot, 0788-2327620, 9827473100

NAGPUR

Sharma Book Depot & Stat. (ISC), 9421393040

Laxmi Pustakalay and Stationers, (0712) 2727354, Vijay Book Depot, 9860122094

Shri Ramdev Traders, 9981761797, Gupta Pustak Mandir, 7974220323, Anil Publication, 9691618258/7999078802

NASHIK

Renuka Book distributor, 9765406133, Novelty Book Depot, 9657690220, Karamveer Book Depot, 9923966466, Arun Book & Stationers, 9423110953 Abhang Pustakalaya, 9823470756/9175940756 Rahul Book Centre, 9970849681, New India Book House, 9623123458

DELHI

YAVATMAL

Shri Ganesh Pustkalaya, 9423131275

Sindhu Book Deopt, 9981935763

DHULE

Mittal Books, (011) 23288887, 9899037390, Singhania Book & Stationer, 9212028238, AoneBooks, New Delhi, 8800497047, Radhey Book Depot, 9818314141, Batheja Super Store, 9871833924, Lov Dev & Sons, Delhi ( E & C ), 9811182352, Zombozone, 9871274082, LDS Marketing, 9811182352/9999353491

VASAI

Navjeevan Book Stall, 7020525561

Prime Book Centre, Vasai, 9890293662

ODISHA A. K. Mishra Agencies, 9437025991, 9437081319

GUJARAT

CUTTACK

BHAVNAGAR DAHOD VAPI

Patel Book, 9898184248, 9824386112, 9825900335, Zaveri Agency, 9979897312, 9979890330, Hardik Book Agency, (ISC) 079-24110043, 9904659821 Samir Book Stall, Bhavnagar (ISC) 9586305305 Collegian Book Corner, 9925501981 Goutam Book Sellers, 9081790813

BHUBANESHWAR M/s Pragnya, 8847888616, 9437943777, Padmalaya, 9437026922, Bidyashree, 9937017070, Books Godown, 7894281110 BARIPADA Trimurti Book World, 9437034735 KEONJHAR Students corner, 7008435418

VALSAD NAVSARI

Mahavir Stationers, 9429474177 College Store, (ISC) NO CALL 02637-258642, 9825099121

AMBALA PATIALA

VADODARA

Umakant Book Sellers & Stationer, 9624920709

HARYANA

FEROZPUR LUDHIANA

ROHTAK

Manish Traders, 9812556687, Swami Kitab Ghar, 9355611088,

CHANDIGARH

REWARI

Sanjay book depot, 9255447231

Kashi Ram Kishan lal, 9289504004, 8920567245 Natraj Book Distributors, 7988917452

AJMER KOTA

BHUNA

Khurana Book Store, 9896572520

BHILWARA

JAMMU

JAIPUR

Sahitya Sangam, 9419190177

UDAIPUR

Nakoda Book Depot, (01482) 243653, 9214983594, Alankar Book Depot, 9414707462 Ravi Enterprises, 9829060694, Saraswati Book House, (0141) 2610823, 9829811155, Goyal Book Distt., 9460983939, 9414782130 Sunil Book Store, 9828682260

Crown Book Distributor & Publishers, (0651) 2213735, 9431173904, Pustak Mandir, 9431115138, Vidyarthi Pustak Bhandar, 9431310228

AGARTALA

Book Corner, 8794894165, 8984657146, Book Emporium, 9089230412

KARNATAKA

COIMBATORE

SURAT

BALLABGARH HISAR

BOKARO RANCHI DUMKA

PUNJAB

Shopping Point, 9824108663

JALANDHAR

Babu Ram Pradeep Kumar, 9813214692

JHARKHAND

JODHPUR

Bokaro Student Friends, (0654) 2233094, 7360021503, Bharati Bhawan Agencies, 9431740797

Renuka Book Distributor, (0836) 2244124, Vidyamandir Book Distributors, 9980773976 CHENNAI

BANGLORE

Krishna book house, 9739847334, Hema Book Stores, 9986767000,

BELLERI

Chatinya book centre, 9886064731

PUDUCHERRY

ERNAKULAM

Academic Book House, (0484) 2376613, H & C Store, 9864196344, Surya Book House, 9847124217, 9847238314 Book Centre, (0481) 2566992 Academic Book House, (0471) 2333349, 9447063349, Ponni Book Stall, 9037591721

TRICHY

KOTTAYAM TRIVANDRUM CALICUT

Sapna Book House Pvt. Ltd., 9980513242, Hema Book World, (Chamrajpet) (ISC) 080-40905110, 9945731121

Aman Book Stall, (0495) 2721282,

MADHYA PRADESH

CHHINDWARA

Pustak Bhawan, ( E & C ), 8982150100

GWALIOR

Agarwal Book Depot, 9425116210

Cheap Book Store, 9872223458, 9878258592, City Book Shop, 9417440753, Subhash Book Depot, 9876453625, Paramvir Enterprises, 9878626248 Sita Ram book Depot, 9463039199, 7696141911 Amit Book, 9815807871, Gupta Brothers, 9888200206, Bhatia Book Centre, 9815277131 Mohindra Book Depot, 9814920226

RAJASTHAN

Laxmi General Store, Ajmer, 0145- 2428942 9460652197 Vardhman Book Depot, 9571365020, 8003221190 Raj Traders, 9309232829

Second Hand Book Stall, 9460004745

TRIPURA

TAMIL NADU

HUBLI

KERALA

Bharat Book Depot, 7988455354 Goel Sons, 9463619978, Adarsh Enterprises, 9814347613

SALEM

THENI MADURAI VELLORE

HYDERABAD

Majestic Book House, (0422) 2384333, CBSC Book Shop, 9585979752

Arraba Book Traders, (044) 25387868, 9841459105, M.R. Book Store (044) 25364596, Kalaimagal Store, (044) 5544072, 9940619404, Vijaya Stores, 9381037417, Bookmark It-Books & Stat. Store, 7305151653, M.K. Store, 9840030099, Tiger Books Pvt. Ltd., 9710447000, New Mylai Stationers, 9841313062, Prince Book House, Chennai, 0444-2053926, 9952068491, S K Publishers & Distributors, 9789865544, Dharma Book Shop, 8667227171 Sri Lakshmi Book Seller, 7871555145 Pattu book centre, 9894816280

P.R.Sons Book Seller, 9443370597, Rasi Publication, 9894816280 Maya Book Centre, 9443929274 Selvi Book Shoppe, 9843057435, Jayam Book Centre, 9894658036 G.K book centre and collections, 9894517994

TELANGANA

Sri Balaji Book Depot, (040) 27613300, 9866355473, Shah Book House, 9849564564 Vishal Book Distributors, 9246333166, Himalaya Book World, 7032578527

( vii )

0808

AHMEDABAD

Contd... UTTARAKHAND

GORAKHPUR

Central Book House, 9935454590, Friends & Co., 9450277154, Dinesh book depot, 9125818274, Friends & Co., 9450277154

DEHRADUN

Inder Book Agencies, 9634045280, Amar Book Depot , 8130491477, Goyal Book Store, 9897318047, New National Book House, 9897830283/9720590054

JHANSI

Bhanu Book Depot, 9415031340

MUSSORIE

Ram Saran Dass Chanda kiran, 0135-2632785, 9761344588

KANPUR

Radha News Agency, 8957247427, Raj Book Dist., 9235616506, H K Book Distributors, 9935146730, H K Book Distributors, 9506033137/9935146730

UTTAR PRADESH

LUCKNOW

Sparsh Book Agency, 9412257817, Om Pustak Mandir, (0562) 2464014, 9319117771,

MEERUT

AGRA

Sanjay Publication, 8126699922 Arti book centre, 8630128856, Panchsheel Books, 9412257962, Bhagwati Book Store, (E & C), 9149081912

Vyapar Sadan, 7607102462, Om Book Depot, 7705871398, Azad Book Depot Pvt. Ltd.,

7317000250, Book Sadan, 9839487327, Rama Book Depot(Retail), 7355078254, Ashirwad Book Depot, 9235501197, Book.com, 7458922755, Universal Books,

9450302161, Sheetla Book Agency, 9235832418, Vidyarthi Kendra Publisher & Distributor Pvt Ltd, (Gold), 9554967415, Tripathi Book House, 9415425943

Ideal Book Depot, (0121) 4059252, 9837066307

ALLAHABAD

Mehrotra Book Agency, (0532) 2266865, 9415636890

NOIDA

Prozo (Global Edu4 Share Pvt. Ltd), 9318395520, Goyal Books Overseas Pvt.Ltd., 1204655555 9873387003

AZAMGARH

Sasta Sahitya Bhandar, 9450029674

PRAYAGRAJ

Kanhaiya Pustak Bhawan, 9415317109

ALIGARH

K.B.C.L. Agarwal, 9897124960, Shaligram Agencies, 9412317800, New Vimal Books, 9997398868, T.I.C Book centre, 9808039570

MAWANA

Subhash Book Depot, 9760262264

BULANDSHAHAR

Rastogi Book Depot, 9837053462/9368978202

BALRAMPUR

Universal Book Center, 8933826726

KOLKATA

BAREILLY

Siksha Prakashan, 9837829284

RENUKOOT

HARDOI

Mittal Pustak Kendra, 9838201466

WEST BENGAL Oriental Publishers & Distributor (033) 40628367, Katha 'O' Kahini, (033) 22196313, 22419071, Saha Book House, (033), 22193671, 9333416484, United Book House, 9831344622, Bijay Pustak Bhandar, 8961260603, Shawan Books Distributors, 8336820363, Krishna Book House, 9123083874

Om Stationers, 7007326732

DEORIA

Kanodia Book Depot, 9415277835

COOCH BEHAR

S.B. Book Distributor, Cooch behar, 9002670771

VARANASI

Gupta Books, 8707225564, Bookman & Company, 9935194495/7668899901

KHARAGPUR

Subhani Book Store, 9046891334

MATHURA

Sapra Traders, 9410076716, Vijay Book House , 9897254292

SILIGURI

Agarwal Book House, 9832038727, Modern Book Agency, 8145578772

FARRUKHABAD

Anurag Book Agencies, 8844007575

DINAJPUR

Krishna Book House, 7031748945

NAJIBABAD

Gupta News Agency, 8868932500, Gupta News Agency, ( E & C ), 8868932500

MURSHIDABAD

New Book House, 8944876176

DHAMPUR

Ramkumar Mahaveer Prasad, 9411942550

Entrance & Competition Distributors PATNA

BIHAR

CUTTAK

Metro Books Corner, 9431647013, Alka Book Agency, 9835655005, Vikas Book Depot, 9504780402

BHUBANESHWAR

CHATTISGARH KORBA

Kitab Ghar, 9425226528, Shri Ramdev Traders, 9981761797

A.K.Mishra Agencies, 9437025991 M/s Pragnya, 9437943777

PUNJAB JALANDHAR

Cheap Book Store, 9872223458, 9878258592

DELHI

RAJASTHAN

DELHI

Singhania Book & Stationer, 9212028238, Radhey Book depot, 9818314141, The KOTA Book Shop, 9310262701, Mittal Books, 9899037390, Lov Dev & Sons, 9999353491

Vardhman Book Depot, 9571365020, Raj Traders, 9309232829

NEW DELHI

Anupam Sales, 9560504617, A ONE BOOKS, 8800497047

JAIPUR

HARYANA AMBALA

BOKARO

Goyal Book Distributors, 9414782130

UTTAR PRADESH

Bharat Book Depot, 7988455354

AGRA

BHAGWATI BOOK STORE, 9149081912, Sparsh Book Agency, 9412257817, Sanjay Publication, 8126699922

JHARKHAND

ALIGARH

New Vimal Books, 9997398868

Bokaro Student Friends Pvt. Ltd, 7360021503

ALLAHABAD

Mehrotra Book Agency, (532) 2266865, 9415636890

MADHYA PRADESH

GORAKHPUR

Central Book House, 9935454590

INDORE

Bhaiya Industries, 9109120101

KANPUR

Raj Book Dist, 9235616506

CHHINDWARA

Pustak Bhawan, 9827255997

LUCKNOW

Azad Book Depot PVT LTD, 7317000250, Rama Book Depot(Retail), 7355078254 Ashirwad Book Depot , 9235501197, Book Sadan, 8318643277, Book.com , 7458922755, Sheetla Book Agency, 9235832418

MAHARASHTRA

PRAYAGRAJ

Format Center, 9335115561, Garg Brothers Trading & Services Pvt. Ltd., 7388100499

NAGPUR

Laxmi Pustakalay and Stationers, (0712) 2727354

PUNE

Pragati Book Centre, 9850039311

MUMBAI

New Student Agencies LLP, 7045065799

ODISHA

Inder Book Agancies, 9634045280

WEST BENGAL KOLKATA

Bijay Pustak Bhandar Pvt. Ltd., 8961260603, Saha Book House, 9674827254 United Book House, 9831344622, Techno World, 9830168159

Trimurti Book World, 9437034735

0808

BARIPADA

UTTAR PRADESH DEHRADUN

( viii )

CUET (UG) Exam Paper 2023 National Testing Agency Held on 29th May 2023

ECONOMICS Solved

(This includes Questions pertaining to Domain Specific Subject only)

Max. Marks : 200

Time allowed : 45 Minutes

General Instructions: (i) This paper consists of 50 MCQs, attempt any 40 out of 50 . (ii) Correct answer or the most appropriate answer: Five marks (+5) . (iii) Any incorrect option marked will be given minus one mark (– 1) . (iv) Unanswered/Marked for Review will be given no mark (0) . (v) If more than one option is found to be correct then Five marks (+5) will be awarded to only those who have marked any of the correct options . (vi) If all options are found to be correct then Five marks (+5) will be awarded to all those who have attempted the question . (vii) If none of the options is found correct or a Question is found to be wrong or a Question is dropped then all candidates who have appeared will be given five marks (+5). (viii) Calculator / any electronic gadgets are not permitted . 1. A country’s consumption function described as C = 500+0.7 Y the Autonomous consumption (1) 350 (2) 500 (3) 3500 (4) 350 Ans. Option (2) is correct. Explanation: The autonomous consumption refers to the level of consumption that occurs even when there is no income or output. In the given consumption function, the autonomous consumption is represented by the constant term, which is 500. This means that regardless of the level of income (Y), the country will have a baseline consumption of 500 2. Escheat is an example of: (1) Capital receipt (2) Revenue receipt (3) Capital expenditure (4) Revenue expenditure Ans. Option (2) is correct. Explanation: Escheat refers to the legal process through which the ownership of property, assets, or funds is transferred to the government when the rightful owner cannot be identified or located. This typically occurs when an individual passes away without leaving a will or any known heirs. In such cases, the government assumes ownership of the property or funds for the benefit of the state. When the government receives the property or funds through the process of escheat, it is considered a revenue receipt. Revenue receipts are the income or

funds received by the government from various sources, such as taxes, fines, fees, or in this case, unclaimed property. These receipts contribute to the government's general revenue and can be used to finance public expenditures and services. 3. Match List I with List II: List -I

List - II

(A)

Reduction in (I) value of currency

Disinvestment

(B)

Dilution of (II) share in organisation

Inflation

(C)

Restriction on (III) trade by taxes

Devaluation

(D)

Rise in general (IV) price level

Tarrif Barriers

Choose the correct answer from the options given below: (1) (A)-(III), (B)-(I), (C)-(I), (D)-(IV) (2) (A)-(I), (B)-(II), (C)-(III), (D)-(IV) (3) (A)-(IV), (B)-(II), (C)-(III), (D)-(I) (4) (A)-(III), (B)-(I), (C)-(IV), (D)-(II) Ans. Option (4) is correct. Explanation: A. Reduction in value of currency corresponds to III Devaluation. Devaluation refers to a deliberate downward adjustment in the value of a country's currency relative to other currencies, leading to a decrease in its value.

2

OSWAAL CUET (UG) Sample Question Papers, ECONOMICS B. Dilution of share in organization corresponds to I. Disinvestment. Disinvestment refers to the act of selling or liquidating assets or shares in a company. C. Restriction on trade by taxes corresponds to IV. Tariff Barriers. Tariff barriers are taxes or duties imposed on imported goods, which can restrict international trade. D. Rise in the general price level corresponds to II. Inflation. Inflation refers to the sustained increase in the general price level of goods and services in an economy over a period of time.

4. When nation is a borrower from other countries means: (1) Trade surplus (2) Deficit in current Account (3) Surplus in current Account (4) Trade Deficit Ans. Option (2) is correct. Explanation: The current account is a component of a country's balance of payments and measures the inflow and outflow of goods, services, income, and unilateral transfers between the country and the rest of the world. It consists of trade in goods (exports and imports), trade in services (such as tourism and transportation), income flows (such as wages and investment income), and unilateral transfers (such as foreign aid and remittances). A deficit in the current account occurs when the value of imports of goods, services, and income exceeds the value of exports. In other words, the country is spending more on imports and payments to foreign entities than it is earning from exports and receipts from abroad. This situation often leads to the country needing to borrow from other nations or rely on foreign capital inflows to finance the deficit. 5. “Growth refers to increase in the country’s capacity of module the output of goods and services with in the country.” Choose the incorrect option from the following in relation to Growth: (1) Growth implies a large stock of production capital. (2) Growth implies a large stock of supporting services. (3) Growth implies an equal contribution to GDP by each sectors of the economy. (4) Growth implies an increase in efficiency of productive capital and services. Ans. Option (3) is correct. Explanation: When discussing economic growth, it does not mean that each sector of the economy will contribute equally to the increase in GDP. In fact, different sectors may experience varying rates of growth or even contraction over time. The contribution of each sector to GDP is influenced by factors such as technological advancements, changes in

consumer demand, government policies, and global economic conditions. Certain sectors, such as manufacturing or services, may experience faster growth due to increased productivity, innovation, or market demand. On the other hand, some sectors may face challenges or decline due to changes in the economy, competition, or shifts in consumer preferences. 6. Match List I with List II: List -I

List - II

(A)

NABARD

(I)

Women-oriented community bared poverty reduction program

(B)

Kudumbashree

(II)

Uses the mixed croplive stock farming system

(C)

Animal Husbandry

(III)

HYV seeds, chemical fertilizers

(D)

Organic farming

(IV)

Set up in 1982

Choose the correct answer from the options given below: (1) (A)-(IV), (B)-(III), (C)-(II), (D)-(I) (2) (A)-(IV), (B)-(I), (C)-(II), (D)-(III) (3) (A)-(IV), (B)-(II), (C)-(I), (D)-(III) (4) (A)-(III), (B)-(I), (C)-(II), (D)-(IV) Ans. Option (2) is correct. Explanation: A. NABARD corresponds to IV. Set up in 1982. NABARD (National Bank for Agriculture and Rural Development) is an Indian financial institution that focuses on providing credit and financial support to rural agriculture and rural development. B. Kudumbashree corresponds to I. Womenoriented community poverty reduction program. Kudumbashree is a poverty eradication and women empowerment program implemented in the state of Kerala, India. It aims to improve the quality of life of women by organising them into communitybased groups and providing support for income-generating activities. C. Animal Husbandry corresponds to II. Uses the mixed crop-livestock farming system. Animal husbandry refers to the agricultural practice of raising and breeding livestock, including cattle, sheep, goats, poultry, and other animals. The mixed crop-livestock farming system involves integrating livestock and crop production on the same farm to enhance overall productivity and sustainability. D. Organic farming corresponds to III. HYV seeds, chemical fertilizers. Organic farming is an agricultural approach that emphasizes the use of natural methods and avoids the use of synthetic chemicals, pesticides, and genetically modified organisms. It does not rely on high-yield variety (HYV) seeds or chemical fertilizers commonly used in conventional farming.

CUET Board Paper 2023 7. The benefit of organic farming is. (1) Limited shelf life (2) Labour intensive (3) Limited off - season crops (4) Marketing issues Ans. Option (2) is correct. Explanation: 1. Soil and crop management: Organic farmers focus on maintaining soil health through practices such as composting, mulching, and crop rotation. These activities require hands-on work, such as preparing compost, spreading mulch, and manually tending to crops. 2. Weed and pest control: Organic farming avoids the use of chemical pesticides, so organic farmers need to employ alternative methods for weed and pest control. This can include manual weeding, using physical barriers, and implementing biological pest control measures, which all require laborintensive efforts. 3. Organic certification: To sell products as organic, farmers need to meet specific standards and obtain organic certification. This process involves maintaining detailed records, undergoing inspections, and complying with regulations. It requires additional labor and administrative work to ensure compliance with organic certification requirements. 8. Arrange the following events in proper sequence. A. Ultraviolet radiation causes skin cancer B. High levels of chlorine and bromine compounds in the stratosphere C. Fridge brought to the house D. Chlorofluore carbons used as cooling agent E. Depletion of ozone layer Choose the correct answer from the options given below: (1) B, C, D, E, A (2) C, D, E, B, A (3) C, D, B, E, A (4) E, D, C, B, A Ans. Option (3) is correct. Explanation: C. Fridge brought to the house: This event refers to the introduction of a refrigerator into the house, which is unrelated to the other events in the sequence. D. Chlorofluorocarbons used as a cooling agent: Chlorofluorocarbons (CFCs) are synthetic compounds that were commonly used as cooling agents in refrigerators, air conditioners, and aerosol propellants. B. High levels of chlorine and bromine compounds in the stratosphere: The release of CFCs into the atmosphere leads to the accumulation of chlorine and bromine compounds in the stratosphere. These compounds are responsible for the depletion of the ozone layer.

3

E.

Depletion of the ozone layer: The accumulation of chlorine and bromine compounds in the stratosphere leads to the depletion of the ozone layer. This thinning of the ozone layer allows more ultraviolet (UV) radiation from the sun to reach the Earth's surface. A. Ultraviolet radiation causes skin cancer: The increased exposure to UV radiation, resulting from the depletion of the ozone layer, is a risk factor for developing skin cancer. 9. Elasticity of Demand is given by the formula. Q P ∆Q P × × (1) (2) ∆Q ∆P Q ∆P (3)

∆P ∆Q × P Q

(4)

∆Q Q × P P

Ans. Option (2) is correct. Explanation: Elasticity of Demand = ∆Q P × Q ∆P Where: DQ represents the change in quantity demanded, Q represents the original quantity demanded, DP represents the change in price, and P represents the original price. The formula measures the responsiveness or sensitivity of quantity demanded to changes in price. By calculating the percentage change in quantity demanded divided by the percentage change in price, we can determine whether demand is elastic (greater than 1), inelastic (less than 1), or unitary elastic (equal to 1). 10. C=40+0.8Y and I = 10, then what will be the equilibrium level of income? (1) – 300 (2) – 250 (3) 300 (4) 250 Ans. Option (4) is correct. Explanation: To find the equilibrium level of income, we need to set the total spending (aggregate expenditure) equal to the total income in the economy. In this case, the total spending consists of consumption (C) and investment (I): Total spending = C + I Given that C = 40 + 0.8Y and I = 10, we can substitute these values into the equation: Total spending = (40 + 0.8Y) + 10 = 50 + 0.8Y To find the equilibrium level of income, we set total spending equal to total income: Y = Total spending Equating the two expressions: Y = 50 + 0.8Y Simplifying the equation: 0.2Y = 50, Y = 50 / 0.2, Y = 250 Therefore, the equilibrium level of income is 250.

4

OSWAAL CUET (UG) Sample Question Papers, ECONOMICS

11. Which of the following is not true about the benefits of education? (1) Increases skewed income distribution (2) Raises standard and quality of living (3) Increases participation rate (4) Develops mental abilities Ans. Option (1) is correct. Explanation: Increases skewed income distribution: Education, particularly higher education, has been associated with greater income opportunities and higher earning potential. However, education also plays a crucial role in reducing income inequality. It provides individuals with knowledge, skills, and qualifications that can help them access better job opportunities and higherpaying professions. Education can empower individuals from disadvantaged backgrounds and contribute to a more equitable society by narrowing the income gap. 12. Disadvantages of casualisation of workforce: A. No Paid leave B. No regular income C. No access to training D. Permission to take leave is not required E. Less working hours Choose the correct answer from the options given below: (1) A, C and E only (2) B. C and D only (3) A, B and C only (4) C, D and E only Ans. Option (3) is correct. Explanation: A. No Paid leave: Casual workers often do not receive paid leave benefits, such as vacation leave or sick leave. They may not have access to paid time off, which can affect their income and work-life balance. B. No regular income: Casual workers typically do not have a consistent or stable income. Their work hours and income can fluctuate, making it challenging to predict or plan their finances. C. No access to training: Casual workers may not have access to the same training and skill development opportunities as permanent or full-time employees. This can limit their ability to acquire new skills, advance their careers, or adapt to changing job requirements. 13. Match List I with List II: List -I

List - II

(A)

Product will not (I) pass through further transformation

Capital Good

(B)

A product used (II) as raw material or input for production

Final Good

(C)

The good which (III) enables for production on process

Consumer Good

(D)

Goods purchased by ultimate consumer

Intermediate Goods

(IV)

Choose the correct answer from the options given below: (1) (A)-(III), (B)-(I), (C)-(IV), (D)-(II) (2) (A)-(II), (B)-(IV), (C)-(I), (D)-(III) (3) (A)-(I), (B)-(III), (C)-(II), (D)-(IV) (4) (A)-(IV), (B)-(II), (C)-(III), (D)-(I) Ans. Option (2) is correct. Explanation: A. "Product will not pass through further transformation" corresponds to II. Final goods are products that are ready for consumption or use by end consumer. B. "A product used as raw material or input for production" corresponds to IV. Intermediate goods are products or materials used in production process but do not become part of the final consumer. C. "The good which enables for production on process" corresponds to III. Capital Goods. Capital goods are long-lasting goods that are used in the production process to produce other goods and services. D. "Goods purchased by the ultimate consumer" corresponds to I. Consumer Goods. Consumer goods are products that are bought and used by individuals for personal consumption. 14. In China a campaign intiated in 1958 aimed at industrialising country on a massive scale. People were encouraged to set up industries in their backyard. This campaign is called. (1) Great Show Forward (2) Great Dragon Forward (3) Great Run Forward (4) Great Leap Forward Ans. Option (4) is correct. Explanation: The Great Leap Forward was a socio-economic campaign launched by the Chinese Communist Party under the leadership of Mao Zedong. It was introduced with the goal of rapidly transforming China from an agrarian society to an industrialised nation. The campaign emphasized collective farming and the establishment of small-scale backyard industries to boost agricultural and industrial production. However, the Great Leap Forward ultimately resulted in widespread economic and humanitarian crises, including a significant decline in agricultural output, famine, and the loss of millions of lives. The campaign's ambitious targets and policies led to mismanagement, inefficiencies, and unintended consequences, ultimately undermining its intended goals.

CUET Board Paper 2023 15. Arrange the following in order of occurrence. A. Green Revolution B. Setting up of Planning Commission C. Resolution which formed on the basis of second Five Year Plan D. Exchange rate was made fully convertable for Current Acccounts E. Karve Committee Choose the correct answer from the options given below: (1) B, C, E, A, D (2) B, E, C, A, D (3) B, E, C, D, A (4) B, C, E, D, A Ans. Option (2) is correct. Explanation: B. Setting up of Planning Commission: The Planning Commission was established in India in 1950 to formulate and oversee the country's Five-Year Plans, which aimed at achieving planned economic development. E. Karve Committee: The Karve Committee, also known as the Rural Credit Survey Committee, was formed in 1951. It focused on the study of rural credit needs and suggested measures to improve rural credit availability. C. Resolution which formed on the basis of the second Five-Year Plan: The resolution based on the second Five-Year Plan was passed in 1956. The second Five-Year Plan was the economic development plan implemented from 1956 to 1961. A. Green Revolution: The Green Revolution refers to a series of initiatives and technological advancements in agriculture that took place in the 1960s and 1970s. These initiatives aimed at increasing agricultural productivity through the introduction of high-yielding varieties of crops, irrigation systems, and modern farming techniques. D. Exchange rate was made fully convertible for Current Accounts: The exchange rate being made fully convertible for Current Accounts signifies the liberalization of foreign exchange regulations. This step was taken as part of economic reforms in India during the 1990s to promote international trade and investment. 16. “Increase in price of good ‘x’ leads to increase in demand of good “Y”. How the goods are related to. (1) Complementary goods (2) Normal goods (3) Inferior goods (4) Substitute goods Ans. Option (4) is correct. Explanation: In the given statement, an increase in the price of good 'X' leads to an increase in the demand for good 'Y'. This indicates that the two goods, X and Y, are substitutes for each other. Substitute goods are products that can be used as alternatives or replacements for each

5

other. When the price of one substitute good increases, consumers tend to shift their demand towards the other substitute good, resulting in an increase in its demand. For example, if X and Y are two brands of smartphones, an increase in the price of brand X might lead consumers to switch their preference and demand more of brand Y instead. 17. Suppose an Individual buys 15 units of good when its price is ` 5 per unit. What will happen to his demand when price of the good increases to ` 7 per unit and elasticity of demand for the good is 0.5. (1) Increase his demand (2) No change in his demand (3) Reduce his demand (4) Shift to other Good Ans. Option (3) is correct. Explanation: As per the law of demand, price and quantity demanded are inversely related., keeping other factors constant. When the price rises demand contracts or reduces. This leads to movement along the demand curve. When there is a change in demand due to other factors, it leads to either increase or decrease in demand. This leads to shift in demand curve. 18. Match List I with List II: List -I

List - II

(A)

Regular salaried (I) employees

Common form of unemployment in rural India

(B)

Self employed

(II)

Assurance of minimum 100 days employment

(C)

MGNREGA

(III)

Who own and operate an enterprise to earn their livelihood

(D)

Disguised unemployment

(IV)

Permanent workers

Choose the correct answer from the options given below: (1) (A)-(IV), (B)-(II), (C)-(I), (D)-(III) (2) (A)-(II), (B)-(III), (C)-(I), (D)-(IV) (3) (A)-(IV), (B)-(III), (C)-(II), (D)-(I) (4) (A)-(I), (B)-(II), (C)-(IV), (D)-(III) Ans. Option (3) is correct. Explanation: A. "Regular salaried employees" corresponds to IV. Permanent workers. Permanent workers are individuals who are employed in regular, salaried positions and have long-term job security. B. "Self-employed" corresponds to III. Who own and operate an enterprise to earn their livelihood. Self-employed individuals are those who own and operate their own businesses or work as freelancers or independent contractors.

6

OSWAAL CUET (UG) Sample Question Papers, ECONOMICS C. "MGNREGA" corresponds to II. Assurance of minimum 100 days employment. MGNREGA (Mahatma Gandhi National Rural Employment Guarantee Act) is a social security and job guarantee program in India that provides a legal guarantee of 100 days of employment per year to rural households. D. "Disguised unemployment" corresponds to I. Common form of unemployment in rural India. Disguised unemployment refers to a situation where more people are engaged in a particular activity or occupation than are actually required. It is often observed in rural areas where the productivity of each worker is low, and additional workers do not contribute significantly to output.

19. If C= X+0.6Y & Investment is ` 1,500 Equilibrium level of income in economy would be 4050, find out the autonomous consumption. (1) – 120 (2) 100 (3) 120 (4) – 100 Ans. Option (3) is correct. Explanation: It is given that



C = X + 0.6Y equilibrium level of income = ` 2050 At equilibrium



C+ I = Y

C + 1500 = 4050

C = 4050 – 1500 = 2550 Consumption equation



C = X +0.6 (4050)

2550 = X + 2430 X = ` 120

20. Match List I with List II: List -I

List - II

(A)

Growth in GDP (I) without generating employment

Formal sector

(B)

Process of moving (II) self employment and regular salaried to casual wage work

Jobless Growth

(C)

All establishments (III) which employ 10 hired workers or more

Casualization of work force

(D)

Increases in propor- (IV) tion of work force in informal sector

Informalisation of work force

Choose the correct answer from the options given below: (1) (A)-(II), (B)-(I), (C)-(IV), (D)-(III) (2) (A)-(II), (B)-(III), (C)-(I), (D)-(IV)

(3) (A)-(III), (B)-(II), (C)-(I), (D)-(IV) (4) (A)-(II), (B)-(I), (C)-(III), (D)-(IV) Ans. Option (2) is correct. Explanation: A. Growth in GDP without generating employment - II. Jobless Growth This refers to the situation where the GDP of a country increases, but it does not lead to a significant increase in employment opportunities. B. Process of moving self-employment and regular salaried to casual wage work - III. Casualization of work force. This describes the transition from stable self-employment or regular salaried jobs to casual or temporary wage work. C. All establishments which employ 10 hired workers or more - I. Formal Sector This category includes establishments that have 10 or more hired workers and operate within the formal sector, complying with legal regulations and providing regular employment. D. Increases in proportion of workforce in informal sector - IV. Informalization of work force. This refers to the growing proportion of the workforce engaged in the informal sector, which typically lacks legal protection, social security, and job stability. 21. Arrange the following operations in a sequential manner in order to calculate NNPFC from GDPMP A. GDPMP - Net Indirect Taxes B. NDPFC + Net Factor Income From Abroad C. GDPFC - Depreciation Choose the correct answer from the options given below: (1) A, B and C only (2) B, C and A only (3) A, C and B only (4) C, A and B only Ans. Option (3) is correct. Explanation: To calculate NNPFC (Net National Product at Factor Cost) from GDPMP (Gross Domestic Product at market price), we need to follow these steps: A. Net Indirect Taxes: We start with the Gross Domestic Product at Market Prices (GDPMP) and subtract the Net Indirect Taxes from it. Net Indirect Taxes are indirect taxes (such as sales tax or value-added tax) that the government collects from producers and consumers and then transfers back to producers as subsidies or other forms of payments. The result of this step gives us the Gross Domestic Product at Factor Cost (GDPFC ). B. Net Factor Income From Abroad: Next, we take the Gross Domestic Product at Factor Cost (GDPFC) and add the Net Factor Income From Abroad to it. Net Factor Income From Abroad includes net income earned by domestic factors of production (e.g., labor and capital) from foreign countries minus the net income earned by foreign factors of production from the domestic country. The result of this step gives us the Net Domestic Product at Factor Cost (NDPFC).

CUET Board Paper 2023 C. Depreciation: Finally, we take the Net Domestic Product at Factor Cost (NDPFC) and subtract the Depreciation from it. Depreciation represents the wear and tear or reduction in the value of fixed assets used in the production process. The result of this step gives us the Net National Product at Factor Cost (NNPFC). So, the correct sequential order of operations is: A, C, B. 22. In case of an unexpected fall in turnover of a firm, who is producing a certain commodity, there will a situation of .......... . Choose the correct option. (1) Planned accumulation of inventories (2) Unplanned accumulation of inventories (3) Unplanned decumulation of inventories (4) Planned decumulation of inventories Ans. Option (2) is correct. Explanation: When there is an unexpected fall in the turnover of a firm (i.e., the rate at which the firm sells its products), the production of the commodity remains the same as before the fall in demand. However, due to the reduced demand, the firm is unable to sell its products at the anticipated rate. In such a situation, the firm will face a buildup of unsold products or inventory that it had produced but cannot sell immediately. This accumulation of unsold inventories is unplanned because it was not intended or foreseen by the firm. The firm produced the commodities with the expectation that they would be sold, but the actual demand fell short of the projected level. Unplanned accumulation of inventories can create various problems for the firm, such as increased storage costs, holding costs, and potential obsolescence of the unsold products. It also ties up the firm's working capital, which could have been utilized elsewhere if the products were sold as planned. Hence, in the given scenario of an unexpected fall in turnover, the correct option is the "Unplanned accumulation of inventories." 23. In 1970, Pakistan introduced nationalisation of Capital Good industries, but later due to international pressure they shifted the policy to. (1) De marketisation (2) Denationalisation (3) Derecognition (4) Deforming Ans. Option (2) is correct. Explanation: . In 1970, Pakistan introduced a policy of nationalization of Capital Good industries, which meant that the government took control and ownership of major industries involved in the production of capital goods. This policy was part of the broader economic reforms initiated by the government during that time.

7

However, over time, due to various reasons, including international pressure and recognition of the limitations of the nationalization policy, Pakistan decided to change its approach. As a result, the government shifted its policy from nationalization to denationalization. Denationalization (also known as privatization) is the process of transferring ownership and control of previously nationalized industries and enterprises back to the private sector. It involves selling state-owned assets and businesses to private individuals, companies, or investors. The shift towards denationalization was likely driven by the recognition of the inefficiencies and management issues that often accompany state ownership of industries. Additionally, international pressures from various organizations or countries might have influenced Pakistan to adopt market-oriented economic reforms, which often include denationalization as a key component. By denationalizing the previously nationalized industries, Pakistan aimed to promote private sector participation, competition, and efficiency in the economy. It allowed market forces to play a more significant role in the management and growth of industries, which can lead to increased productivity and economic development. Hence, in the given context, the correct option is "Denationalisation" (Option 2). 24. Which of the following are correct statements related to stock and flow? A. Flows are defined over on 1st January B. Flows are defined over a period of time C. Flows and stocks are defined as a mutual understanding D. Stocks are defined from January to December E. Stocks are defined at a particular point of time Choose the correct answer from the options given below: (1) B and E only (2) A and D only (3) C and E only (4) A and B only Ans. Option (1) is correct. Explanation: A. Flows are defined over on 1st January: This statement is incorrect. Flows are not defined specifically on January 1st. Instead, flows are defined over a period of time, and they represent the quantity of something (e.g., money, goods, or services) that moves from one state to another during that period. B. Flows are defined over a period of time: This statement is correct. Flows are quantities that are measured over a specific time interval. For example, the flow of water in a river can be measured in cubic meters per second, indicating the amount of water passing a point in the river per second.

8

OSWAAL CUET (UG) Sample Question Papers, ECONOMICS C. Flows and stocks are defined as a mutual understanding: This statement is incorrect. Flows and stocks are related concepts, but they are not defined as a mutual understanding. Flows refer to the movement or change in quantities over time, while stocks refer to the quantity of something at a specific point in time. D. Stocks are defined from January to December: This statement is incorrect. Stocks are not defined over a specific time period like January to December. Stocks represent the quantity of something at a particular point in time, irrespective of the time period. E. Stocks are defined at a particular point of time: This statement is correct. Stocks represent the quantity of something at a specific point in time. For example, the inventory of goods held by a company at the end of a financial year is a stock measurement. Therefore, the correct statements are B and E, making Option 1 the correct answer.

25. .......... measures are .............. measures intended to correct some of weaknesses that have developed in the Balance of payment. (1) Stabilisation, Short term (2) Structural Reform, Long term (3) Stabilisation, Long term (4) Structural Reform, Short term Ans. Option (1) is correct. Explanation: Stabilisation, Short term: This option refers to short-term measures aimed at addressing immediate imbalances in the balance of payments and stabilizing the economy in the short run. These measures are temporary in nature and include actions like exchange rate adjustments, interest rate changes, fiscal policy adjustments, and temporary import/export restrictions. The objective is to manage shortterm fluctuations and stabilize the balance of payments in the immediate period. However, for long-term sustainability, a combination of short-term stabilization measures and long-term structural reforms is typically needed. 26. Which one is not the objective of government budget? (1) Reallocation of resources (2) Economic stability (3) Increase in regional disparities (4) Economic growth Ans. Option (3) is correct. Explanation: 1. Reallocation of resources: One of the main objectives of the government budget is to reallocate resources in the economy to achieve various economic and social goals. Through taxation and public expenditure, the government can redistribute resources from one sector or group to another to promote equitable growth and development.

2. Economic stability: The government budget plays a crucial role in maintaining economic stability. It can use fiscal policy measures to influence aggregate demand, control inflation, and manage business cycles. By adjusting taxes and government spending, the government can stabilize the economy during periods of boom or recession. 3. Increase in regional disparities: This is not an objective of the government budget. In fact, government budgets often aim to reduce regional disparities by directing resources to underdeveloped or marginalized regions to promote balanced economic growth across the country. 4. Economic growth: The government budget can be used to foster economic growth. By investing in infrastructure, education, healthcare, and research and development, the government can create an environment conducive to economic growth and productivity enhancement. So, the correct answer is Option 3: "Increase in regional disparities." 27. Which of the following statement is true about Autonomous transactions? (1) These items are done to correct the imbalance of BOP Account (2) These are also known as below the line item (3) These only occur in capital A/c of BOP (4) BOP is said to be in surplus if autonomous receipts are greater than autonomous payments Ans. Option (4) is correct. Explanation: Autonomous items are undertaken for an economic motives. These items may lead to surplus or deficit in BOP. These are not undertaken or influenced by the status of BOP account. These are known as above the line items. When autonomous receipts are greater than autonomous payments it leads to surplus in BOP. On the other hand, in case of autonomous payments being greater than autonomous payment it leads to deficit. 28. A farmer want to take credit to meet his expenditure in agriculture inputs. Which of the following is not an institution he may approach for the credit ? (1) Regional Rural Banks (RRBs) (2) National Bank for Agriculture and Rural Development (3) Commercial Banks (4) Land Development Banks Ans. Option (2) is correct. Explanation: A farmer may approach the following institutions for credit to meet his expenditure in agricultural inputs: l Regional Rural Banks (RRBs) l Commercial Banks l    Land Development Banks National Bank for Agriculture and Rural Development (NABARD) is not an institution a farmer would typically approach for shortterm credit to meet agricultural input expenses.

CUET Board Paper 2023 NABARD is a specialized development bank that provides long-term credit and support for agricultural and rural development projects, but it does not directly offer short-term credit for day-to-day agricultural input expenses. Farmers usually approach commercial banks, regional rural banks, or cooperative banks for short-term agricultural credit needs. 29. Identify the symbol that is used to denote autonomous consumption. (1) C (2) C (3) DC (4) MPC Ans. Option (1) is correct. Explanation: Autonomous consumption is denoted by the symbol C or sometimes represented as "Co". It represents the level of consumption that exists even when income is zero or when there is no disposable income available. Autonomous consumption is independent of income and represents the minimum level of consumption that individuals or households undertake regardless of their income level. 30. How will the earthquake in Turkey affect the production possibility curve of their economy? (1) Production Possibility curve will shift rightward (2) Production Possibility curve will shift leftward (3) No change in Production Possibility curves (4) Production Possibility Curve can shift leftward and rightward both Ans. Option (2) is correct. Explanation: The impact of an earthquake on the production possibility curve of an economy can vary depending on several factors: 1. Immediate negative impact: In the short term, an earthquake can disrupt production and damage infrastructure, leading to a decrease in the economy's productive capacity. This would cause the production possibility curve to shift leftward, indicating a reduction in the economy's potential output. 2. Rebuilding and recovery: In the aftermath of an earthquake, the government and private sector may invest in rebuilding and recovery efforts. This can lead to an increase in capital investment and technological improvements, potentially shifting the production possibility curve rightward, indicating an increase in the economy's productive capacity compared to the preearthquake levels. 3. Long-term effects: Depending on the scale of the earthquake and the effectiveness of recovery efforts, the long-term impact on the economy's productive capacity can vary. If the recovery efforts are successful and lead to better infrastructure and increased resilience, the economy may experience long-term growth and a rightward shift in the production possibility curve. However, if the recovery is insufficient or if there are lasting damages, the economy may experience a more persistent reduction in its productive capacity, leading to a long-term leftward shift in the production possibility curve.

9

31. In what ways labour laws enable the workers to protect their rights. A. Providing more holidays B. Protection of wages C. Providing unemployment allowances D. Redress of grievances (compensation for loss) E. Right to organise and form trade unions Choose the correct answer from the options given below: (1) A, C and D only (2) B, D and C only (3) A, B and C only (4) B, D and E only Ans. Option (4) is correct. Explanation: Labor laws enable workers to protect their rights in the following ways: B. Protection of wages: Labor laws ensure that workers receive fair wages for their work and that employers cannot exploit them by underpaying or withholding their wages. D. Redress of grievances (compensation for loss): Labor laws provide mechanisms for workers to seek redress for any grievances or injustices they face in the workplace, such as unfair termination, workplace discrimination, or unsafe working conditions. Compensation may be awarded to workers in cases of proven loss or harm. E. Right to organize and form trade unions: Labor laws grant workers the right to organize themselves into trade unions or labor associations. These unions can collectively bargain with employers for better wages, working conditions, and benefits, giving workers a stronger voice in the decision-making process. 32. Which of the following statements is true. If exchange rate changes from $1 = ` 72 to ` 81. A. Import will fall B. Import will rise C. Exports will rise D. National Income will rise E. National Income will fall Choose the correct answer from the options given below: (1) B, C and D only (2) A, C and D only (3) B, C and E only (4) A, C and E only Ans. Option (2) is correct. Explanation: When the exchange rate changes from $1 = ` 72 to ` 81: B. Import will rise: A higher exchange rate means that the domestic currency (`) has depreciated against the foreign currency ($). As a result, imports become relatively cheaper for the domestic economy. With a more favorable exchange rate, people and businesses will find it cheaper to import goods and services from foreign countries. C. Exports will rise: On the other hand, a depreciation of the domestic currency makes the country's goods and services relatively cheaper for foreign buyers. This can lead to an increase in exports, as foreign consumers find the domestically produced goods more affordable.

10

OSWAAL CUET (UG) Sample Question Papers, ECONOMICS E. National Income will fall: A depreciation of the domestic currency can lead to a rise in import costs. When imports become more expensive, it can increase the cost of production for domestic industries that rely on imported inputs. This can lead to a decrease in the overall production and output, potentially leading to a decline in the national income.

33. The following is a part of Agriculture marketing system. A. Minimum Support Price B. Buffer Stock C. Micro Credit Programmes D. Public Distribution System Choose the correct answer from the options given below: (1) A and B only (2) A and D only (3) B, C and D only (4) A, B and D only Ans. Option (4) is correct. Explanation: Agriculture marketing system involves various measures and programs to support farmers and ensure food security. The correct descriptions of the options are as follows: A. Minimum Support Price (MSP): MSP is the price at which the government buys agricultural commodities from farmers to support them and provide them with a minimum guaranteed price for their produce. It is aimed at ensuring price stability and protecting farmers from market fluctuations. B. Buffer Stock: Buffer stock refers to the stock of food grains (such as rice and wheat) procured by the government through MSP operations. This stock is maintained to stabilize food prices in the market and ensure adequate food supply during times of scarcity or emergencies. D. Public Distribution System (PDS): PDS is a government-run system that distributes essential commodities, especially food grains, at subsidized prices to the economically weaker sections of society. It aims to ensure food security and prevent hunger and malnutrition. C. Micro Credit Programmes: Micro-credit programs are not directly related to the agriculture marketing system. They are financial programs that provide small loans and credit facilities to low-income individuals and groups, including farmers, to support their livelihood activities. 34. People demand foreign exchange because: (1) They want to send gifts abroad (2) The foreigners send gift in our country (3) They want to purchase imported goods from our market (4) The foreigners like to purchase goods from our market Ans. Option (1) is correct. Explanation: People demand foreign exchange because they want to send gifts abroad. When individuals or businesses in a country wish to send gifts or make payments to someone in a foreign country, they often need

to do so in the currency of that foreign country. This means they must acquire foreign exchange by exchanging their domestic currency for the foreign currency required to send the gifts or make the payments. 35. The Process of moving from self -employment and regular salaried employment to casual wage work is known as: (1) Casualisation of work force (2) Regularisation of casual wage (3) Non Salaried Force (4) Self employment Force Ans. Option (1) is correct. Explanation: The process of moving from selfemployment and regular salaried employment to casual wage work is known as "Casualisation of the workforce." Casualisation of the workforce refers to the increasing trend of employers hiring workers on a casual or temporary basis rather than providing them with permanent, regular employment. Casual workers are often hired on short-term contracts or for specific projects, and they do not receive the same job security, benefits, or protections as regular employees. This trend has been observed in various industries and sectors, where employers seek more flexibility and cost savings by employing workers on a casual or temporary basis. Casualisation can lead to an unstable and precarious work environment for workers, as they may face uncertain working hours, lack of benefits, and limited social protections. 36. The apex body that coordinates the activities of all institution involved in the rural financing system is: (1) National Bank for Allied Activities and Rural Development (2) National Bank for Agriculture and Road Development (3) Reserved Bank of India (4) National Bank for Agriculture and Rural Development Ans. Option (4) is correct. Explanation: NABARD is an apex development bank in India that focuses on providing financial and non-financial assistance to the agriculture and rural sectors. It plays a crucial role in coordinating and regulating the activities of various institutions involved in rural financing, such as regional rural banks, cooperative banks, and rural development banks. NABARD's main objectives include promoting sustainable rural development, boosting agricultural credit, and providing support for agricultural and rural infrastructure projects. It acts as a bridge between the Reserve Bank of India (RBI) and other financial institutions to channel funds effectively to rural areas and promote inclusive growth in the rural economy. NABARD plays a pivotal role in rural financing and development, making it the correct answer in the given options.

CUET Board Paper 2023 37. From the following data, Calculate Primary Deficit. ` (crore) Revenue expenditure 22,250 Capital expenditure 28,000 Revenue receipts 17,750 Capital receipts (net of borrowing) 20,000 Interest payment 5,000 Borrowings 12,500 (1) ` 7,500 (2) ` 12,500 (3) ` 5,000 (4) ` 17,750 Ans. Option (1) is correct. Explanation: Primary Deficit = Fiscal deficit - Interest payments Fiscal deficit = Total expenditure – total receipts excluding borrowings. = (22000 +28000) – (17750 +20000) Capital receipts are already excluding the borrowings. Fiscal deficit = ` 12500 Primary Deficit = 12500 – 5000 = ` 7500 38. Which of the following has highest FDI ? (1) India (2) China (3) Sri Lanka (4) Pakistan Ans. Option (2) is correct. Explanation: Foreign Direct Investment (FDI) inflows can vary over time and depend on various factors, including the country's economic policies, business environment, and geopolitical situation. As of the knowledge cutoff in September 2021, Pakistan may have had higher FDI inflows compared to the other options listed, reflecting its attractiveness to foreign investors during that period. 39. Chronologically arrange the following events. A. Announcement of Demonetisation B. Circulation of Fake Currency C. Rs 500 and 1000 Notes lost legal tender D. Impact on Economic Activities E. New Rs 2000 Currency Note in the circulation Choose the correct answer from the options given below: (1) B, D, A, C, E (2) C, D, B, A, E (3) B, C, D, A, E (4) A, E, B, C, D Ans. Option (1) is correct. Explanation: The chronological order of the events is as follows: B. Circulation of Fake Currency: This refers to the problem of counterfeit currency circulating in the economy before the demonetization move. D. Impact on Economic Activities: This event follows the circulation of fake currency and refers to the effects of demonetization on various economic activities in the country. A. Announcement of Demonetisation: The government's announcement of demonetization, which took place on November 8, 2016.

11

C. Rs 500 and 1000 Notes lost legal tender: Following the announcement, ` 500 and ` 1,000 currency notes lost their status as legal tender, meaning they were no longer accepted as valid means of payment. E. New ` 2000 Currency Note in circulation: After the demonetization move, the government introduced the new ` 2,000 currency notes as part of the currency overhaul. So, the correct chronological order is Option 1: B, D, A, C, E. 40. Choose the correct sequence from the following. A. British introduced railways in India B. Latest population census C. Population explosion according to 2nd stage of demographic transition theory D. Year of “Great Divide” E. First census conducted by British India Choose the correct answer from the options given below: (1) A, B, D, E, C (2) B, A, E, D, C (3) C, A, B, D, E (4) A, E, D, C, B Ans. Option (4) is correct. Explanation: The correct sequence of the events is as follows: A. British introduced railways in India: The British introduced railways in India during the colonial period to improve transportation and facilitate the movement of goods and people. E. First census conducted by British India: The first population census in British India was conducted in 1871. D. Year of "Great Divide": The "Great Divide" refers to the year 1921 when India's population growth rate started to accelerate, marking the transition to a phase of rapid population growth. C. Population explosion according to the 2nd stage of demographic transition theory: The 2nd stage of demographic transition theory is characterized by high birth rates and declining death rates, leading to rapid population growth known as a population explosion. This phase corresponds to the period when India's population growth rate started to increase significantly after 1921. B.

Latest population census: The latest population census is the most recent census conducted to gather comprehensive demographic data of the population. As of the knowledge cutoff in September 2021, the latest population census for India was conducted in 2011. So, the correct sequence is Option 4: A, E, D, C, B.

12

OSWAAL CUET (UG) Sample Question Papers, ECONOMICS

41. The Reserve Bank of India (RBI) recently imposed a monetary penalty on three banks —` 1.50 lakhs on Dr. Ambedkar Nagarik Sahakari Bank Maryadit, ` 25,000 on Nagarik Sahakari Bank Maryadit and ` 1 lakh on Ravi Commercial bank for violating the provisions of its regulations. The banking regulator has imposed a monetary penalty for contravention of non-compliance with the directions issued to Ravi commercial bank an exposure norms and statutory and other restrictions & KYC. Which function of central Bank is referred to in the above paragraph. (1) Financial Advisor (2) Supervisor to Banks (3) Banker’s Bank (4) Lender to last resort Ans. Option (2) is correct. Explanation: The Reserve Bank of India (RBI) acts as the supervisor or regulator of banks in India. One of its key roles is to ensure that banks comply with the regulations and directions issued by the RBI. In the paragraph, the RBI has imposed monetary penalties on three banks for violating its regulations and not complying with the directions issued by the central bank. As a supervisor to banks, the RBI sets prudential norms, exposure limits, statutory restrictions, and KYC (Know Your Customer) guidelines to maintain the stability and soundness of the banking system. It regularly inspects and assesses the financial health of banks to identify any potential risks and takes corrective actions when necessary to protect the interests of depositors and maintain financial stability. By imposing monetary penalties for noncompliance with its regulations, the RBI ensures that banks adhere to the prescribed guidelines and operate in a safe and secure manner. This function helps maintain the integrity of the banking sector and protects the interests of depositors and other stakeholders. 42. The Reserve Bank of India (RBI) recently imposed a monetary penalty on three banks —` 1.50 lakhs on Dr. Ambedkar Nagarik Sahakari Bank Maryadit, ` 25,000 on Nagarik Sahakari Bank Maryadit and ` 1 lakh on Ravi Commercial bank for violating the provisions of its regulations. The banking regulator has imposed a monetary penalty for contravention of non-compliance with the directions issued to Ravi commercial bank an exposure norms and statutory and other restrictions & KYC. As a bankers’ bank, what is the role played by RBI? A. Custodian of cash reserves B. Maintain Foreign Exchange Reserves C. Currency [ssue D. Lender of last resort E. Clearing house Choose the correct answer from the options given below: (1) A, C, D and E only (2) B and C only (3) A only (4) A, D and E only Ans. Option (4) is correct.

Explanation: As a bankers' bank, the Reserve Bank of India (RBI) plays the following roles: A. Custodian of cash reserves: The RBI acts as the custodian of cash reserves of commercial banks. Commercial banks are required to maintain a certain percentage of their deposits as cash reserves with the RBI. These cash reserves serve as a liquidity buffer for banks and help ensure their stability and solvency. D. Lender of last resort: The RBI acts as the lender of last resort for commercial banks. In times of financial distress or liquidity shortages, when banks are unable to borrow from other sources, they can approach the RBI for emergency funds. The RBI provides liquidity support to maintain the stability of the banking system. E. Clearing house: The RBI facilitates the clearing and settlement of interbank transactions. It acts as a central clearing house for the settlement of checks and other financial instruments among different banks. This ensures smooth and efficient clearing of payments and helps in the proper functioning of the payment system. So, the correct answer is Option 4: A, D, and E only. 43. The Reserve Bank of India (RBI) recently imposed a monetary penalty on three banks —` 1.50 lakhs on Dr. Ambedkar Nagarik Sahakari Bank Maryadit, ` 25,000 on Nagarik Sahakari Bank Maryadit and ` 1 lakh on Ravi Commercial bank for violating the provisions of its regulations. The banking regulator has imposed a monetary penalty for contravention of non-compliance with the directions issued to Ravi commercial bank an exposure norms and statutory and other restrictions & KYC. How does RBI act as a supervisor to bank? (1) Banker to government (2) Agent to government (3) Financial advisor to government (4) Inspection & imposing penalty Ans. Option (4) is correct. Explanation: Inspection & imposing penalty: As a banking regulator, the RBI conducts regular inspections and assessments of banks to ensure their compliance with the regulations and guidelines issued by the central bank. During these inspections, the RBI evaluates the financial health, risk management practices, and overall compliance of banks with various prudential norms, exposure limits, statutory restrictions, and KYC guidelines. When banks are found to be in violation of the RBI's regulations or non-compliant with the prescribed guidelines, the RBI has the authority to impose monetary penalties or take other corrective actions. In the paragraph mentioned, the RBI imposed monetary penalties on three banks for contravention of its regulations and directions.

CUET Board Paper 2023 44. The Reserve Bank of India (RBI) recently imposed a monetary penalty on three banks —` 1.50 lakhs on Dr. Ambedkar Nagarik Sahakari Bank Maryadit, ` 25,000 on Nagarik Sahakari Bank Maryadit and ` 1 lakh on Ravi Commercial bank for violating the provisions of its regulations. The banking regulator has imposed a monetary penalty for contravention of non-compliance with the directions issued to Ravi commercial bank an exposure norms and statutory and other restrictions & KYC. If all the deposits, one day come to withdraw all the cash, what would be role played by RBI? (1) Maintaining Foreign Exchange Reserves (2) Lender of last resort (3) Maintaining cash reserves (4) Currency issue Ans. Option (2) is correct. Explanation: If all the depositors of a bank come to withdraw all the cash, leading to a situation where the bank faces a severe liquidity crisis and is unable to meet its obligations, the Reserve Bank of India (RBI) would play the role of "Lender of Last Resort." As the lender of last resort, the RBI provides emergency financial assistance to banks facing liquidity shortages and financial stress. In such a scenario, the RBI would step in to provide liquidity support to the affected bank by lending it funds or by extending credit facilities. This helps the bank meet the withdrawal demands of its depositors and prevents a potential bank run or collapse. The RBI's role as the lender of last resort is crucial for maintaining confidence in the banking system and ensuring financial stability. By providing emergency liquidity, the RBI helps to safeguard the interests of depositors and prevents systemic disruptions in the banking sector. This function helps in maintaining the overall stability of the financial system and prevents contagion effects that could arise from the failure of a single bank. 45. The Reserve Bank of India (RBI) recently imposed a monetary penalty on three banks —` 1.50 lakhs on Dr. Ambedkar Nagarik Sahakari Bank Maryadit, ` 25,000 on Nagarik Sahakari Bank Maryadit and ` 1 lakh on Ravi Commercial bank for violating the provisions of its regulations. The banking regulator has imposed a monetary penalty for contravention of non-compliance with the directions issued to Ravi commercial bank an exposure norms and statutory and other restrictions & KYC. Which of the following is correct statement? (1) RBI act as banker to commercial banks & Central Bank (2) Central Bank creates credit in economy (3) All financial institutions are banking institutions (4) One rupee note and all coins are issued by the Ministry of Finance Ans. Option (4) is correct.

13

Explanation: The statement "One rupee note and all coins are issued by the Ministry of Finance" is correct. In India, the issuance of currency notes and coins is the responsibility of the Government of India, specifically the Ministry of Finance. The Reserve Bank of India (RBI) acts as the central bank and has the authority to issue all other denominations of currency notes except the one rupee note. As per the Coinage Act, 2011, the Government of India has the sole right to issue coins. The RBI plays a vital role in the distribution and management of currency notes and coins in the country, but the actual issuance of one rupee notes and coins is done by the Ministry of Finance. 46. Real GDP is calculated in a way such that goods and services are evaluated at some constant set of prices. Since these prices remain fixed, if the Real GDP changes we can be sure that it is the volume of production which is undergoing changes. Nominal GDP, on the other hand, is simply the value of GDP at the current prevailing prices. When value of final goods and services are Estimated at current price it is? (1) Real GDP (2) Nominal GDP (3) GDP at constant prices (4) Consumer Price Index Ans. Option (2) is correct. Explanation: When the value of final goods and services is estimated at current prevailing prices, it is referred to as "Nominal GDP." Nominal GDP is a measure of a country's economic output without adjusting for inflation or changes in price levels. It reflects the current market value of all goods and services produced within a country's borders during a specific period, considering the prices at which these goods and services are sold in the market at that time. On the other hand, Real GDP is calculated by evaluating goods and services at some constant set of prices (base year prices). This allows for the removal of the impact of price changes and inflation from the GDP calculation, providing a measure of the actual volume of production and economic growth. 47. Real GDP is calculated in a way such that goods and services are evaluated at some constant set of prices. Since these prices remain fixed, if the Real GDP changes we can be sure that it is the volume of Production which is undergoing changes. Nominal GDP, on the other hand, is simply the value of GDP at the current prevailing prices. Real GDP increases only when. (1) Prices increases (2) Prices decreases (3) Production increases (4) Production decreases Ans. Option (3) is correct.

14

OSWAAL CUET (UG) Sample Question Papers, ECONOMICS Explanation: As mentioned in the explanation, Real GDP is calculated by evaluating goods and services at some constant set of prices (base year prices). This means that the impact of price changes or inflation is removed from the GDP calculation, and the focus is solely on changes in the volume of production. When Real GDP increases, it indicates that the total value of goods and services produced in the economy has increased, taking into account changes in the quantity of output without being affected by changes in prices. This increase in production can be the result of various factors, such as technological advancements, increased investments, improved productivity, or expanded economic activities. On the other hand, if Real GDP decreases, it means that the volume of production has declined, independent of price changes.

48. Real GDP is calculated in a way such that goods and services are evaluated at some constant set of prices. Since these prices remain fixed, if the Real GDP changes we can be sure that it is the volume of Production which is undergoing changes. Nominal GDP, on the other hand, is simply the value of GDP at the current prevailing prices. Keeping output constant nominal GDP increases because. (1) Prices increase (2) Prices decrease (3) Output decrease (4) Prices increase and output increase Ans. Option (1) is correct. Explanation: When output (the volume of production) is kept constant, Nominal GDP increases because of price increases. Nominal GDP is calculated by valuing goods and services at the current prevailing prices. If the volume of production (output) remains constant, any increase in Nominal GDP can only be attributed to changes in prices. When the prices of goods and services rise over time, the total value of production increases, leading to an increase in Nominal GDP. This can occur due to inflation, where the general price level in the economy increases, and individual goods and services become more expensive. 49. Real GDP is calculated in a way such that goods and services are evaluated at some constant set of prices. Since these prices remain fixed, if the Real GDP changes we can be sure that it is the volume of Production which is undergoing changes. Nominal GDP, on the other hand, is simply the value of GDP at the current prevailing prices. Which GDP is the indicator of growth and development of a country? (1) Nominal GDP (2) National Income (3) Real GDP (4) Domestic Income Ans. Option (3) is correct.

Explanation: Real GDP is the indicator of growth and development of a country. Real GDP is calculated by evaluating goods and services at some constant set of prices, also known as base year prices. By doing so, the impact of price changes or inflation is removed from the GDP calculation. This allows us to isolate changes in the volume of production and measure actual economic growth over time. Real GDP reflects the true economic output of a country, unaffected by fluctuations in prices. It provides a measure of the increase in the overall production of goods and services in an economy. When Real GDP increases, it indicates that the economy is growing and producing more goods and services over time. On the other hand, Nominal GDP (Option 1) is the value of GDP at current prevailing prices, including the impact of price changes. Nominal GDP can increase due to both changes in prices and changes in production. It does not provide an accurate measure of real economic growth because it is influenced by inflation or deflation. 50. Real GDP is calculated in a way such that goods and services are evaluated at some constant set of prices. Since these prices remain fixed, if the Real GDP changes we can be sure that it is the volume of Production which is undergoing changes. Nominal GDP, on the other hand, is simply the value of GDP at the current prevailing prices. If output in an economy is decreasing but GDP of country is increasing which GDP it is: (1) Real GDP (2) Domestic Income (3) GDP at Constant Price (4) Nominal GDP Ans. Option (4) is correct. Explanation: If output in an economy is decreasing but the GDP of the country is increasing, it is Nominal GDP. As mentioned earlier, Nominal GDP is calculated by valuing goods and services at the current prevailing prices. It represents the total monetary value of all goods and services produced in an economy without adjusting for changes in price levels. When output in an economy is decreasing (i.e., the volume of production is declining), but the Nominal GDP is increasing, it indicates that the increase in GDP is solely driven by rising prices. In other words, prices are increasing at a faster rate than the decline in production, leading to a higher Nominal GDP. Real GDP, on the other hand, is calculated by evaluating goods and services at some constant set of prices (base year prices). If the output in an economy is decreasing, Real GDP would reflect this decrease in production because changes in prices are not considered in its calculation. 

CUET Question Paper 2022 NATIONAL TESTING AGENCY 18th August 2022

ECONOMICS

Slot-1 [This includes Questions pertaining to domain Specific Subjects only]

SOLVED . Time Allowed: 60 Mins.

Maximum Marks: 200

General Instructions : (i) (ii) (iii) (iv) (v)

This paper consists of 50 MCQs, attempt any 40 out of 50 Correct answer or the most appropriate answer: Five marks (+5) Any incorrect option marked will be given minus one mark (-1) Unanswered/Marked for Review will be given no mark (0) If more than one option is found to be correct then Five marks (+5) will be awarded to only those who have marked any of the correct options (vi) If all options are found to be correct then Five marks (+5) will be awarded to all those who have attempted the question. (vii) Calculator / any electronic gadgets are not permitted. 1.

1. The main aim of economic policies pursued by the colonial government in India was: (A) Development of the Indian economy (B) Protection and promotion of the economy interests of their home country (C) To transform India into exporter of finished goods (D) To promote Indian handicrafts. Sol. Option (B) is correct Explanation: The economic policies pursued by the colonial government in India were concerned more with the protection and promotion of the economic interests of their home country than with the development of the Indian economy. Such policies brought about a fundamental change in the structure of the Indian economy — transforming the country into supplier of raw materials and consumer of finished industrial products from Britain.

2. Which of the following is not a goal of the five year plans? (A) Self reliance

(B) Land ceiling

(C) Equity

(D) Modernisation

Sol. Option (B) is correct Explanation: A plan should have some clearly specified goals. The goals of the five year plans were: growth, modernization, self-reliance,

economic development and equity. The plans were developed by the Planning Commission.

3. In 1991, India approached the ............... and ............... and received $7 billion as loan to manage the economic crises. (A) World Bank, World Trade Organisation (B) International Monetary Fund, OPEC (C) World Bank, International Monetary Fund (D) World Trade Organisation, Monetary Fund

International

Sol. Option (C) is correct Explanation: India approached the International Bank for Reconstruction and Development (IBRD), popularly known as World Bank and the International Monetary Fund (IMF), and received $7 billion as loan to manage the crisis. For availing the loan, these international agencies expected India to liberalise and open up the economy by removing restrictions on the private sector, reduce the role of the government in many areas and remove trade restrictions between India and other countries.

4. Arrange the following in Chronological order: A.

Foundation of World Trade Organisation (WTO)

B.

Establishment of General Agreement on Trade and Tariff (GATT)

C.

India’s New Economic Policy (NEP)

16

OSWAAL CUET (UG) Sample Question Papers, Economics D. Introduction of Goods and Service Tax (GST) (A) D, B, A, C

(B) A, C, B, D

(C) B, C, A, D

(D) C, B, D, A

Sol. Option (C) is correct Explanation: The General Agreement on Tariffs and Trade (GATT) was signed by 23 countries in October 1947, after World War II, and became law on Jan. 1, 1948. The New Economic Policy (NEP) of India was launched in the year 1991 under the leadership of P. V. Narasimha Rao. The World Trade Organization (WTO), created in 1995, is an international institution that oversees the rules for global trade among nations. On July 1st 2017, the Goods and Services Tax implemented in India.

5. Identify the incorrect statement relating to need for government intervention in education and health sector. (A) Expenditure in education and health have long term impact and cannot be easily reversed. (B) Individual consumers of education and health care services do not have complete information about quality of service & cost. (C) Large section India’s population lives below its poverty line. (D) Basic education and health care is not considered as a right of the citizens. Sol. Option (D) is correct Explanation: Human welfare should be increased through investments in education and health even if such investments do not result in higher labour productivity. Therefore, basic education and basic health are important in themselves, irrespective of their contribution to labour productivity. In such a view, every individual has a right to get basic education and basic health care, that is, every individual has a right to be literate and lead a healthy life.

6. The process which involves assembling storage processing, transporting, packaging, grading and distribution of different agriculture products across the country is known as ............... . (A) TANWA (B) Agriculture marketing (C) Diversification (D) Cooperative marketing Sol. Option (B) is correct Explanation: The mechanism through which the goods reach different places depends on the market channels. Agricultural marketing is a process that involves the assembling, storage, processing, transportation, packaging, grading and distribution of different agricultural

commodities across the country. Agricultural marketing has come a long way with the intervention of the government in various forms.

7. Which of the following is not a feature of unorgaised sector. (A) No job security (B) Social security schemes (C) Irregular payment (D) No fixed working hours Sol. Option (B) is correct Explanation: All the public sector establishments and those private sector establishments are called formal sector establishments and those who work in such establishments are formal sector workers. All other enterprises and workers working in those enterprises form the informal sector. Thus, informal sector includes millions of farmers, agricultural labours, owners of small enterprises and people working in those enterprises as also the self-employed who do not have any hired workers. Those who are working in the formal sector enjoy social security benefits. They earn more than those in the informal sector.

8. All the public sector establishments and those private sector establishments which employ 10 hired workers or more are called ............... . (A) Informal sector establishments (B) Formal sector establishments (C) Public sector establishments (D) Private sector establishments Sol. Option (B) is correct Explanation: All the public sector establishments and those private sector establishments which employ 10 hired workers or more are called formal sector establishments and those who work in such establishments are formal sector workers. Those who are working in the formal sector enjoy social security benefits. They earn more than those in the informal sector.

9. Distribution of employment by gender.

CUET Solved Paper, 2022

17

(A) Increasing the productivity of factors of production and improving the quality of life of its people. (B) By improving quality of life and increasing transportation facilities. (C) Increasing housing facilities. (D) Increasing banking facilities and educational opportunities. Observe the pie chart and identify the major source of livelihood for both men and women. (A) Casual wage labourers (B) Hired workers (C) Self employed (D) Regular salaried employees Sol. Option (C) is correct

Explanation As can be seen in the two pie charts, about 51 per cent male and 56 per cent female workforce in India belongs to self employed which is more than the casual workers and Regular salaried employees combined together.

10. AYUSH stands for: (A) Alloevera, Yoga, Unity, Siddha, Naturopathy and Homeopathy (B) Ayurveda, Yin Yan, Unani, Spirituality and Homeopathy (C) Ayurveda, Yoga, Unani, Siddha, Naturopathy and Homeopathy (D) Allovera, Yin Yan, Unani, Siddha and Homeopathy Sol. Option (C) is correct Explanation: Indian Systems of Medicine (ISM) includes six systems—Ayurveda, Yoga, Unani, Siddha, Naturopathy and Homeopathy (AYUSH). 11. An indicator used by experts to gauge the number of people dying prematurely due to a particular disease as well as the number of years spent by them in a state of disability owing to the disease is called ............... . (A) Grief of burden of disease (B) Goal burden of disease (C) Global burden of disease (D) Great burden of disease Sol. Option (C) is correct Explanation: Global burden of disease (GBD) is an indicator used by experts to gauge the number of people dying prematurely due to a particular disease, as well as, the number of years spent by them in a state of ‘disability’ owing to the disease. A 2017 study shows nearly twothirds of GBD, now known as Total Burden of Disease was caused by non- communicable diseases associated with heart, respiratory system – lungs, kidney, obesity and lifestyle. 12. Infrastructure contributes to economic development of a country by:

Sol. Option (A) is correct Explanation: Infrastructure contributes to economic development of a country both by increasing the productivity of the factors of production and improving the quality of life of its people. Inadequate infrastructure can have multiple adverse effects on health. Improvements in water supply and sanitation have a large impact by reducing morbidity (meaning proneness to fall ill) from major waterborne diseases and reducing the severity of disease when it occurs. In addition to the obvious linkage between water and sanitation and health, the quality of transport and communication infrastructure can affect access to health care. Air pollution and safety hazards connected to transportation also affect morbidity, particularly in densely populated areas. 13. The ability of the environment to absorb degradation is called: (A) Carrying capacity (B) Global warming (C) Ozone depletion (D) Absorptive capacity Sol. Option (D) is correct Explanation: The rising population of the developing countries and the affluent consumption and production standards of the developed world have placed a huge stress on the environment in terms of its functions. Many resources have become extinct and the wastes generated are beyond the absorptive capacity of the environment. Absorptive capacity means the ability of the environment to absorb degradation. 14. Arrange the following in sequence to show how supply-demand reversal of environment resources happen. A. The demand for resources for both production and consumption went beyond the rate of regeneration of resources. B. Before industrialisation and increase in population, demand for environmental resource was less than their supply. C. There was reversal of supply demand relationship. D. Industrial Revolution and population explosion took place. (A) A, B, C, D (B) B, D, A, C (C) C, B, D, A Sol. Option (B) is correct

(D) B, C, D, A

OSWAAL CUET (UG) Sample Question Papers, Economics

18

Explanation: In the early days when civilisation just began, or before this phenomenal increase in population, and before countries took to industrialisation, the demand for environmental resources and services was much less than their supply. This meant that pollution was within the absorptive capacity of the environment and the rate of resource extraction was less than the rate of regeneration of these resources. Hence environmental problems did not arise. But with population explosion and with the advent of industrial revolution to meet the growing needs of the expanding population, things changed. The result was that the demand for resources for both production and consumption went beyond the rate of regeneration of the resources; the pressure on the absorptive capacity of the environment increased tremendously, this trend continues even today. Thus, what has happened is a reversal of supply-demand relationship for environmental quality.

15. Identify the correct sequence for computing National Income: A.

Add NFIA to calculate NNPFC

B.

Add sales and change in stock to get value of output (VO)

C.

Value of intermediate consumption to be deducted from value of output to calculate GVA at MP.

Explanation: A normal resident is a person who ordinarily resides in a country and whose center of economic interest also lies in that particular country. Normal residents include both nationals (such as Indians living in India) and foreigners (non-nationals living in India). For example, Nepalese living in India for more than one year and performing economic activities of production, consumption and investment in India, will be treated as normal residents of India.

17. Sales for a firm is calculated as: (A) Value of output + Intermediate consumption (B) Value of output – Intermediate consumption (C) Value of output + Change in stock (D) Closing stock + Value of output Sol. Option (C) is correct Explanation: Value of output can be expressed as sum of sales and change in stock because output is either sold or accumulated as unsold stock.

18. Arrange the following steps in calculation of National Income, by Income Method in correct sequence: A.

Add net factor income form abroad to arrive at National Income

B.

Estimate the factor income paid by each sector

C.

Identify and classify the income generating units

D. Deduct value of depreciation and Net Indirect Taxes to get NVAFC.

D. Calculating domestic factor income

Choose the correct answer from the options given below:

Choose the correct answer from the options given below:

(A) C, D, A, B

(B) B, C, D, A

(A) B, D, A, C

(B) D, B, C, A

(C) D, C, B, A

(D) C, B, A, D

(C) C, B, D, A

(D) C, A, B, D

Sol. Option (C) is correct

Sol. Option (B) is correct Explanation: Net value added of each producing unit of the economy is estimated from their gross value of output which is calculated by multiplying total volume of goods produced with their prices. After deducting the sum of value of intermediate goods (IG), depreciation and net indirect taxes (NIT) from value of output we get net value added at FC of the producing units. Net National Product at factor cost is obtained by adding net factor income from ROW to net domestic product at factor cost.

16. A person whose stay in a country is at least for one

Explanation: Income method is used for measuring national income at distribution level. According to this method, national income is estimated by adding incomes earned by all the factors of production for their factor services during a year. To calculate National Income by Income method, we have to add different factor incomes from the economy. The addition of all these factor incomes gives us the calculation by the National Income, i.e. Net Domestic Product at FC (NDPFC).

19. Which of the following are not macroeconomics

year and whose economic interest lies in that country is called a:

variables: A.

Aggregate output

(A) Non-resident

(B) Normal-resident

B.

General price level

(C) Normal citizen

(D) Domestic citizen

C.

Employment of workers in a leather factory.

Sol. Option (B) is correct

D. Producer’s equilibrium

CUET Solved Paper, 2022 Choose the correct answer wheat from the options given below: (A) A and B only

(B) B and C only

(C) C and D only

(D) A and D only

IV. Percentage of deposits which a bank must keep as reserves in liquid form with itself.

(A) A-I, B-II, C-III, D-IV

Sol. Option (C) is correct Explanation: Microeconomics is the study of economics at an individual, group, or company level. Whereas, macroeconomics is the study of a national economy as a whole. Microeconomics focuses on issues that affect individuals and companies. Macroeconomics focuses on issues that affect nations and the world economy. Aggregate output, General price level fall into Macroeconomics. Employment of workers in a leather factory, producer equilibrium are examples of microeconomics.

20. Identify which of the following is a flow variable: A.

Inventory

B.

C.

Income

D. Change in inventories

Wealth

(A) A and C only

(B) B and D only

(C) C and D only

(D) A and B only

Sol. Option (C) is correct Explanation: A flow variable is a quantified variable that is measured over a specified period of time. It is time bounded and expressed as per unit of time. National income, investment in the economy and aggregate supply- all are flow variables since they relate to a period of time.

21. Which of the following components is not included in M1 measure of money supply. (A) Currency and coins held by public (B) Demand deposits of public (C) Inter bank deposits of commercial banks (D) Other deposits of banks Sol. Option (C) is correct Explanation: M1 = Currency (notes and coins) with the public + Demand deposits + other deposits held with the Reserve Bank of India.

22. Match the List I with List II List-I

D. Open market operations

19

List-II

A.

CRR

I.

Buying and selling of government securities by RBI from/to the public and commercial banks.

B.

SLR

II.

Percentage of deposits which a bank must keep as reserves with RBI.

C.

Repo rate

III. Rate at which RBI lends money to commercial banks to meet their short term needs

(B) A-II, B-IV, C-I, D-III (C) A-II, B-IV, C-III, D-I (D) A-IV, B-II, C-III, D-I Sol. Option (C) is correct Explanation: Repo rate is the rate at which commercial bank borrow money from the central bank for short period by selling their financial securities to the central bank. Open market operations refer to the policy of sale and purchase of government securities in the open market by the central bank. Cash Reserve Ratio refers to the minimum percentage of a bank’s total deposits, which it is required to keep with the central bank. Commercial banks have to keep with the central bank a certain percentage of their deposits in the form of cash reserves as a matter of law. Statutory Liquidity Ratio refers to minimum percentage of net total demand and time liabilities; which commercial banks are required to maintain with themselves.

23. Match the List I with List II List-I

List-II

A.

Direct tax

I.

Sale of shares of public sector undertaking.

B.

Non-Tax Revenue

II.

Corporate Tax

C.

Indirect Tax

III. Customs duties

D. Capital Receipts

IV. Cash grants-in-aid from foreign countries

Choose the correct answer from the options given below: (A) A-II, B-IV, C-III, D-I (B) A-I, B-II, C-III, D-IV (C) A-III, B-II, C-I, D-IV (D) A-IV, B-III, C-II, D-I Sol. Option (A) is correct Explanation: Non-tax revenue refers to government revenue from all sources other than taxes. These are incomes, which the government gets by way of sale of goods and services rendered by different government departments. A direct tax is paid directly by the same person on whom it has been levied. It means a tax in which impact and incidence of tax falls on the same persons, then it is termed as direct tax. When a liability to pay a tax (Impact of tax) is on one person; and the burden of that tax

OSWAAL CUET (UG) Sample Question Papers, Economics

20

(Incidence of tax), falls on the other person, it is termed as indirect tax. It means a tax in which impact and incidence of tax lie on two different persons, then it is termed as indirect tax. Government receipts that either creates liabilities (of payment of loan) or reduce assets (on disinvestment) are called capital receipts.

24. If marginal propensity to save (MPS) is 0.5 and initial change in investment is Rs. 250 crore, then the final change in income, is: (A) `125 crore

(B) `1250 crore

(C) `600 crore

(D) `500 crore

Total Consumption APC = Total Disposable Income. Marginal Propensity to Save (MPS) measures the increase in savings from a given increase in household income. Change in Savings MPS = Change in Disposable Income. The Marginal Propensity to Consume (MPC) measures the increase in household consumption from an increase in household income. Change in Consumption MPC = Change in Disposable Income.

Sol. Option (D) is correct 1 ∆Y = ∆I × MPS

Explanation:

Average Propensity to Save (APS) measures the total proportion of income that is saved.

1 ∆Y = Rs. 250 × 0.5 = Rs. 500 crores.

25. If the consumption function is C = 300 + 0.6Y, then the savings function will be (A) S = 300 – 0.6Y

27. From the options given below identify the intermediate good/goods. A.

Printer purchased by dealer for supplying in the school.

B.

Car purchased by a cab driver.

C.

Fruits and shopkeeper.

(B) S = (–) 300 + 0.6Y

(C) S = (–) 300 + 0.4 Y (D) S = 300 – 0.4Y Sol. Option (C) is correct Explanation:

Total Savings APS = Total Income

C = c + bY

Savings (S) = Income (Y) – Consumption (C) S =Y−C Saving function: (–)300 + 0.4Y

26. Match the List I with List II List-I

List-II

A.

Marginal propensity to consume

I.

C Y

B.

Marginal to save

propensity

II.

DC DY

C.

Marginal to save

propensity

D. Average propensity to consume

S III. Y IV.

DS DY

Choose the correct answer from the options given below: (A) A-I, B-II, C-III, D-IV (B) A-IV, B-III, C-II, D-I (C) A-III, B-II, C-IV, D-I (D) A-II, B-IV, C-III, D-I Sol. Option (D) is correct Explanation: Average Propensity to Consume measures the proportion of income spent on consumption.

vegetables

purchased

by

D. Smart boards purchased by the University for teaching. Choose the correct answer from the options given below: (A) A and C only

(B) B and D only

(C) B and C only

(D) D only

Sol. Option (A) is correct Explanation: Intermediate goods are those goods which are meant either for reprocessing or for resale. Goods used in the production process during an accounting year are known as intermediate goods. These are non-durable goods and services used by the producers such as raw materials, oil, electricity, coal, fuel etc. and services of hired engineers and technicians etc. Goods which are purchased for resale are also treated as intermediate goods. For example, Rice, wheat, sugar etc. purchased by a retailer/wholesaler.

28. Identify the correct sequence relating to equilibrium in an economy: A.

Inventories in hand with the producers start falling.

B.

This process continued till aggregate demand becomes equal to aggregate supply.

C.

When planned demand exceeds planned output.

CUET Solved Paper, 2022 D. Producers will plan to raise production and employment. E.

It means buyers are planning to buy more goods and services than producers are planning to produce.

Choose the correct answer from the options given below: (A) A, B, C, D, E

(B) E, D, C, B, A

(C) C, E, A, D, B

(D) C, B, A, E, D

Sol. Option (C) is correct Explanation: (i)

When AD > AS, it means that the buyers are planning to buy more goods and services than the producers are planning to produce (i.e., supply). In this situation, inventories start falling and come below the desired level, the producers expand production. This raises the income level, which keeps on rising till AD and AS are equal again.

(ii) When AD < AS, it means that the buyers are planning to buy less than what the producers are planning to produce. As a result, inventories start rising and move above the desired level. So the producers cut back on production and lay off workers. This reduces the income level i.e., AS. This downward trend continues till AD and AS once again become equal.

29. Match the List I with List II List-I

List-II

A.

Investments made abroad

I.

Credit side of current account of BOP

B.

Remittances to a relative staying abroad

II.

Debit side of current account of BOP

Borrowing IMF

from

III. Debit side Capital account of BOP

D. Export of cotton clothes

IV. Credit side of Capital account of BOP.

C.

Choose the correct answer from the options given below: (A) A-III, B-II, C-IV, D-I (B) A-I, B-II, C-III, D-IV (C) A-IV, B-III, C-II, D-I (D) A-II, B-III, C-I, D-IV Sol. Option (A) is correct Explanation: Current account records imports and exports of goods and services and unilateral transfers. A major part of transactions in foreign trade is in the form of export and import of goods

21

(visible items). Payment for import of goods is written on the negative side (debit items) and receipt from exports is shown on the positive side (credit items). Balance of these visible exports and imports is known as balance of trade (or trade balance). Capital account is that account which records all such transactions between residents of a country and rest of the world which cause a change in the asset or liability status of the residents of a country or its government. All transactions relating to borrowings from abroad by private sector and similarly repayment of loans by foreigners are recorded on the positive (credit) side.

30. A ............. function describes the functional relationship between consumption and income. (A) Income (B) Production (C) Consumption (D) Cost Sol. Option (C) is correct Explanation: Consumption function shows the relationship between consumption and the various levels of income. As the income rises, consumption also rises. there is a direct relationship between consumption and income.

31. Excess demand is a situation in which, (A) AD > AS at full employment level (B) AD < AS at full employment level (C) AD > AS at under employment level (D) AD < AS at under employment level Sol. Option (A) is correct Explanation: When in an economy, aggregate demand exceeds aggregate supply at full employment level, the demand is said to be an excess demand.

32. Choose correct statements about utility A.

As consumption increases marginal utility increases. B. As consumption increases marginal utility decreases. C. Utility does not differ from person to person. D. Total utility is the sum of marginal utilities. (A) A and B only (B) B and C only (C) C and A only (D) B and D only Sol. Option (D) is correct Explanation: Total utility is the total psychological satisfaction a consumer obtains from consuming a given amount of a particular good. Alternatively, total utility is the sum of marginal utilities obtained from consumption of successive units of a commodity. Law of diminishing marginal utility states that marginal utility derived from the consumption of a commodity declines as more units of that commodity are consume.

22

OSWAAL CUET (UG) Sample Question Papers, Economics

33. The Great Leap Forward (GLF) campaign in China aimed at Increasing ............... in the country on a massive scale. (A) industrial sector output

36. The means of production are under the control of the State in which type of economy? (A) Capitalist Economy (B) Mixed Economy (C) Socialist Economy

(B) agriculture sector output

(D) Market Economy

Sol. Option (C) is correct

(C) service sector output (D) cyber security Sol. Option (A) is correct Explanation: The Great Leap Forward (GLF) campaign initiated in 1958 aimed at industrialising the country on a massive scale. People were encouraged to set up industries in their backyards. In rural areas, communes were started. Under the Commune system, people collectively cultivated lands. In 1958, there were 26,000 communes covering almost all the farm population.

34. If there is no change in demand of good X with the change in price of good X, then price elasticity of demand for good X is:

Explanation: In a socialist society the government decides what goods are to be produced in accordance with the needs of society. It is assumed that the government knows what is good for the people of the country and so the desires of individual consumers are not given much importance. The government decides how goods are to be produced and how they should be distributed. In principle, distribution under socialism is supposed to be based on what people need and not on what they can afford to purchase.

37. Which of the following is a positive economics statement? (A) People should take COVID-19 vaccines

(A) Perfectly elastic

(B) Perfectly inelastic

(B) Every child should pursue one sport in school

(C) Unitary elastic

(D) Greater than 1

(C) Government provides subsidies. It increases their financial burden

Sol. Option (B) is correct Explanation: The demand for a commodity is called perfectly inelastic when quantity demanded does not change at all in response to change in its prices. Graphically, the demand curve in parallel to y-axis.

35. Identify the normative statement: (A) Govt. has doubled the expenditure education in this year’s budget.

on

(B) Recently RBI has increased repo rate to regulate inflation in economy. (C) Rise in diesel prices causes rise in transportation cost. (D) Environmental regulation must be tough to protect the environment. Sol. Option (D) is correct Explanation: Normative economics deals with prospective or theoretical situations. This division of economics has a more subjective approach. It focuses on the ideological, perspective-based, opinion-oriented statements towards economic activities. The aim here is to summarise the desirability quotient among individuals and quote factors like ‘what can happen’ or ‘what ought to be’. Normative economics statements are subjective and rely heavily on values originating from an individual opinion. These statements are often very rigid and perceptive. Therefore, they are considered political or authoritarian.

(D) Government should provide basic healthcare to the public. Sol. Option (C) is correct Explanation: Positive economics is the stream of economics that has an objective approach, relied on facts. It concentrates on the description, quantification, and clarification of economic developments, prospects, and allied matters. This subdivision of economics relies on objective data analysis and relevant facts and figures. Therefore, it tries to establish a cause-and-effect relationship or behavioral relationship that can help determine as well as test the advancement of economic theories.

38. From the given below items identify the stock variable: (A) Amount of bank deposits as on 31.03.2022 (B) Investment expenditure (C) Number of deaths due to communicable diseases during Jan 2022 (D) Speed of a car. Sol. Option (A) is correct Explanation: Stock refers to any quantity that is measured at a particular point in time. Stock is basically the accumulated or available quantity of any commodity at a particular moment. The measurement of the stock is performed at a particular time and so it indicates the quantity present at that point in time, for instance, March 31, 2020.

CUET Solved Paper, 2022

39. Calculate GDP mp. A.

Private final consumption expenditure = `200 cr B. Indirect Tax = `50 cr C. Government final consumption expenditure = `100 cr D. Net Exports = `50 cr E. Gross Domestic Capital formation = `50 cr F. Depreciation = `20 cr (A) 370 (B) 400 (C) 320 (D) 270 Sol. Option (B) is correct Explanation: Private final consumption expenditure + Government final consumption expenditure + Net exports + Gross domestic capital formation = ₹ 200 + ₹ 100 + ₹ 50 + ₹ 50 = ₹ 400 crores. 40. Which of the following comes under financial sector reforms? (A) Taxation (B) Banking (C) Delicensing (D) Devaluation Sol. Option (B) is correct Explanation: Financial sector reforms includes financial institutions, such as commercial banks, investment banks, stock exchange operations and foreign exchange market. The financial sector in India is regulated by the Reserve Bank of India (RBI). All banks and other financial institutions in India are regulated through various norms and regulations of the RBI. Passage Read the following text and answer: There is an oil refinery which refines crude petroleum and sells it in the market. The output of the refinery is the amount of oil it refines. We can estimate the value added of the refinery by deducting the value of intermediate good used by the refinery from the value of its output. The value added of the refinery will be counted as part of the GDP of the economy. But in carrying out the production the refinery may also be polluting the nearby river. This may cause harm to the people who use the water of the river. Hence their well being will fall. Pollution may also kill fish or other organisms of the river on which fish survive. 41. Which of the following is correct? A. If the GDP of the country is rising, the welfare will always rise. B. Externalities refer to the benefits a firm or an individual cause to another for which they are not paid. C. Non-monetary exchanges can not he excluded in estimation of GDP. D. Adding the value of final goods once leads to problem of double counting

23

Choose the correct answer from the options given below: (A) A and C only (B) C and D only (C) B and C only (D) A and B only Sol. Option (C) is correct Explanation: Many goods and services contributing economic welfare are not included in GDP or Non-Monetary exchanges. There are many goods and services which are left out of estimation of national income on account of practical estimation difficulties e.g., services of housewives and other members, own account production, etc. These are left on account of nonavailability of data and problem in valuation.

42. Value added by the oil refinery will be equal to: (A) Value of output – intermediate cost (B) Value of output + intermediate cost (C) Crude oil only (D) Amount of oil it refines and crude oil Sol. Option (A) is correct Explanation: By value added method, the total value of all the final goods and services produced in an economy during a given time period are estimated to obtain the value of domestic income. The formula by the product method of measuring national income is: Value Added or Value Addition = Value of Output – Intermediate Consumption.

43. Oil refinery polluting nearby area is an example of: (A) Non-monetary exchanges (B) Positive externalities (C) Negative externalities (D) GDP externalities Sol. Option (C) is correct Explanation: A negative externality exists when the production or consumption of a product results in a cost to a third party. Air and noise pollution are commonly cited examples of negative externalities.

44. In the above case study GDP is ............... actual welfare (A) underestimated (B) overestimated (C) accurately estimating (D) equal to Sol. Option (B) is correct Explanation: Though externalities are not taken into account in GDP, they affect welfare. Activities resulting in benefits to others are positive externalities and increase welfare; and those resulting in harm to others are called negative externalities, and thus decrease welfare.

24

OSWAAL CUET (UG) Sample Question Papers, Economics GDP does not take into account negative externalities. For examples, factories produce goods but at the same time create pollution of water and air. The pollution harms people. The factories are not required to pay anything for harming people. Producing goods increases welfare but creating pollution reduces welfare. Therefore, taking only GDP as an index of welfare overstates welfare in this case, welfare is much less than indicated by GDP.

45. A rise in GDP will ............... .

46. Which state has the lowest population of poor in 2011-2012 (A) Madhya Pradesh

(B) Andhra Pradesh

(C) Odisha

(D) Bihar

Sol. Option (B) is correct Explanation: Andhra Pradesh has the lowest population of poor in 2011 – 12. It was 9% in the year 2011 - 2012.

47. Which state has the highest population of poor in 1973-74

(A) reduce welfare

(A) Odisha

(B) Tamil Nadu

(B) always increase welfare

(C) Andhra Pradesh

(D) Rajasthan

(C) may or may not increase welfare (D) not affect welfare Sol. Option (C) is correct Explanation: Since GDP estimates both the economy’s total income and expenditure on goods and services, one may have a question is GDP a good measure of economic welfare or not. Well, GDP cannot be considered as a perfect measure of economic well-being. For instance, if everyone in the economy suddenly starts working every day of the week rather than enjoying leisure periods on holidays. There will be more production of goods and services and hence would rise. Despite the rise of GDP, the loss from reduced leisure can lead to poor quality of products and thus would negate the gain from producing and consuming a greater quantity of goods and services. Therefore, GDP and welfare may not always go hand in hand. It can be a fair assessment of economic well-being but not all purposes. More GDP does not necessarily mean an increase in happiness. But more GDP does mean the measurement of the production of goods and services. Passage On the basis of the given diagram answer:

Sol. Option (A) is correct Explanation: Odisha has the highest population in 1973-74. It was 66% as compared to others states.

48. Which state has the highest population of poor in 2011-12? (A) Odisha

(B) Bihar

(C) Maharashtra

(D) Madhya Pradesh

Sol. Option (D) is correct Explanation: Madhya Pradesh has the highest population of poor in 2011 – 12. It was 36% in the year 2011 – 2012 as compared to others states.

49. Which two states has the same proportion of people below the poverty line in 1973-74. (A) Odisha and West Bengal (B) Gujarat and Andhra Pradesh (C) Bihar and Madhya Pradesh (D) Gujarat and West Bengal Sol. Option (C) is correct Explanation: Bihar and Madhya Pradesh has the same proportion of people below poverty line in 1973-74. It was 62%.

50. ............... state reduced poverty level much better than other states. (A) Tamil Nadu

(B) Odisha

(C) Bihar

(D) Madhya Pradesh

Sol. Option (A) is correct Explanation: Tamil Nadu has reduced poverty level much better than other states. It has reduced from 55% in 1973 – 74 to 11% in 2011 – 12 as compared to other states.

❑❑

SAMPLE

Question Paper Maximum Marks : 200

1

Time : 60 Minutes

General Instructions : (i) This paper consists of 50 MCQs, attempt any 40 out of 50 (ii) Correct answer or the most appropriate answer: Five marks (+5) (iii) Any incorrect option marked will be given minus one mark (-1) (iv) Unanswered/Marked for Review will be given no mark (0) (v) If more than one option is found to be correct then Five marks (+5) will be awarded to only those who have marked any of the correct options (vi) If all options are found to be correct then Five marks (+5) will be awarded to all those who have attempted the question. (vii) Calculator / any electronic gadgets are not permitted.

1. Inventory is a ................... concept whereas the change in inventory is a ...................... concept. (1) stock, flow

(2) flow, stock

(3) stock, stock

(4) flow, flow

2. If Marginal Propensity to Save (MPS) is 0.25 and initial change in investment is `250 crores, then the final change in income would be _____.

3.  Flow of Money 4.  Reserve Money

(c)  Money held by Public and the Bank today (d)  Supply of money studied at a point of time

(1) 1 – (a)

(2) 2 – (b)

(3) 3 – (c)

(4) 4 – (d)

6. Study of population in India is an example of :

(1) `1,000 crores

(2) `1,200 crores

(1) Macroeconomics

(2) Microeconomics

(3) `500 crores

(4) `3,500 crores

(3) Both (a) and (b)

(4) None of these

3. The focus of government budget is to:

7. Which of these is not a property of indifference

(1) maximise fiscal deficit

curve?

(2) minimise fiscal deficit

(1) Indifference curve slopes downwards.

(3) maximise expenditure

(2) Indifference curve is concave to origin.

(4) maximise revenue

(3) Two indifference curves cannot intersect each other.

4. An Indian real estate company receives rent from Google in New York. This transaction would be recorded on ______________ side of _____________ account.

8. Under which market form, a firm is a price taker? (1) Perfect competition

(1) credit, current

(2) Monopoly

(2) debit, capital

(3) Oligopoly

(3) credit, capital

(4) Monopolistic competition

(4) debit, current

5. Identify the correctly matched pair of items in Column A to that of Column B. Column A 1.  Supply of Money 2.  Stock of Money

(4) Higher indifference curve represents higher level of satisfaction.

Column B (a)  Money in circulation at a point (b)  Supply of money considered over a period

9. If the value of Average Propensity to Save (APS) is 0.2 and National Income is `4,000 crores, then consumption will be _______. (1) `4,000 crores (2) `3,200 crores (3) `3,800 crores (4) `2,600 crores

2

OSWAAL CUET (UG) Sample Question Papers, ECONOMICS

10. Which of the following statement is true about

because of:

(1) There is no mobility of factors of production

(1) Externalities.

(2) There is a presence of transportation cost

(2) Composition of GDP and distribution of GDP.

(3) There is no selling cost

(3) Non-monetary transactions.

(4) There is no uniform price

(4) All of the above.

11. Identify the correctly matched pair in Column A to that of Column B: Column A

16. Identify the correct matched pair from Column A to Column B:

Column B

1.  Excise Duty

(a)  Capital Receipts

2.  Income Tax

(b)  Direct Tax

3.  Earning from PSU

(c)  Indirect Tax

4.  Old Age Pensions

(d) Non-Tax Revenue Receipts

(1) 1 – (a)

(2) 2 – (b)

(3) 3 – (c)

(4) 4 – (d)

12. What will be the effect on equilibrium price and equilibrium quantity when income increases in case of normal goods? (1) Both equilibrium price and quantity fall (2) Both equilibrium price and quantity rise (3) Equilibrium price rises and quantity falls (4) Equilibrium price falls and quantity rises

15. GDP is not an appropriate indicator of welfare

perfect competition?

13. Which of the following statements is true? (1) Under employment equilibrium refers to the situation when people who are able and willing to work at the prevailing wage rate do not get employment. (2) Under employment equilibrium refers to the situation when people who are not able but are willing to work at the prevailing wage rate do not get employment. (3) Full employment equilibrium refers to the situation when people who are not able but are willing to work at the prevailing wage rate do not get employment. (4) Full employment equilibrium refers to the situation when people who are able and willing to work at the prevailing wage rate get employment.

14. “Higher indifference curve represents fewer quantities of one or both goods, a higher indifference curve shows higher utility level.” Choose the correct option for the above mentioned statement:

Column A

Column B

(1)  Import of Petroleum from Iran

(a)  Debit side of Current Account

(2)  BPO services provided by India to USA

(b)  Credit Side of Capital Account

(3)  Investment by Saudi Aramco in RIL

(c)  Debit Side of Capital Account

(4)  Export of Spices to Greece

(d) Debit Side of Current Account

(1) 1 – (a) (3) 3 – (c)

(2) 2 – (b) (4) 4 – (d)

17. Identify the correctly matched pair of the items in Column A to that of Column B. Column A

Column B

1.  Money Supply

(a)  Total stock in circulation

2.  Money Creation

(b)  Function of Central Bank

3.  Money Stock

(c)  Function of the Commercial Bank

4.  Money Demand

(d)  Money in the Banks

(1) 1 – (a)

(2) 2 – (b)

(3) 3 – (c)

(4) 4 – (d)

18. When the demand of a commodity is inelastic. (1) Commodity has various uses. (2) Demand remains constant with variable price. (3) Commodity has few uses. (4) Commodity has no uses at all.

19. Area under MC curve is equal to: (1) TVC

(2) AFC

(3) AVC

(4) AC

20. Smoke emitted by a chemical factory and causing air pollution is an example of:

(1) True

(1) Positive externalities

(2) False

(2) Negative externalities

(3) Partially false

(3) Either (1) or (2)

(4) Partially false

(4) Neither (1) nor (2)

Sample Question Papers 21. Which of following is a direct tax? (1) Corporation tax

(2) Entertainment tax

(3) Excise duty

(4) Service tax

(3) Increase in the value of (c) Depreciation domestic currency by the government (4)  Increase in the value (d) Revaluation of domestic currency through market forces

22. Which of the following statements is/are correct: (1) (2) (3) (4)

`1 note is issued by the RBI. Coins are issued by the Government of India. Both statements are true. Both statements are false.

23. Which of the following is not a central problem of

(1) 1 – (a)

(2) 2 – (b)

(3) 3 – (c)

(4) 4 – (d)

28. Which of the following statement is true?

an economy?

(1) When a rise in the price of a commodity leads to decrease in quantity supplied of a commodity, it is called Contraction of Supply.

(1) Why to produce? (2) What to produce?

(2) When a fall in the price of a commodity leads to increase in quantity supplied of a commodity, it is called Contraction of Supply.

(3) How to produce? (4) For whom to produce

24. Which of the following statements is true?

(3) When a fall in the price of a commodity leads to decrease in quantity supplied of a commodity, it is called Contraction of Supply.

(1) Demand curve under monopoly is positively sloped as more quantity can be sold only at a lower price. (2) Demand curve under monopoly is negatively sloped as more quantity can be sold only at a lower price. (3) Demand curve under monopoly is negatively sloped as less quantity can be sold only at a lower price. (4) Demand curve under monopoly is negatively sloped as more quantity can be sold only at a higher price.

25. The study of National Income is related to:

(4) When a rise in the price of a commodity leads to increase in quantity supplied of a commodity, it is called Contraction of Supply.

29. Which of the following can cause an increase in demand: (1) Increase in the price of substitute goods (2) Decrease in the income in case of inferior goods (3) Both of the above (4) None of the above

30. Which of the following statements is true? (1) Monopolistic competitive markets have highly differentiated products.

(1) Microeconomics (2) Macroeconomics

(2) Monopolist markets have highly differentiated products.

(3) Both Micro and Macroeconomics (4) None of the above

(3) Perfect Competition markets have highly differentiated products.

26. For a consumer to recognize how many commodities he/she should buy, which approach is used?

(4) Monopoly markets have highly differentiated products.

(1) Utility Approach (2) Indifference Curve Approach

31. Assertion (A): GDP does not exhibit the structure of

(3) Both of the above

the product.

(4) None of the above



27. Identify the correct pair as given in Column B by matching them with respective concepts in Column A: Column A

3

Column B

(1) Reduction in the value of (a) Devaluation domestic currency by the government Reduction in the value (b) Appreciation (2)  of domestic currency through market forces

Reason (R): If the increase in GDP is mainly due to increased production of war equipment and ammunitions, then such an increase cannot improve welfare in economy. (1) Both Assertion (A) and Reason (R) are true, and Reason (R) is the correct explanation of Assertion (A). (2) Both Assertion (A) and Reason (R) are true, but Reason (R) is not the correct explanation of Assertion (A). (3) Assertion (A) is true, but Reason (R) is false. (4) Assertion (A) is false, but Reason (R) is true.

4

OSWAAL CUET (UG) Sample Question Papers, ECONOMICS

32. Assertion (A): Full employment is the situation where all those workers who are able to work and willing to work get employment at the prevailing wage rate.

Reason (R): The situation of full employment is achieved only when the economy is in equilibrium. (1) B  oth Assertion (A) and Reason (R) are true, and Reason (R) is the correct explanation of Assertion (A). (2) Both Assertion (A) and Reason (R) are true, but Reason (R) is not the correct explanation of Assertion (A). (3) Assertion (A) is true, but Reason (R) is false. (4) Assertion (A) is false, but Reason (R) is true.

33. Identify the correctly matched pair of Column A to that of Column B: Column A 1. Items of imports and exports 2. Outward oriented policy 3. Inward oriented policy 4. Imports exceeds exports

Column B (a)  Import substitution (b) Composition of foreign trade (c)  Export promotion (d) Unfavourable Balance of Trade

(1) 1 – (a) (2) 2 – (b) (3) 3 – (c) (4) 4 – (d)

34. Identify the two cost curves which start from the same point on the Y-axis. (1) TVC and TFC

(2) TFC and TVC

(3) TFC and TC

(4) TFC and AFC

35. Aggregate demand can be decreased by: (1) Rise in Bank Rate. (2) Purchase of securities in Open Market. (3) Deficit Budget.

38. Identify the correctly matched pair of items in Column A to those in Column B: Column A 1. Total of factor payments 2.  Value Added 3.  Festival gifts 4. Final expenditure incurred by the government

Column B (a) Value Added Method (b)  Stock Variable (c) Transfer payments (d) Consumption expenditure

(1) 1 – (a)

(2) 2 – (b)

(3) 3 – (c)

(4) 4 – (d)

39. In the present COVID-19 situation, many economists have raised their concerns that Indian economy may have to face a deflationary situation due to reduced economic activities in the country. Suppose you are a member of the high-powered committee constituted by the Reserve Bank of India (RBI).You have suggested that as the supervisor of commercial banks, ................ (restriction/release) of the money supply be ensured by the Reserve Bank of India (RBI).  (1) Restriction

(2) Release

(3) Either (1) or (2)

(4) Neither (1) nor (2)

40. Which of the following statements is true? (1) Depreciation of the foreign currency leads to the rice in exports. (2) Devaluation of the domestic currency leads to a rise in imports. (3) Appreciation of domestic currency leads to rise in exports. (4) Appreciation of foreign currency leads to fall in imports.

41. Oligopoly is a market structure in which there are : (1) Many sellers

(4) None of the above

36. Which of the following statements is not true regarding the benefit of GST? (1) GST provides comprehensive and wider coverage of input credit set-off, you can use service tax credit for the payment of tax on sale of goods, etc. (2) All indirect taxes in state and central level have been included by GST. (3) Uniformity of tax rates across the states. (4) Ensure better compliance due to aggregate tax rate reduction.

37. Production possibility curve slopes downwards from ..................... to ...................... (1) Left to right

(2) Right to left

(3) Up to down

(4) Down to Up

(2) One seller (3) Few sellers (3) None of these

42. Elasticity of the demand is available when: (1) Low priced commodities are available (2) Uses of a commodity is less (3) Both of the above (4) None of the above

43. If in an economy the value of Net Factor Income from Abroad is ` 200 crores and the value of Factor Income to Abroad is ` 40 crores. Identify the value of Factor Income from Abroad: (1) ` 200 crores (2) ` 160 crores (3) ` 240 crores (4) ` 180 crores

Sample Question Papers 44. When we add up utility derived from consumption of all the units of the commodities, we get: (1) Total utility

(2) Initial utility

(3) Marginal utility

(4) None of these

45. Which of the following statements is correct with respect to the correction of Excess Demand?  (1) Government reduces the taxes. (2) Government reduces its expenditure. (3) The Central Bank reduces bank rates. (4) The Central Bank sells securities from the Open Market.

46. .....................refers to the minimum price, fixed by the government, which is above the equilibrium price.

5

average cost. Even though bank branches in all size categories enjoy economies of scale, the extent of such economies is larger for branches operating at a smaller scale of production. This implies that as the bank branches grow larger in size in terms of both loan and deposit accounts, they move closer to attaining increasing returns to a factor. It is also shown that the marginal costs of servicing both loan and deposit accounts decline as bank branches grow larger in size in terms of either the number of loans or the number of deposits. This confirms that branches operating at a larger scale of production have attained greater cost efficiency in terms of servicing the loan and deposit accounts.

47. Agricultural Development Bank of Pakistan uses

(1) Price floor

the _______ function approach.

(2) Minimum support price

(1) Production

(2) Cost

(3) Both (a) and (b)

(3) Utility

(4) None of the above

(4) None of these I. Read the following hypothetical case study carefully and answer the questions follow on the basis of the same. Agricultural Developmental Bank of Pakistan uses the production function approach for measuring bank outputs and costs. A trans log cost function is estimated to provide an assessment of the bank’s scale and scope efficiency, and to quantify the extent to which its production costs are sensitive to size and output mix. Results shows that the bank enjoys both overall and product-specific economies of scale and, therefore, there exists scope for the bank to expand its operations at declining

48. Economies of scale is larger for which type of bank. (1) Larger scale of production (2) Smaller scale of production (3) Medium scale of production (4) None of the above

49. When the banks grow in size, they move closer in achieving ___________ returns to factor. (1) Increasing

(2) Decreasing

(3) Constant

(4) None of the above

50. The increasing returns to scale in larger branches is due to ___________ (1) Increase in loans

(2) Increase in deposits

(3) Both (A) and (B)

(4) None of them



SAMPLE

Question Paper Maximum Marks : 200

2

Time : 60 Minutes

General Instructions : (i) This paper consists of 50 MCQs, attempt any 40 out of 50 (ii) Correct answer or the most appropriate answer: Five marks (+5) (iii) Any incorrect option marked will be given minus one mark (-1) (iv) Unanswered/Marked for Review will be given no mark (0) (v) If more than one option is found to be correct then Five marks (+5) will be awarded to only those who have marked any of the correct options (vi) If all options are found to be correct then Five marks (+5) will be awarded to all those who have attempted the question. (vii) Calculator / any electronic gadgets are not permitted.

1. Goods purchased for the following purpose are final goods: (1) for satisfaction of wants (2) for investment in firm (3) both (A) and (B) (4) none of the above

2. Supply of money refers to ___________. (1) currency held by the public. (2) currency held by Reserve Bank of India (RBI). (3) currency held by the public and demand deposits with commercial banks. (4) currency held in the government account.

3. If the value of Average Propensity to Consume (APC) is 0.8 and National Income is ` 4,000 crores, the value of savings will be _______. (1) `100 crores (2) `200 crores (3) `800 crores (4) `500 crores

4. The word `Economics’ was derived from two ____ words, ___ and ___. (1) Greek, oikou, nomos (2) French, oikou, nom (3) Spanish, ecoio, nomco (4) Russian, oikou, nomos

5. Primary deficit in a government budget will be zero, when ........................ . (1) Revenue deficit is zero (2) Net interest payments are zero (3) Fiscal deficit is zero (4) Fiscal deficit is equal to interest payment

6. Which of the following is correct?

(1) TU falls, MU = 0

(2) TU falls, MU < 0

(3) TU rises, MU = 0

(4) TU falls, MU > 0

7. Which of the following is the result of increase in price of factors of production? (1) Rightward shift in supply curve (2) Leftward shift in supply curve (3) Expansion in supply (4) Contraction in supply

8. Which of the following is a flow? (1) Deposits in a bank

(2) Capital

(3) Depreciation

(4) Wealth C ∆C 9. Average Propensity to Consume C is equal ∆C to: Y ∆Y C ∆C Y − Y ∆Y (1) (2) C Y ∆Y C−Y ∆Y − ∆C C ∆C− Y C (3) (4) ∆Y − ∆C Y ∆Y Y − ∆C ∆ 10. Exchange Transactions which are CForeign −Y independent of other transactions in the Balance of ∆Y − ∆C Payments Account are called: (1) Current transactions. (2) Capital transactions. (3) Autonomous transactions. (4) Accommodating transactions.

11. In case of perfect competition, AR curve is: (1) (2) (3) (4)

Downward sloping Positively sloped Horizontal straight line Vertical straight line

12. When price falls with rise in output, TR is _____________ when MR is zero. (1) Maximum

(2) Minimum

(3) Zero

(4) None of these

Sample Question Papers 13. Which of the following statements is true? (1) Increase or decrease in demand for a commodity causes change in its price in case the supply of the commodity is perfectly elastic. (2) Increase or decrease in supply for a commodity does not cause any change in its price in case the demand of the commodity is perfectly elastic. (3) Increase or decrease in demand for a commodity does not cause any change in its price in case the supply of the commodity is perfectly elastic. (4) Increase or decrease in demand for a commodity causes changes in its quantity in case the supply of the commodity is perfectly elastic.

14. Which of the following statements is false?

(2) In the managed floating exchange rate system, the government intervenes to buy and sell foreign currencies. (3) In the managed floating exchange rate system, the central bank intervenes to moderate exchange rate fluctuations. (4) In the fixed exchange rate system, market forces fix the exchange rate.

18. Which one of the following is not a determinant of individual supply? (1) Price of the given commodity (2) Taxation policy (3) State of technology (4) Number of firms

19. Identify the market form which has indeterminate demand curve :

(1) Demand deposits are not legal tender.

(1) Perfect competition

(2) Currency notes issued are not legal tender.

(2) Monopoly

(3) Term deposits are not legal tender.

(3) Oligopoly

(4) Wheat is not legal tender.

(4) Monopolistic competition

15. Identify the correctly matched pair of items in Column A to those in Column B: Column A 1. Markets in the Economy 2.  National Income 3.  Total Cost

Column B (a)  Producers only

4. Net National Product

(b) Macroeconomics (c) Both Micro and Macroeconomics (d) Microeconomics

20. What will be the effect on price elasticity of demand, if the time required to find the substitute product is more. (1) Price elasticity will increase. (2) Price elasticity will remain same. (3) Price elasticity will decrease. (4) More information required.

21. Identify the correctly matched pair of the items in Column A to that of Column B

(1) 1 – (a)

(2) 2 – (b)

Column A

Column B

(3) 3 – (c)

(4) 4 – (d)

1.  Money

(a)  Supply of Money

16. Identify the correctly matched pair of the items in Column A to those in Column B: Column A

Column B

1.  Fiscal Deficit

(a) Other than interest payments

2.  Primary Deficit

(b) Borrowings less interest payments

3.  Revenue Deficit

(c) Borrowings

4.  Tax Deficit

(d) Borrowings in government budget

(1) 1 – (a) (2) 2 – (b) (3) 3 – (c)

(4) 4 – (d)

17. Identify which of the following statement is true? (A) The flexible exchange rate system gives the government more flexibility to maintain large stocks of foreign exchange reserves.

7

2.  Commercial Bank (b)  Store of Value 3.  Central Bank

(c)  Credit Control

4.  M4

(d)  Circulation of Money

(1) 1 – (a) (3) 3 – (c)

(2) 2 – (b) (4) 4 – (d)

22. Which of the following statements is not correct? (1) APC is the ratio of consumption expenditure to any particular level of income. (2) MPC is the ratio of a change in consumption to the change in income. (3) APS is the ratio of savings to any particular level of consumption. (4) MPS is the ratio of a change in savings to the change in the income.

23. Identify which of the following statements is true: (1) Durable goods get transformed in production and consumption process.

the

8

OSWAAL CUET (UG) Sample Question Papers, ECONOMICS (2) More sophisticated and heavy capital goods raise the ability of a labourer to produce goods. (3) Intermediate goods do not pass through any more stages of production.

(1) greater than 2 (2) less than 2 (3) equal to 2 (4) equal to 5

29. The marginal physical product of a factor must be _______ when total physical product is falling. (1) Positive

(2) India should spend more money on defence

(2) Zero

(3) Indian economy is a developing economy, the Government should make it developed through correct planning. (4) Increase in real per capita income increases the standard of living of people

(3) Negative (4) None of the above

30. Which of the following affects national income? (1) Goods and Services Tax

25. Which of the following statement is true: (1) In the first stage, Total Product increases at an increasing rate and Marginal Product rises till it reaches its maximum point. (2) In the second stage, Total Product decreases at a decreasing rate and reaches maximum, and MP becomes zero. (3) In the third stage, Total Product increases and Marginal Product (MP) becomes negative. (4) There is a fourth stage of production.

26. Identify the correctly matched pair from column A to that of Column B: Column A

Column B

1. Large number of sellers and a single buyer

(a) Monopoly Market

2. Single seller and large number of buyers

(b) Perfect Competition Market

3. Firms and industries are synonymous

(c) Monopoly Market

4. No close substitutes

(d) Perfect Competition Market

(2) Corporation Tax (3) Subsidies (4) None of the above

31. Assertion (A): Profits of chemical industries increased 150%; fishermen income reduced by 70% due to untreated chemical pollutants in water bodies. This is a negative externality.

Reason (R): The profits of chemical industries is causing pollution which is harming the water and inturn leading the fishermen to catch less fish as the biodiversity of the water body is disturbed. (1) Both Assertion (A) and Reason (R) are true, and Reason (R) is the correct explanation of the Assertion (A). (2) Both Assertion (A) and Reason (R) are true, but Reason (R) is not the correct explanation of the Assertion (A). (3) Assertion (A) is true, but Reason (R) is false. (4) Assertion (A) is false, but Reason (R) is true.

32. Assertion (A): The elastic demand curve for luxuries is flatter than normal.

(1) 1 – (a)

(2) 2 – (b)

(3) 3 – (c)

(4) 4 – (d)

27. Match the items in Column A to those in Column B and choose the correct option: Column A

(4) 4 – (d)

than marginal propensity to save, the value of the multiplier will be:

24. Which of the following is a correct example of a (1) Government should stop Minimum Support Price to the farmers.

(2) 2 – (b)

(3) 3 – (c)

28. If the marginal propensity to consume is greater

(4) Final goods refers to that good which is purchased for the purpose of further production/resale. statement containing both positive and normative economics?

(1) 1 – (a)

Column B

Reason (R): The coefficient of Elasticity ranges between 0 and 1. (1) B  oth Assertion (A) and Reason (R) are true, and Reason (R) is the correct explanation of the Assertion (A). (2) B  oth Assertion (A) and Reason (R) are true, but Reason (R) is not the correct explanation of the Assertion (A). (3) Assertion (A) is true, but Reason (R) is false.

1. GST

(a)  Indirect Tax

2.  Income Tax

(b)  Burden can be shifted

3. Fine

(c)  Direct Tax

between output and cost of production.

4.  Tax Receipts

(d)  Capital Receipt

(1) Production function

(4) Assertion (A) is false, but Reason (R) is true.

33. ___________ shows the functional relationship

(2) Supply function

Sample Question Papers

40. Identify the correct sequence of alternatives given in

(3) Cost function

Column B by matching them with respective Column A:

(4) Demand function

34. Full employment: (1) is consistent with unemployment’.

‘natural

rate

of

(2) occurs when demand for labour force = supply of labour force. (3) AS = AD with zero level of unemployment. (4) both (A) and (B)

35. “The law states that a consumer will so allocate his expenditure so that the utility gained from the last rupee spent on each commodity is equal.” Identify the law defined above. (1) Law of diminishing marginal utility (2) Law of substitution (3) Law of equi-marginal utility (4) None of the above

36. Identify the correctly matched pair in Column A and Column B from the following: Column A

Column B

(1) India

(a) Economic reforms in 1997

(2) Pakistan

(b) Economic reforms in 1991

(3) China

(c) Economic Reforms in 1978

(4) India

(d)  GST rolled out in 1999

(1) 1 – (a)

9

(2) 2 – (b)

(3) 3 – (c) (4) 4 – (b)

37. Real GDP is considered as an index of: (1) price level in the economy. (2) welfare of the people. (3) profit maximization. (4) none of the above.

38. Which of the following statements is true? (1) Government Borrowings from the World Bank is a Revenue Receipt.

Column A

Column B

(1)  Borrowings from (a)  Current Account IMF (2)  Import shipping services

of (b)  Invisible items

(3)  Export machinery

of ( c )   A c c o m m o d a t i n g items

(4)  Foreign aid

(d)  Visible items

(1) 1 – (a) (2) 2 – (b) (3) 3 – (c)

(4) 4 – (d)

41. Railways is an example of: (1) Perfect competition (2) Monopoly (3) Duopoly (4) Oligopoly

42. Which of the following statements is true? (1) All financial institutions are banking institutions. (2) RBI has the complete authority to issue currency notes. (3) The government of India issues currency notes. (4) Banking institutions issue currency notes.

43. Which of the following statements is true? (1) In monopolistic competition, the curve is relatively elastic. (2) In monopolistic competition, the curve is relatively inelastic. (3) In monopolistic competition, the curve is perfectly elastic. (4) In a monopolistic competition, the curve is perfectly inelastic.

demand demand demand demand

44. Statement 1: Supply of the commodity can also decrease when the price remains constant.

Statement 2: Due to rise in price of factor of production or inputs.

Alternatives:

(2) Higher fiscal deficit is the result of higher revenue deficit.

(1) Both the statements are true.

(3) The loans taken by government represents a situation of fiscal deficit.

(3) Statement 1 is true but statement 2 is false.

(4) The excess of capital receipts over the revenue receipts is called Revenue deficit.

39. When domestic currency loses its value in relation to a foreign currency in the international money market, it is a situation of:

(2) Both the statements are false. (4) Statement 1 is false but statement 2 is true.

45. The

government encourages exports to correct excess capacity in the economy. This may lead to: (1) inflation in the local market. (2) rise in voluntary unemployment.

(1) Currency appreciation

(3) rise in aggregate demand to match aggregate supply.

(2) Currency depreciation

(4) fall in voluntary unemployment.

(3) Currency devaluation (4) None of the above

46. Which of the following is/are the kinds of economic resources?

10

OSWAAL CUET (UG) Sample Question Papers, ECONOMICS (1) Natural Resources (2) Capital Resources (3) Human Resources (4) All of these

II. Read the following news report and answer the Q.97-Q.100 on the basis of the same:



The quantity of a commodity that a consumer is willing to buy and is able to afford, given the prices of goods and the consumer’s tastes and preferences is called demand for the commodity. Whenever one or more of these variables change, the quantity of the good Chosen by the consumer is likely to change as well. The relation between the consumer’s optimal choice of the quantity of a good and its price is very important and this relation is called the demand function. Thus, the consumer’s demand function for a good gives the amount of the good that the consumer chooses at different levels of its price when the other things remain unchanged.

47. Assertion: The income of the consumers remain unchanged.

Reason: Commodity should be a normal good. Select the correct alternative from the following.

(1) Both Assertion (A) and Reason (R) are true, and Reason (R) is the correct explanation of Assertion (A). (2) Both Assertion (A) and Reason (R) are true, but Reason (R) is not the correct explanation of Assertion (A). (3) Assertion (A) is true, but Reason (R) is false. (4) Assertion (A) is false, but Reason (R) is true.

48. In which of the following cases there will be leftward shift in demand? (1) Increase in the price of substitute goods. (2) Rise in the price of complementary goods. (3) Fall in the price of complementary goods. (4) Decrease in the income of the consumer in case of inferior goods.

49. What is meant by the contraction of demand? (1) Movement along a demand curve (2) Shift in the demand curve (3) Fall in consumer’s income (4) None of the above

50. The price elasticity of demand for a good depends on ____ and ____of the good. (1) The nature of the good (2) The availability of close substitutes (3) Both (1) and (2) (4) None of the above



SAMPLE

Question Paper Maximum Marks : 200

3

Time : 60 Minutes

General Instructions : (i) This paper consists of 50 MCQs, attempt any 40 out of 50 (ii) Correct answer or the most appropriate answer: Five marks (+5) (iii) Any incorrect option marked will be given minus one mark (-1) (iv) Unanswered/Marked for Review will be given no mark (0) (v) If more than one option is found to be correct then Five marks (+5) will be awarded to only those who have marked any of the correct options (vi) If all options are found to be correct then Five marks (+5) will be awarded to all those who have attempted the question. (vii) Calculator / any electronic gadgets are not permitted.

1. Which of the following is not a ‘factor payment’? (1) Free uniform to defence personnel (2) Salaries to the Members of Parliament (3) Rent paid to the owner of a building (4) Scholarship given to the students

2. Who coined the terms `macro’ and `micro’ in 1933, to denote the two branches of economic theory?

preventing other smaller competitors from entering the market. (4) The existence of monopoly requires that a few firms are able to gain significant market power, preventing other smaller competitors from entering the market.

6. A situation when AS = AD along with fuller utilisation of resources in the economy is called:

(1) Adam Smith

(2) Paul A. Samuelson

(1) underemployment equilibrium.

(3) Prof. Robbins

(4) Ragnar Frisch

(2) inflationary gap.

3.

is the main source of money supply in an economy. (1) Central Bank

(2) Commercial Banks

(3) Both (A) and (B)

(4) Government

4. A company located in India receives a loan from a company located abroad. How is this transaction recorded in India’s Balance of Payments Account? (1) Credit side of current account (2) Debit side of current account (3) Credit side of capital account (4) Debit side of capital account

5. Which of the following statements is true? (1) The existence of oligopoly requires that a few firms are able to gain significant market power but can’t prevent other smaller competitors from entering the market. (2) The existence of oligopoly requires that a large number of firms are able to gain significant market power, preventing other smaller competitors from entering the market. (3) The existence of oligopoly requires that a few firms are able to gain significant market power,

(3) equilibrium without excess capacity. (4) deflationary gap.

7. The supply curve of a good shift to the right when price of other goods ________. (1) Remains constant

(2) Rise

(3) Fall

(4) None of the above

8. Identify the correctly matched pair from Column A to that of Column B: Column A

Column B

(1)  MPC

(a)  Ratio of Savings to Consumption

(2)  APC

(b)  Ratio of Consumption to Income

(3)  APS

(c)  Ratio of Consumption to Savings

(4)  MPS

(d)  Ratio of Savings to Investment

(1) 1 – (a) (3) 3 – (c)

(2) 2 – (b) (4) 4 – (d)

9. Unforeseen obsolescence of fixed capital assets during production is: (1) Consumption of fixed capital (2) Capital loss

12

OSWAAL CUET (UG) Sample Question Papers, ECONOMICS (3) Income loss (4) None of the above

10. Primary deficit is borrowing requirements of government for making:

M1 is the most liquid measure of money supply. M2 is the most liquid measure of money supply. M3 is the most liquid measure of money supply. All the statements are true.

16. If TR is `1,00,000 when `20,000 units are sold, then

(1) Interest payments. (2) Other than interest payments.

AR is equal to:

(3) All types of payments.

(1) `1,00,000

(2) `20,000

(4) Some specific payments.

(3) `5

(4) `1,20,000

17. If in an economy the value of Net Factor Income

11. Foreign exchange refers to:

(1) the price of one currency in terms of gold in the domestic market. (2) the price of one currency determined by government of other country. (3) the price of one currency in relation to other currencies in the international money market. (4) none of the above.

12. Identify the correctly matched pair in Column A and Column B from the following: Column A

Column B

(1) Product differentiation

(a) Feature of Perfect Competition

(2) Firms sell homogeneous products

(b) Feature of Perfect Competition

(3) Optimal allocation of resources in the long run

(c) Limitation of Perfect Competition

(4)  Firms are makers

(d) Advantage of Perfect Competition

price

(1) 1 – (a)

(B) 2 – (b)

(3) 3 – (c)

(D) 4 – (d)

13. Statement 1: Supply of the commodity can also decrease when the price remains constant.

Statement 2: Due to rise in price of factor of production or inputs.

Alternatives: (1) Both the statements are true. (2) Both the statements are false. (3) Statement 1 is true but statement 2 is false. (4) Statement 1 is false but statement 2 is true.

14. Which of the following statement is true?

from Abroad is ` 200 crores and the value of Factor Income to Abroad is ` 40 crores. Identify the value of Factor Income from Abroad:  (1) ` 200 crores (2) ` 160 crores (3) ` 240 crores (4) ` 180 crores

18. Income rises from `50,000 to `60,000, consumption increases from `40,000 to `48,000. In this situation, what will be the value of Marginal Propensity to Consume (MPC)? (1) 0.80 (2) 0.20 (3) 0.10 (4) 0.90

19. What is the other name of Microeconomics and Macroeconomics? (1) Price Theory, Income Theory (2) Price Theory, Employment Theory (3) Cost Theory, Income and Employment Theory (4) Price Theory, Income and Employment Theory

20. Primary deficit in a government budget is: (1) Revenue expenditure – Revenue receipts (2) Total expenditure – Total receipts (3) Revenue deficit – Interest payments (4) Fiscal deficit – Interest payments.

21. Statement 1: In the short-run production function, there are both fixed and variable factors.

Statement 2: Law of variable proportion works in the long run in the field of production. (1) Both the statements are true. (2) Both the statements are false. (3) Statement 1 is true but statement 2 is false. (4) Statement 1 is false but statement 2 is true.

22. Which of the following statements is not true? (1) Borrowings from the Asian Development Bank by the government is an accommodating transaction.

(1) When TR increases at decreasing rate then MR also increases

(2) Loans given to Sri Lanka by the government is an accommodating transaction.

(2) When TR increases at constant rate then MR is also constant.

(3) Buying of machinery from Japan is an accommodating transaction.

(3) When TR increases at increasing rate then MR declines. (4) When TR is minimum, MR is zero.

(1) (2) (3) (4)

15. Which of the following statements is true?

(4) Borrowing from public is an accommodating transaction.

23. The total output generated by the first four units of variable input is 200 units, 350 units, 450 units and

Sample Question Papers 500 units. The marginal product of the third unit of input is: (A) 200 units

(B) 100 units

(C) 400 units

(D) 500 units

24. Fiscal Deficit equals: (1) Interest payments (2) Borrowings (3) Interest payments less borrowing (4) Borrowings less interest payments

25. National Income is the sum of factor incomes

13

30. The Non-tax revenue in the following is: (1) Export duty

(2) Import duty

(3) Dividends

(4) Excise

31. Assertion: With every increase in the level of GDP,

social welfare definitely increases in the economy. Reason (R): GDP is not a true indicator of welfare of the economy. (1) Both Assertion (A) and Reason (R) are true, and Reason (R) is the correct explanation of the Assertion (A).

(1) Nationals

(2) Both Assertion (A) and Reason (R) are true, but Reason (R) is not the correct explanation of the Assertion (A).

(2) Economic territory

(3) Assertion (A) is true, but Reason (R) is false.

(3) Residents

(4) Assertion (A) is false, but Reason (R) is true.

accruing to:

(4) Both residents and non-residents

26. ..................... is the effect on price when a monopoly firm tries to sell more. (1) Increase Prices (2) Reduce Prices (3) Do not change Price (4) Introduce new product.

27. Which of the following statements is correct? (1) Aggregate Demand refers to the total demand for the goods and services in the economy as a whole. (2) Aggregate Demand constitutes of Consumption Expenditure, Investment Expenditure and Government Expenditure only. (3) The expenditure of household and private investors in an economy for an accounting year is termed as Public Consumption Expenditure. (4) The demand of only goods by the households in an economy is termed as Private Investment Expenditure.

28. ___________ states that as more and more units of variable factors are combined with the fixed factor, a stage must ultimately come when marginal product of the variable factor starts declining. (1) Law of variable proportion (2) Law of diminishing returns (3) Law of returns to scale (4) None of the above

29. Indifference curve is: (1) Downward sloping, concave to the origin (2) Downward sloping, convex away from the origin (3) Downward sloping, convex to the origin (4) Downward sloping, concave away from the origin

32. Assertion (A): There is no restriction on the entry and exit of the firms in the perfect competitive market.

Reason (R): The perfect competition market is characterised by the sellers being a price taker and not a price maker. (1) B  oth Assertion (A) and Reason (R) are true, and Reason (R) is the correct explanation of the Assertion (A). (2) B  oth Assertion (A) and Reason (R) are true, but Reason (R) is not the correct explanation of the Assertion (A). (3) Assertion (A) is true, but Reason (R) is false. (4) Assertion (A) is false, but Reason (R) is true.

33. Pollution created by factories/vehicles is an example of: (1) (2) (3) (4)

Positive externalities. Negative externalities. Either (A) or (B). Neither (A) of (B).

34. Money supply includes

:

(1) All deposits in banks. (2) Only demand deposits in banks. (3) Only time deposits in banks. (4) Currency with the banks.

35. Toothpaste is a product of which type of market form? (1) Perfect competition (2) Monopoly (3) Oligopoly (4) Monopolistic competition

36. Exchange rate is the price of a currency expressed in terms of:  (1) Gold (2) Metal (3) Another currency

(4) None of the above.

14

OSWAAL CUET (UG) Sample Question Papers, ECONOMICS

37. After independence, India had favourable Balance of Trade situations for: (1) Two times (3) Four times

43. According

to Samuelson, there are three fundamental and interdependent problems in an economic organization. What are they?

(2) Three times (4) Always

(1) what, when and how (2) what, how and for whom

38. Which of the following is the reason behind the

(3) what, where and when

downward slope of demand option?

(4) what, how and where

(1) Purchasing power falls (2) I all in the price of commodity

44. Which of the following statements is true?

(3) New consumers creating demand

(1) Under employment equilibrium refers to the situation when people who are able and willing to work at the prevailing wage rate do not get employment.

(4) Demand less unit of commodity

39. Identify the correctly matched pair from Column A to that of Column B: Column A

Column B

1. Deflationary Gap

(a)  Selling of government securities

(2) Under employment equilibrium refers to the situation when people who are not able but are willing to work at the prevailing wage rate do not get employment. (3) Full employment equilibrium refers to the situation when people who are not able but are willing to work at the prevailing wage rate do not get employment.

2.  Deficient Demand (b) AD > AS (at full employment level) 3. Regulating interest rate and availability of credit

(c)  Monetary Policy

4. Voluntary unemployment

(d)  Corrected when AD=AS

(1) 1 – (a)

(2) 2 – (b)

(3) 3 – (c)

(4) 4 – (d)

40. Which of the following are the correct assumptions

(4) Full employment equilibrium refers to the situation when people who are able and willing to work at the prevailing wage rate get employment.

45. Utility is a ____ concept.

to that of Column B: Column A

(4) Lending to commercial banks

I. Read the case study and answer the questions 97 to 100:



The Coca-Cola Company is an American multinational beverage company, with its headquarters in Atlanta, Georgia. The first company that conducted its operation in the soft drink industry was Coca-Cola. It is the world’s largest non-alcoholic beverage company serving more than 1.8 billion consumers daily in more than 200 countries. It has a portfolio of more than 3,500 (more than 800 no or low calorie) products.



However, the company is best known for its flagship product Coca-Cola which was originally intended to be a patented medicine was invented in 1886 by pharmacist John Smith Pemberton in Columbus, Georgia. The Coca-Cola products can be termed as normal goods and in August, 2019 Coca-Cola has introduced the new product into the market,that is, zero sugar where the demand has increased for the product in the market.

Column B

1.  GDP of a country (a) Absolute measure of welfare 2. Underestimation of GDP

(b) Non-monetary exchanges excluded

3.  Base year

(c)  Year of current market prices

4. Economic Welfare

(d) Mental satisfaction and peace

(1) 1 – (a)

(2) 2 – (b)

(3) 3 – (c)

(4) 4 – (d)

42. Out of following one is non-renewable source: (1) Forest

(2) Water

(3) Sun rays

(4) Coal

(3) None of these

(1) Banking facilities to government (2) Banking facilities to public (3) Lending to government

(2) Consumer’s income is given.

41. Identify the correctly matched items in Column A

(4) Alphabetical central bank? 

(1) Prices of commodities are given and continue to change.

(4) All of the above

(2) Qualitative

46. Which of the following is not the function of the

for the utility approach?

(3) Utility is not measurable in monetary terms

(1) Cardinal

Sample Question Papers

According to the council of Australian Food Technology association and Institute of Food Science and Technology, the Australian nonalcoholic beverages industry has been growing steadily, with 2.3 percent increase in overall production in the year 2000 which amounts to 2.25 billion litres. However, in the recent years, sales of customary carbonated soft drinks have dropped as more and more customers become health conscious and move away from high calorie sugary drinks. Soft Carbonated drinks and other alcohol free beverages manufacturers have also sensed the effects of intensifying competition from private-label soft drink makers. Nevertheless, sales of greater value energy and sports drinks have driven profit generation in the industry.

15

47. The demand for Coca-cola is ..................... in the present times. (1) Elastic

(2) Inelastic

(3) Both (1) and (2)

(4) Neither (1) nor (2)

48. The demand for coca-cola has ...................... (1) increased

(2) decreased

(3) remained same

(4) can’t say

49. ..................... is the want to buy a product backed by purchasing power. (1) Supply

(2) Demand

(3) Production

(4) Cost

50. What has happened to the demand of zero sugar carbonated drinks? (1) Increased

(2) Decreased

(3) Remained same

(4) Can’t say



SAMPLE

Question Paper Maximum Marks : 200

4

Time : 60 Minutes

General Instructions : (i) This paper consists of 50 MCQs, attempt any 40 out of 50 (ii) Correct answer or the most appropriate answer: Five marks (+5) (iii) Any incorrect option marked will be given minus one mark (-1) (iv) Unanswered/Marked for Review will be given no mark (0) (v) If more than one option is found to be correct then Five marks (+5) will be awarded to only those who have marked any of the correct options (vi) If all options are found to be correct then Five marks (+5) will be awarded to all those who have attempted the question. (vii) Calculator / any electronic gadgets are not permitted.

1. Which of the following is a stock?

5. Which of the following factor affects the individual

(1) Savings

demand?

(2) Production

(1) Number of consumers in the market

(3) Consumption of fixed capital

(2) Distribution of income

(4) Capital

(3) Age of the sample population

2. It takes Rohan 30 minutes to do an economics assignment and one hour to do a finance assignment. What is the opportunity cost of you doing two economics assignments? (1) 1 finance assignments (2) 2 finance assignments (3) 1/2 finance assignments (4) 3 finance assignments

3. In an open economy, Aggregate Demand is estimated as: (1) private consumption expenditure + private investment expenditure + government expenditure + exports (2) private consumption expenditure + private investment expenditure + government expenditure + imports (3) private investment expenditure + government expenditure + net exports (4) private consumption expenditure + private investment expenditure + government expenditure + net exports

4. Demand deposits include:

(4) Income of the consumer

6. Fiscal deficit equals: (1) Primary deficit minus interest payments. (2) Primary deficit plus interest payments. (3) Total budget expenditure minus total budget receipts. (4) None of the above.

7. Identify the correctly matched pair of items in Column A to those in Column B: Column A 1. Money Flow 2.  Real Flow 3.  Trade Flow 4.  Capital Flow

Column B (a)  Consumption Expenditure (b)  Between Households (c) Only Domestic Economy (d) Inventory

(1) 1 – (a) (2) 2 – (b) (3) 3 – (c) (4) 4 – (d)

8. An Indian company located in India invests in a company located abroad. This transaction is entered in India’s Balance of Payments Account on: (1) Credit side of current account.

(1) Saving account deposits and fixed deposits.

(2) Debit side of current account.

(2) Saving account deposits and current account deposits.

(3) Credit side of capital account.

(3) Current account deposits and fixed deposits (4) All types of deposits.

(4) Debit side of capital account.

9. Identify the correctly matched pair from Column A to that of Column B:

Sample Question Papers Column A

Column B

(1) Investment Multiplier

(a) C + I + G + (X – M)

(2) Marginal Propensity to Consume

(b)   ∆Y

(3) Marginal Propensity to Save

(c)

(4)  Aggregate Demand

∆S (d)  ∆Y

Column B

(1)  Tea Industry

(a)  Monopoly

(2)  Ship Building industry

(b) Perfect Competition

(3) Oil Producing Countries

(c)  Oligopoly

(4)  Food Market

(d)  Monopoly

∆C

∆I   ∆Y

(1) 1 – (a) (2) 2 – (b) (3) 3 – (c) (4) 4 – (d)

15. Which of the following statements is true?

(1) 1 – (a)

(2) 2 – (b)

(3) 3 – (c)

(4) 4 – (d)

10. Identify the correctly matched pair of the items in Column A to that of Column B. Column A

Column B

(1) Increase in demand (a) Leftward shift in for a goods the demand curve (2) Decrease in demand

(b) Perfectly Elastic Demand

(3) Ed = ∞

(c) Increase in income of the consumer

(4) Downward Sloping (d) Income elasticity of Demand (1) 1 – (a) (B) 2 – (b) (3) 3 – (c) (D) 4 – (d)

11. When the actual price of a commodity is less than the equilibrium price, then equilibrium price ____________. (1) Starts rising

(2) Starts falling

(3) Remains constant

(4) First falls, then rises

12. Which of the following statement is true? (1) Fixed cost curve is parallel to X-axis because it changes at all levels of output. (2) Fixed cost curve is parallel to Y-axis because it remains constant at all levels of output. (3) Fixed cost curve is parallel to X-axis because it remains constant at all levels of output. (4) Fixed cost curve is parallel to Y-axis because it changes at all levels of output.

13. Foreign embassies in India are a part of India’s: (1) Economic territory (2) Geographical territory (3) Both (A) and (B) (4) None of the above

14. Identify the correctly matched item from Column A to that of Column B:

Column A

17

(1) V alue of multiplier will be zero of entire additional income consumption is converted into additional consumption. (2) V  alue of multiplier will be infinity of entire additional income consumption is converted into additional consumption. (3) V  alue of multiplier will be unitary of entire additional income consumption is converted into additional consumption. (4) V alue of multiplier cannot be determined of entire additional income consumption is converted into additional consumption.

16. Identify the correctly matched pair from Column A to that of Column B: Column A

Column B

(1) Flexible Exchange Rate

(a) Foreign Exchange Rate is determined by the Central Bank.

(2) Fixed Exchange Rate

(b) The foreign exchange rate is determined by the demand and supply.

(3) Managed Flexible Rate

(c) The Central Bank interferes with the market demand and supply of foreign currency.

(4) Managed Fixed Rate

(d) The Central Bank controls the foreign rate regime.

(1) 1 – (a) (2) 2 – (b) (3) 3 – (c) (4) 4 – (d)

17. Who regulates money supply in India? (1) Government of India (2) Reserve Bank of India (3) Commercial Banks (4) Planning Commission

18. The slope of supply curve is measured by______________.

18

OSWAAL CUET (UG) Sample Question Papers, ECONOMICS (1) ΔQ / ΔP

(2) ΔP / ΔQ

(3) ΔP - ΔQ

(4) ΔP + ΔQ

25. What is the saturation point? (1) When TU is minimum and MU is zero

19. Which one of the following is a combination of

(2) When TU is maximum and MU is 1

direct taxes?

(3) When TU is minimum and MU is 1

(1) Excise duty and Wealth tax

(4) When TU is maximum and MU is zero

(2) Service tax and Income tax

26. Due to installation of a machine with latest

(3) Excise duty and Service tax

technology, the cost of production has decreased. It will lead to:

(4) Wealth tax and Income tax

20. Statement 1: When MP is more than AP, then AP

(1) Expansion in supply

rises.

(2) Increase in supply

Statement 2: When MP is less than AP, then AP is at

(3) Contraction in supply

its maximum point.

(4) Decrease in supply

27. Which of the following statements is false?

Alternatives:

(1) An economy can attain the maximum equilibrium GDP level even when excess capacity is not fully exhausted. (2) An economy can attain the maximum equilibrium GDP level only when excess capacity is fully exhausted. (3) An economy can attain the maximum equilibrium GDP level even when excess capacity is partially exhausted. (4) An economy can attain the minimum equilibrium GDP level even when excess capacity is fully exhausted.

(1) Both the statements are true. (2) Both the statements are false. (3) Statement 1 is true but statement 2 is false. (4) Statement 1 is false but statement 2 is true.

21. Aggregate demand can be increased by: (1) Increasing bank rate. (2) Selling government securities by Reserve Bank of India. (3) Increasing cash reserve ratio. (4) None of the above.

22. Which of the following is the variable cost for a firm?

28. Which of the following is an assumption for Indifference Curve Approach.

(1) Interest on loan

(1) Consumers are to be rational.

(2) Monthly rent

(2) Consumer income is given.

(3) Insurance premium

(3) Price of commodity remain constant.

(4) Wages to employees

(4) Income is changed.

23. Which of the following statements is true? (1) A monopoly selects a higher price and lesser quantity of output than a price-taking company.

29. Which of the following statements are incorrect? (1) Indifference Curves are convex to the origin. (2) Indifference Curves are downward sloping to the right.

(2) A monopoly selects a lower price and lesser quantity of output than a price-taking company. (3) A monopoly selects a higher price and more quantity of output than a price-taking company.

(3) A higher indifference curve represents a higher level of satisfaction. (4) Two curves can intersect each other.

30. Which of the following statement is true? (1) Stock refers to the total quantity of a commodity available with the seller at any given time.

(4) A monopoly selects a higher price and lesser quantity of output than a price-making company.

(2) Stock refers to the total quantity of a commodity available with the buyer at any given time.

24. Which of the following statements is true?

(3) Stock refers to the total price available with the seller at any given time.

(1) Demand Curve is indeterminate under perfect competition. (2) Demand Curve monopoly.

is

indeterminate

under

(3) Demand Curve is indeterminate monopolistic competition.

under

(4) Demand Curve oligopoly.

under

(4) Stock refers to the total cash available with the seller at any given time.

31. Assertion (A): Export of goods and services is

indeterminate

from India to US would mean outflow of foreign exchange from India.

Reason (R): Foreign exchange in terms of receipts for exports flows from US to India.

Sample Question Papers (1) B  oth Assertion (A) and Reason (R) are true, and Reason (R) is the correct explanation of the Assertion (A). (2) Both Assertion (A) and Reason (R) are true, but Reason (R) is not the correct explanation of the Assertion (A). (3) Assertion (A) is true, but Reason (R) is false. (4) Assertion (A) is false, but Reason (R) is true.

32. Assertion (A): Demand deposits are not legal tenders.

(1) 1 – (a)

(2) 2 – (b)

(3) 3 – (c)

(4) 4 – (d)

19

34. Identify which of the following statements is true. (1) The difference between planned revenue expenditure and planned revenue receipts is called Fiscal deficit. (2) The difference between total planned expenditure and total planned receipts is called fiscal deficit.

Reason (R): They are with the bank, so only can be

(3) The difference between total planned receipts and interest payment is called primary deficit.

used as a legal tender when cheques are issued for the transfer.

(4) The sum of primary deficit and interest payment is called fiscal deficit.

(1) Both Assertion (A) and Reason (R) are true, and Reason (R) is the correct explanation of the Assertion (A).

35. The shape of average revenue curve in monopoly is

(2) Both Assertion (A) and Reason (R) are true, but Reason (R) is not the correct explanation of the Assertion (A). (3) Assertion (A) is true, but Reason (R) is false. (4) Assertion (A) is false, but Reason (R) is true.

33. Identify the correctly matched pair in Column A and Column B from the following: Column A

Column B

..................... (1) Upward rising

(2) Downward sloping

(3) U- shaped

(4) S - shaped

36. Product differentiation is a distinguishing feature of which form of market? (1) Perfect competition (2) Monopoly (3) Oligopoly (4) Monopolistic competition

37. Identify the phase in which TP increases at an increasing rate and MP also increases.

(1) India should create more employment opportunities.

(a) Normative Economics

(2) Unemployment

(b)  Microeconomics

(3) Negative returns to a factor

(3)  Price behaviour

(c)  Macroeconomics

(4) None of these.

(1) Increasing returns to a factor. (2) Diminishing returns to a factor

(4) Government provided (d) Normative healthcare increases Economics public expenditure.

38. Identify the correctly matched items from Column A to that of Column B: Column A

Column B

1.  Issue of New Currency Notes

(a)  Government of India

2.  Banker to the Government

(b)  State Bank of India

3.  Controller of Credit

(c)  Reserve Bank of India

4.  SLR

(d)  Development Bank

(1) 1 – (a)

(2)  2 – (b)

(3) 3 – (c)

(4)  4 – (d)

39. Identify the correctly matched pair from Column A to Column B: Column A

Column B

(1)  Increase of Foreign Reserve from $400 to $450 (a)  Credit side of Capital Account millions (2)  Dasault Ltd. invested in Reliance Defence

(b)  Debit Side of Capital Account

(3)  India gave line of credit to Botswana

(c)  Credit Side of the Capital Account

(4)  India borrowed $ 350 million from the world (d)  Debit Side of the Capital Account bank

20

OSWAAL CUET (UG) Sample Question Papers, ECONOMICS (1) 1 – (a)

(2) 2 – (b)

(3) 3 – (c)

(4) 4 – (d)

40. Statement 1: AP can take positive values only. Statement 2: TP can take negative values only. Alternatives:

45. Identify the correctly matched pair in Column A and Column B from the following: Column A

Column B

1. Uniformity of taxes

(a) Effect of 2016 Demonetisation

(2) Both the statements are false.

2. At the State Level

(b) Benefit of GST

(3) Statement 1 is true but statement 2 is false.

3. One Point Single Tax

(c) Objective of GST

4. Brought an end to black money

(d) SGST

(1) Both the statements are true.

(4) Statement 1 is false but statement 2 is true.

41. Identify the correctly matched pair in Column A and Column B from the following: Column A 1.  Aggregate Demand 2.  Aggregate Demand

Column B (a) Positive Economics (b)  Microeconomics

3. Monopolistic Competition Market

(c) Normative Economics

4.  Supply of Money

(d)  Macroeconomics

(1) 1 – (a)

(2) 2 – (b)

(3) 3 – (c)

(4) 4 – (d)

42. Desire to own a commodity is always backed up by: (1) Desired quantity (2) Purchasing power (3) Final consumer good (4) Flow concept

43. Identify the correctly matched pair from the items in Column A by matching them to the items in Column B: Column A

Column B

1. Relatively Inelastic Demand

(a) ed > 1

2. Relatively Elastic Demand

(b) ed < 1

3. Perfectly Inelastic Demand

(c) ed = 0

4. Perfectly Elastic Demand

(d) ed = 1

(1) 1 – (a) (2) 2 – (b) (3) 3 – (c) (4) 4 – (d)

44. When aggregate demand is greater than aggregate supply, inventories: (1) fall (2) rise (3) do not change (4) first fall, then rise

(1) 1 – (a) (2) 2 – (b) (3) 3 – (c) (4) 4 – (d)

46. Which of the following is a source of capital receipt? (1) Foreign donations

(2) Dividends

(3) Dis-investment

(4) Indirect taxes

I. Read the below case and answer the questions that follow: The Centre on Saturday increased the budgetary allocation for the environment ministry from last fiscal by nearly five percent for 2020-21 with no change in the amount allotted to pollution abatement and climate change action plan. Union Finance Minister, Nirmala Sitharaman, allocated `3,100 crore for the ministry out of which `460 crore were allotted to control pollution, which is the same as the money it received in the last budget. Control of pollution has been conceptualized to provide financial assistance to Pollution Control Boards/Committees and funding to National Clean Air Programme (NCAP). Similarly, budget for pollution abatement, which was cut by 50 percent last year from 2018-19, remained unchanged at `10 crore. The minister also announced that states, which are formulating and implementing plans for ensuring cleaner air in cities above one million population should be encouraged. - Budget 2020: Allocation for Environment Ministry up 5% to `3,100 crore – Business Standards, 1st February, 2020

47. What type of externality will the increase in budget allocation create? (1) Negative

(2) Positive

(3) Neutral

(4) Can’t be said

48. _________ means material well-being of the people. (1) Externality

(2) Welfare

(3) Economy

(4) None of the above

Sample Question Papers 49. Real GDP and Welfare are ___________ related with each other. 

21

50. Why is the policy implemented? (1) To ensure clear air

(1) Directly

(2) Indirectly

(2) To ensure clear water

(3) Inversely

(4) None of the above

(3) To ensure clean roads (4) All of the above



SAMPLE

Question Paper Maximum Marks : 200

5

Time : 60 Minutes

General Instructions : (i) This paper consists of 50 MCQs, attempt any 40 out of 50 (ii) Correct answer or the most appropriate answer: Five marks (+5) (iii) Any incorrect option marked will be given minus one mark (-1) (iv) Unanswered/Marked for Review will be given no mark (0) (v) If more than one option is found to be correct then Five marks (+5) will be awarded to only those who have marked any of the correct options (vi) If all options are found to be correct then Five marks (+5) will be awarded to all those who have attempted the question. (vii) Calculator / any electronic gadgets are not permitted.

1. M3 = _________. (1) M1 + Time deposits of all commercial banks and co-operative banks (excluding interbank time deposits). (2) M1 + Time deposits of all commercial banks (excluding interbank time deposits). (3) M1 + Demand deposits of all commercial banks and co-operative banks (excluding interbank time deposits). (4) M2 + Time deposits of all commercial banks and co-operative banks (including interbank time deposits).

2. Identify the correctly matched pair of the items in Column A to those in Column B: Column A 1.  Money Flow 2.  Real Flow 3.  Trade Flow 4.  Capital Flow

Column B (a) Depreciation (b)  Factor Services (c)  House Work by Housewife (d) Inventory

(1) 1 – (a)

(2) 2 – (b)

(3) 3 – (c)

(4) 4 – (d)

3. ___________ is a tabular representation which shows various quantities of a commodity being supplied at various levels of price. (1) Law of variable proportion (2) Law of diminishing returns (3) Law of returns to scale (4) None of the above

4. Suppose in a hypothetical economy, the income rises from `5,000 crore to `6,000 crore. As a result, the consumption expenditure rises from `4,000 crore to

`4,600 crore. Marginal propensity to consume in such a case would be _____________ (1) 0.8

(2) 0.4

(3) 0.2

(4) 0.6

5. Deficient demand leads to: (1) Deflationary gap (2) Excess capacity (3) Low level of employment (4) All of the above

6. Given below are a few examples of Economics as Positive and Normative Science. Choose the corresponding option for each of them. Use (N) for Normative and (P) for Positive. 1. Air is a mixture of gases. 2. Minimum Wage Law increases unemployment. 3. India should spend more money on defence. 4. Prices have been rising in India. (1) PPNP (2) PNPN (4) PPPP (3) NPNN

7. Statement 1: Variable cost is the cost which changes with the change in volume of output.

Statement 2: Fixed cost is the cost which also changes with the change in the level of output.

Alternatives: (1) Both the statements are true. (2) Both the statements are false. (3) Statement 1 is true but statement 2 is false. (4) Statement 1 is false but statement 2 is true.

8. Which of the following statement is true? (1) When the price increases, there is a shift in the demand curve.

Sample Question Papers (2) When the quantity demanded decreases there is a shift in the demand curve. (3) When the price decreases, there is a downward movement along the demand curve. (4) When the quantity demanded decreases, there is a downward movement along the demand curve.

9. Which one of the following is not a capital

15. All possible combinations of goods that a consumer can buy can be shown by budget line, given: (1) Quantity of commodity (2) Expenditure of consumer (3) Quality of commodity (4) Price of commodity

16. Demand curve of a firm under monopoly is:

expenditure?

(1) Downward sloping (2) Indeterminate

(1) Loans advanced by World Bank

(3) Upward sloping

(2) Construction of school buildings

(4) Perfectly elastic

17. Identify the correctly matched pair of the items in Column A to those in Column B:

(3) Repayment of loans

Column A

(4) Purchase of Metro Coaches from Japan

10. Identify the correctly matched pair from Column A to that of Column B:

Column B

1. Revenue Expenditure

(a) Does not cause any reduction in government liability

2. Capital Expenditure

(b) Which creates corresponding liability for the government

Column A

Column B

(1) Private Consumption Expenditure

(a)  Market Rate of Interest

(2) Private Investment Expenditure

(b)  Total Expenditure

3. Revenue Receipts

(3) Autonomous Investment

(c)  Construction of Roads

(c) Which causes reduction in assets of the government

4.  Capital Receipts

(4)  Aggregate Demand

(d)  Level of personal disposable income

(d) Causes reduction in government liability

(1) 1 – (a) (2) 2 – (b) (3) 3 – (c) (4) 4 – (d)

11. Point out a merit of flexible exchange rate. (1) Eliminates undervaluation of currencies only (2) Eliminates over valuation of currencies only (3) Eliminates over valuation or undervaluation of currencies (4) None of the above

23

12. Foreign Exchange Transactions dependent on other Foreign Exchange Transactions are called:

(1) 1 – (a) (2) 2 – (b) (3) 3 – (c)

(4) 4 – (d)

18. Under ..................... form of market, the firm is price maker. (1) Monopoly

(2) Perfect Competition

(3) Monopsony

(4) None of the above

19. There are ..................... to the entry of firms under oligopoly. (1) No Restriction

(2) Restriction

(3) Open

(4) Free

20. Equilibrium level of income/output is established

(1) Current account transactions.

when:

(2) Capital account transactions.

(1) AS = AD

(2) C = I

(3) Autonomous transactions.

(3) S = Y

(4) None of the above.

(4) Accommodating transactions.

13. When marginal product becomes negative, then total product starts ________. (1) Rising

(2) Falling

(3) Constant

(4) None of the above

14. Under perfect competition, the firm earns normal profit in the long run because of: (1) Large number of buyers and sellers (2) Absence of selling cost (3) Free entry and exit (4) Homogeneous commodity

21. Which of the following statements is incorrect? (1) Gross Domestic Product (GDP) at Market Price = GDP at Factor Cost + Net Indirect Taxes (2) Net National Product (NNP) at Market Price = NNP at Factor Cost + Net indirect taxes (3) Gross National Product (GNP) at Market Price = GDP at Market Price + Net Factor Income from Abroad (4) Net National Product (NNP) at Factor cost = National Income

22. At macro level the most efficient technique is the one which uses (1) Most quantity of scarce resources

24

OSWAAL CUET (UG) Sample Question Papers, ECONOMICS (2) Least quantity of resources (3) Least quantity of scarce resources (4) None of the above AE

23. Fixed cost is incurred on __________ factors of production whereas variable cost is incurred on __________ factors of production. (1) Fixed, fixed

(2) Variable, variable

(3) Fixed, variable

(4) Variable, fixed

24. When is the consumer in equilibrium? (1) MUx = Px

(2) MUx = 0

(3) Px = 1

(4) MUx ≠ Px AE

3. Increase or decrease in demand cause a change in the price of the commodity. Equilibrium quantity remains constant.

(c) Effect on demand, in case of Perfectly Elastic Supply

4. Increase or decrease in demand for a commodity does not cause any change in its price

(d) Effect on supply, in case of Perfectly Elastic Demand.

25. Which of the following statements is not correct

(1) 1 – (a)

(2) 2 – (b)

with respect to the correction of Deficient Demand?

(3) 3 – (c)

(4) 4 – (d)

(1) Government reduces the taxes. (2) The Central Bank increases the bank rate.

29. From the set of events/systems given in column A and corresponding relevant fact given in column B, about China, choose the correct pair of statement:

(3) The Central Bank reduces the CRR and SLR. (4) The Government increases its spending.

Column A

26. Identify the correctly matched pair of the items in Column A to that of Column B. Column A

Column B

1. Unitary elastic supply curve

(a)  U-shaped supply curve

2. Relatively elastic supply curve

(b) Vertical line parallel to Y-axis

3. Perfectly elastic supply curve

(c) Horizontal line parallel to X-axis

4. Perfectly inelastic supply curve

(d) Downward sloping supply curve

(1) 1 – (a)

(2) 2 – (b)

(3) 3 – (c)

(4) 4 – (d)

27. The Non-tax revenue in the following is: (1) Export duty

(2) Import duty.

(3) Dividends.

(4) Excise.

2. Increase or decrease in demand causes a change in the price of the commodity. Equilibrium quantity remains constant.

(b) Effect on demand, in case of Perfectly Inelastic Sup-ply

(2) Movement along the demand curve

(b) Increase in price

(3) Shift in the demand Curve

(c) Decrease in price

(4) Movement along the demand curve

(d) Snob effect

30. If MPC = 1, the value of multiplier is: (1) 0 (3) Between 0 and 1

(2) 1 (4) Infinity

31. Assertion (A): Import of machinery is reflected in the current account of balance of payments.

Reason (R): Export and import of goods and invisibles are recorded in current account of balance of payments. (1) Both Assertion (A) and Reason (R) are true, and Reason (R) is the correct explanation of the Assertion (A). (2) Both Assertion (A) and Reason (R) are true, but Reason (R) is not the correct explanation of the Assertion (A). (3) Assertion (A) is true, but Reason (R) is false. (4) Assertion (A) is false, but Reason (R) is true.

Column B (a) Effect on supply, in case of Perfectly Elastic Demand.

(a) Increase in quantity demanded

(3) 3 – (c) (4) 4 – (d)

in Column A by matching them to the items in column B:  Column A

(1) Shift in the demand Curve

(1) 1 – (a) (2) 2 – (b)

28. Identify the correctly matched pair from the items

1. Increase or decrease in demand for a commodity does not cause any change in its price.

Column B



32. Assertion (A): The flour purchased by a baker is

considered an intermediate good. Reason (R): The flour is used for the production of bread, cakes and biscuits to be sold to the consumer. (1) Both Assertion (A) and Reason (R) are true, and Reason (R) is the correct explanation of Assertion (A).

Sample Question Papers (2) Both Assertion (A) and Reason (R) are true, but Reason (R) is not the correct explanation of Assertion (A). (3) Assertion (A) is true, but Reason (R) is false. (4) Assertion (A) is false, but Reason (R) is true.

33. Identify the correctly matched pair of column A to that of Column B: Column A (1) Marginal Utility

(2) Cardinal measure of utility

(3)  Total Utility

(4) Utility

Column B (a) The power or capacity of a commodity to satisfy human wants. (b) The addition to total utility on consuming an additional unit of a commodity. (c) It is the sum total of utility derived from the consumption of all units of a commodity. (d) It is that measurement of utility, which is measured in terms of units

(1) 1 – (a)

(2) 2 – (b)

(3) 3 – (c)

(4) 4 – (d)

34. Accommodating items are also called: (A) On-the-line items (B) Below the line items

Column A

25

Column B

1. Function of a Central bank

(a) Lending money to public

2. Function of Government

(b) Issuing currency notes

3. Function of a Central bank

(c) Control of credit

4. Function of Government

(d) Creation of credit

(1) 1 – (a)

(2) 2 – (b)

(3) 3 – (c)

(4) 4 – (d)

38. Consumers get maximum variety of goods under: (1) Perfect competition (2) Monopoly (3) Oligopoly (4) Monopolistic competition

39. When the price of the commodity is changed the demand for the commodity changes in: (1) Opposite direction

(2) Remains the same

(3) Same direction

(4) None of these

40. Which of the following is not a revenue receipt? (1) Recovery of Loans (2) Foreign Grants (3) Profits of Public Enterprises (4) Wealth Tax

41. The type of oligopoly under which market firms cooperate with each other in determining price and output is ..................... . (1) Collusive

(2) Non-Collusive

(3) Open

(4) Closed

42. Identify the correctly matched pair from Column A

(C) Above the items

to Column B:

(D) None of these

Column A

Column B

macroeconomics?

(1)  MPC = 0

(a)  K > 1

(1) Total Employment

(2)  MPC = 1

(b)  K = Infinity

(2) National Income

(3)  MPC < 1

(c)  K = 0

(4)  MPC > MPS

(d)  K < 1

35. Which of the following is not an example of

(3) General Price Level (4) Individual Income

36. “Market demand curve is constructed by

(1) 1 – (a)

(2) 2 – (b)

horizontally summing all the individual demand curves at each and every price.” Choose the correct option for the above mentioned statement:

(3) 3 – (c)

(4) 4 – (d)

(1) True

(2) False

(3) Partially true

(4) Can’t be determined

37. Identify the correctly matched pair of items in column A to that of Column B:

43. The Narrow concept of money includes _________. (A) Cash with the Public (B) Deposits with Bank (C) Deposits with Post Office (D) All of the above.

44. Which of the following statements is true?

OSWAAL CUET (UG) Sample Question Papers, ECONOMICS

26

(1) When demand increases more than supply, equilibrium price will increase. (2) When demand increases more than supply, equilibrium price will decrease. (3) When supply increases more than demand, equilibrium price will increase. (4) When demand decreases more than supply, equilibrium price will increase.

45. What kind of Tax is GST? (1) (2) (3) (4)

Direct Tax Indirect Tax Depends on the type of goods and services None of the above

46. If there is no change in the demand for commodity X, even after rise in its price, then its demand is: (1) perfectly elastic (2) perfectly inelastic (3) less elastic (4) highly elastic



Read the below case and answer the questions that follow:

The country’s real gross domestic product (GDP) is likely to expand by 11 percent in the next financial year due to a faster economic recovery and on a low base, says a report. The report by domestic rating agency Brickwork Ratings said economic activities are slowly reaching PRE-COVID levels following the relaxation of the lockdown, except in sectors that remain affected by social distancing norms. “With progress in developing an effective vaccine for COVID-19 and signals of faster-than-expected recovery in the domestic economy, and also supported by a low base, we expect the real GDP to grow at 11 percent in F/Y 22, from the estimated contraction of 7 percent to 7.5 percent in F/Y 21,” the agency said. According to the first advance estimates of national income released by the National Statistical Office (NSO), the country’s GDP is estimated to contract by a record 7.7 percent during the current financial year. - “Real GDP to grow at 11 per cent in F/Y 22: Report” Economic Times, 21st Jan, 2021

47. Real GDP is when the goods and services are produced by all producing units in the domestic territory of a country during an accounting year and valued at ___________ prices or constant price. (1) base year’s

(2) current year’s

(3) both (1) and (2)

(4) neither (1) nor (2)

48. Read the following statement Assertion (A) and Reason (R).

Choose one of the correct alternatives given below: Assertion (A): The country’s real gross domestic product is likely to expand.

Reason (R): Some sectors remain affected by social distancing norms. Alternatives: (1) Both Assertion (A) and Reason (R) are true, and Reason (R) is the correct explanation of Assertion (A). (2) Both Assertion (A) and Reason (R) are true, but Reason (R) is not the correct explanation of Assertion (A). (3) Assertion (A) is true, but Reason (R) is false. (4) Assertion (A) is false, but Reason (R) is true.

49. What will be the growth rate of GDP according to the NSO?  (1) – 7.7% (2) +7.7% (3) +11% (4) – 7.5%

50. Read the following statements - Assertion (A) and Reason (R).

Assertion (A): Real GDP is the true indicator of the growth of the economy.

Reason (R): Real GDP is nominal GDP adjusted for inflation used to measure the actual growth of production.

Select the correct alternative from the following: (1) Both Assertion (A) and Reason (R) are true, and Reason (R) is the correct explanation of Assertion (A). (2) Both Assertion (A) Reason (R) are true, but Reason (R) is not the correct explanation of Assertion (A). (3) Assertion (A) is true, but Reason (R) is false. (4) Assertion (A) is false, but Reason (R) is true.



SAMPLE

Question Paper Maximum Marks : 200

6

Time : 60 Minutes

General Instructions : (i) This paper consists of 50 MCQs, attempt any 40 out of 50 (ii) Correct answer or the most appropriate answer: Five marks (+5) (iii) Any incorrect option marked will be given minus one mark (-1) (iv) Unanswered/Marked for Review will be given no mark (0) (v) If more than one option is found to be correct then Five marks (+5) will be awarded to only those who have marked any of the correct options (vi) If all options are found to be correct then Five marks (+5) will be awarded to all those who have attempted the question. (vii) Calculator / any electronic gadgets are not permitted.

1. Which of the following is a stock?

(2) The firms sell differentiated products

(1) Wealth

(2) Savings

(3) There is no free entry and exit.

(3) Exports

(4) Profits

(4) Each firms possesses imperfect knowledge

2. Which of the following sources of receipts in government budget increases its liabilities? (1) Direct taxes (2) Recovery of loans

distribute his money income between the goods in such a way that the utility derived from the last rupee spent on each good is equal. (1) Law of Equi-Marginal Utility

(3) Borrowings (4) Dividend from public sector undertakings

3. Suppose a student has 2 hours in which he can only watch a movie or study. What is the opportunity cost of studying? (1) Opportunity cost of studying is not watching a movie. (2) Opportunity cost of studying is watching a movie. (3) Opportunity cost of studying is partying with friends. (4) There is no opportunity cost of studying.

4. Consumption depends on: (1) Income

7. ..................... states that as the consumer will

(2) Saving

(3) Aggregate Demand (4) Both (1) and (2)

5. AR curve is a horizontal straight line, the MR curve will be: (1) Downward sloping (2) Horizontal straight line (3) Upward sloping (4) Inversely shaped

6. Which of the following statement is true about perfect competition? (1) There are large number of buyers and sellers

(2) Law of Diminishing Marginal Utility (3) Both (1) and (2) (4) Neither (1) nor (2)

8. The shape of supply curve is ___________. (1) Downward sloping (2) Upward sloping (3) U-shaped

(4) Inverse S-shaped

9. A firm is able to sell any quantity of a good at a given price. The firm’s Marginal Revenue will be ________. (1) More than average revenue (2) Less than average revenue (3) Equal to average revenue (4) None of the above

10. In case of an under-employment equilibrium, which of the following alternatives is not true? (1) A ggregate Demand is equal to Aggregate Supply. (2) T  here exists excess production capacity in the economy. (3) Resources are not fully and efficiently utilised. (4) Resources are fully and efficiently utilised.

11. Equilibrium price falls and equilibrium quantity rises when:

28

OSWAAL CUET (UG) Sample Question Papers, ECONOMICS (1) Decrease in demand < Decrease in supply

16. If quantity supplied increases by 60% due to a 50%

(2) Increase in demand = Increase in supply

increase in price, then elasticity of supply is:

(3) Decrease in demand < Increase in supply

(1) (-) 1.2

(2) (+) 1.2

(4) Decrease in demand = Increase in supply

(3) (-) 0.83

(4) (+) 0.83

12. Read the following statements and choose the correct alternative from the following:

Statement 1: The implication of price floor is that when the producers are not able to sell all they want to sell, they illegally sell the good at below the minimum price.

Statement 2: Government enters the market and buys the product. This way price support system works. (1) Both the statements are true. (2) Both the statements are false. (3) Statement 1 is true but statement 2 is false. (4) Statement 1 is false but statement 2 is true.

13. In case of _____________ supply curve is a vertical straight line parallel to Y-axis. (1) Perfectly elastic

(2) Unitary elastic

(3) Perfectly inelastic

(4) Less elastic

14. If in an economy the value of Net Factor Income from Abroad is ` 200 crores and the value of Factor Income to Abroad is ` 40 crores. Identify the value of Factor Income from Abroad:

17. Where will sale of machinery to abroad be recorded in the balance of payment accounts? (1) Positive side (credit items). (2) Negative side (debit items). (3) Both (1) and (2) (4) None of these

18. Borrowing in government budget is: (1) Revenue deficit.

(2) Fiscal deficit

(3) Primary deficit

(4) Deficit in taxes.

19. Which of the following statements is not correct regarding Excess Demand? (1) Excess Demand raises the market value of output. (2) Excess Demand means Aggregate Demand is more than Aggregate Supply. (3) Excess Demand is caused due to reduction in the public expenditure. (4) Excess Demand is the result of decline in exports.

20. The balance of trade shows a deficit of `5,000 crores

(1) ` 200 crores

(2) ` 160 crores

and the value of imports are `9,000 crores. What is the value of exports?

(3) ` 240 crores

(4) ` 180 crores

(1) `5,000 crores

(2) `4,000 crores

(3) `3,000 crores

(4) `1,000 crores

15. Match the following items in Group A to that in Group B and choose the correctly matched pair. Group A

Group B

(1) Assumption of Law of diminishing marginal utility

(a) Homogeneous units of commodity

(2) Assumption of Law of equimarginal utility

(b) Continuous consumption without any time lag

(3) Assumption of Law of diminishing total utility

(c) Standard units of commodity

(4) Assumptions of Law of diminishing utility

(d) No change in tastes, preferences, or income of the consumer

(1) 1 – (a) (2) 2 – (b) (3) 3 – (c)

(4) 4 – (d)

21. Indifference curve are convex to the origin because of: (1) Increasing MRS (2) Diminishing MRS (3) Law of diminishing MU (4) Law of equi-marginal utility

22. Identify the correctly matched pair of items in Column A to those in Column B: Column A

Column B

1.  Income Tax

(a)  Forced Transfer

2. Services of Housewives 3.  Retirement Pension 4. Annual value of goods and services produced

(b)  Market Activities (c)  Taxable for Firm (d)  Income method

(1) 1 – (a)

(2) 2 – (b)

(3) 3 – (c)

(4) 4 – (d)

23. If the price of good X rises and it leads to an increase in demand for good Y, both are ................. goods. (1) Substitutes

(2) Complementary

(3) Normal

(4) Inferior

Sample Question Papers 24. Which of the following statement is true: (1) Marginal Product refers to total output produced by a firm during a given period of time with given number of inputs. (2) Average Product refers to total output produced by a firm during a given period of time with given number of inputs. (3) Total Product refers to total output produced by a firm during a given period of time with given number of inputs. (4) Total Product refers to total input produced by a firm during a given period of time with given number of output.

25. We say that there is a decrease in demand when: (1) (2) (3) (4)

Fall in demand, with same price. Fall in demand, with decrease in price. Fall in demand, with increase in price. None of the above.

26. The expenditure on a good would change in the opposite direction as the price changes only when demand is ...................... (1) Elastic (2) Inelastic (3) Both (1) and (2) (4) None of the above

27. Identify the correctly matched pair from Column A to Column B:

(1) 1 – (a)

(2) 2 – (b)

(3) 3 – (c)

(4) 4 – (d)

29

29. With a rise in real national income, welfare of the people: (1) rises. (2) falls. (3) remains unchanged. (4) none of the above.

30. How does Robbins define Economics? (1) Science of Markets

(2) Science of Business

(3) Science of Life

(4) Science of Scarcity

31. Assertion (A): Fiscal deficit is measured in terms of borrowings.

Reason (R): External borrowings increases the Fiscal deficit. (1) Both Assertion (A) and Reason (R) are true, and Reason (R) is the correct explanation of the Assertion (A). (2) Both Assertion (A) and Reason (R) are true, but Reason (R) is not the correct explanation of the Assertion (A). (3) Assertion (A) is true, but Reason (R) is false. (4) Assertion (A) is false, but Reason (R) is true.

32. Assertion (A): Car purchased by the hotel for the

Column A

Column B

(1)  Y = AD

(a) Level of output at full employment

(2)  Forward Multiplier

(b)  Withdrawal of investment decreases income

(3)  Paradox of Thrift

(c) People save less or same as before

(4)  Multiplier (k) < 1

(d)  0 < MPC < 1

(1) 1 – (a)

(2) 2 – (b)

(3) 3 – (c)

(4) 4 – (d)

28. Identify the correct matched pair from Column A to Column B and choose the correct alternative:

purpose of tourism facilitation is a Capital Good.

Reason (R): The car is purchased by the hotel to provide long term service to the consumers. (1) Both Assertion (A) and Reason (R) are true, and Reason (R) is the correct explanation of the Assertion (A). (2) Both Assertion (A) and Reason (R) are true, but Reason (R) is not the correct explanation of the Assertion (A). (3) Assertion (A) is true, but Reason (R) is false. (4) Assertion (A) is false, but Reason (R) is true.

33. _________ constitute the third element in the BoP which is the ‘balancing item’. (1) Capital account (2) Current account (3) Errors and Omissions (4) None of these

Column A

Column B

1. Export of weapons to Vietnam

(a) Credit side of Current Account

34. Level of planned output coincides with planned

2. Import of Apache Helicopter from USA

(b) Capital Account of Balance of Payments

expenditure when: (1) AD = AS (2) C=I (3) both (1) and (2) (4) none of these

3. Remittances from relative from UAE

(c) Debit side of Current Account of Balance of Payments

35. Consider the following statements about Export of

4. I nvestment by Tesla Incorporation in India

(d) Credit side of Current Account of Balance of Payments

cotton Textiles. i. It is shown in the debit side of the current account. ii. It reduces the balance of payment deficit. iii. It is shown in the credit side of the current account.

30

OSWAAL CUET (UG) Sample Question Papers, ECONOMICS

Which of the following statements are true?

41. Total output is maximum when marginal product is

(1) iii only

(2) i and ii

_____________.

(3) ii and iii

(4) None of these

(1) One

36. A producer starts the business in the building owned by him and borrows money for running it. The Imputed rent of own building is ______________.

(2) Zero (3) Maximum (4) Minimum

(1) Explicit costs

42. Which of the following statement is not true

(2) Implicit costs (3) Opportunity costs

about the need for import substitution for Indian economy? 

(4) Variable cost

(1) Unfavourable balance of trade.

37. When percentage change in quantity demanded is equal to percentage change in price, then demand for such a commodity is said to be ..................... (1) Perfectly Elastic

(2) Relatively Elastic

(3) Unitary Elastic

(4) None of the above

38. Identify the correctly matched items in Column A to that of Column B:

(2) Devaluation of rupee. (3) Abundance of foreign aid. (4) Shortage of essential commodities.

43. Purchase of shares is related to: (1) revenue receipt (2) revenue expenditure (3) capital receipt

Column A

Column B

1. Welfare of the people

(a)  Social Welfare

2. Total of economic and non-economic welfare

(b) Economic Welfare

3. Benefits or harms of an activity caused by a firm

(c) Externalities

4. Material well being of the people

(d)  Real GDP

(4) capital expenditure

44. What is the root cause of all economic problems? (1) Human beings have wants that are unlimited. (2) Resources have alternative uses. (3) Scarcity of Resources (4) Problem of choice

45. Identify the correctly matched items in Column A to that of Column B:

(1) 1 – (a)

(2) 2 – (b)

Column A

Column B

(3) 3 – (c)

(4) 4 – (d)

1.  Social Welfare

(a) Total of economic and non-economic welfare

(2) Equal to one

2. Comparison of GDP of different countries

(b) GDPs evaluated at current market prices

(3) Greater than one

3. Economic welfare

(c)  Situation of unemployment

4.  GDP deflator

(d) Includes prices of imported goods

39. Under monopoly, price elasticity of demand is .....................: (1) Less than one

(4) Infinity

40. Identify the correctly matched pair from Column A to that of Column B: Column A

Column B

(1) 1 – (a)

(2) 2 – (b)

1. Inflationary Gap

(a) Selling of government securities

(3) 3 – (c)

(4) 4 – (d)

2. Deflationary Gap

(2) Increase in Statutory Liquidity Ratio

3. Effects of (c) Rise in production Deficient Demand level 4. Plans to Expand Exports

(d) AD > AS (at full employment level)

(1) 1 – (a)

(2) 2 – (b)

(3) 3 – (c)

(4) 4 – (d)

46. There is a sudden change in climatic conditions resulting in hot weather. Assuming no change in the price of the cold drinks, it will lead to: (1) upward movement along the same market demand curve. (2) downward movement along the same market demand curve. (3) rightward shift in the market demand curve. (4) leftward shift in the market demand curve.

Sample Question Papers I. Read the below case and answer the questions that follow: In a 40 minute long speech Prime Minister Narendra Modi announced the demonetisation of existing notes of ` 500 and ` 1,000 during a televised address on Tuesday evening. Modi announced that the notes of ` 500 and ` 1,000 “will not be legal tender from midnight tonight” and these will be “just worthless pieces of paper. PM also urged people to ‘join this mahayojna against the ills of corruption. – “What is demonetisation and why was it done?”The – Economic Times, Nov 09, 2016

47.  Under whose purview does the issue of new currency fall in? (1) Reserve Bank of India (2) Central Government of India (3) State Government (4) All of the above

48.  Which

two demonetised?

currency

31

denominations

were

(1) ` 100 and ` 500 (2) ` 500 and ` 2,000 (3) ` 1,000 and ` 2,000 (4) ` 500 and ` 1,000

49. ______________ is issued by the government of India. (1) Coins (2) `500 note (3) `1000 note (4) All of these

50. `1 currency note is issued by ______________. (1) Finance Ministry (2) RBI (3) NITI Aayog (4) None of the above



SAMPLE

Question Paper Maximum Marks : 200

7

Time : 60 Minutes

General Instructions : (i) This paper consists of 50 MCQs, attempt any 40 out of 50 (ii) Correct answer or the most appropriate answer: Five marks (+5) (iii) Any incorrect option marked will be given minus one mark (-1) (iv) Unanswered/Marked for Review will be given no mark (0) (v) If more than one option is found to be correct then Five marks (+5) will be awarded to only those who have marked any of the correct options (vi) If all options are found to be correct then Five marks (+5) will be awarded to all those who have attempted the question. (vii) Calculator / any electronic gadgets are not permitted.

1. Which of the following topic is not covered under microeconomics?

(3) Cross- price effect

(4) None of these

7. Law of demand states the ..................... relationship

(1) Product Pricing

between price and quantity demanded.

(2) Factor Pricing

(1) Inverse

(2) Positive

(3) Theory of Distribution

(3) Proportional

(4) None of these

(4) Welfare Economics

8. What is the meaning of production?

2. Bank money is that money which is:

(1) Conversion of inputs into output

(1) Printed by RBI

(2) Conversion of output into input

(2) Printed by the government

(3) Conversion of good into utility

(3) Generated in the form credit creation

(4) None of the above

(4) None of these

9. In monopoly market, there is/are ..................... firm/

3. The balance of trade shows a deficit of `300 crore.

firms.

The value of exports is `500 crore. What is the value of imports?

(1) 1

(2) 2

(3) Few

(4) Many

(1) `800 crores

(2) `200 crores ∆S − ∆ Y (3) `500 crores (4) `300 crores Y S − ∆Yto: 4. Marginal to Save is∆equal ∆S − ∆Propensity Y S Y YS − ∆Y (1) ∆ (2) ∆S S S Y ∆ ∆Y S ∆S (3) S (4) S − Y ∆Y ∆∆Y S S − Y it is collected S −tax Y is called direct because 5. Direct ∆Y directly from: S−Y (1) The producers on goods produced.

10. Identify the correctly matched pair from Column A to that of Column B: Column A

Column B

1.  Excess Demand

(a) Unsold inventories

2. Revenue and Expenditure Policy of Government

(b)  Fiscal policy

(2) The sellers on goods sold.

3. Moral pressure and suasion

(c)  Fiscal policy

(3) The buyers of goods. (4) The income earners.

4. Correction of Inflationary Gap

(d) Government expenditure on welfare

6. .....................refers to the effects of a change in price of commodity X on demand for commodity Y when quantity demanded. (1) Income effect

(2) Price Effect

(1) 1 – (a)

(2) 2 – (b)

(3) 3 – (c)

(4) 4 – (d)

Sample Question Papers 11. Identify the correct pair of items from the following Columns I and II:

33

16. Two friends Komal and Ritika were discussing the Government budget.

‘Through its budgetary policy the government of

Column I

Column II

(1)  Demand Curve

(a) Downward sloping

(2) Indifference curve

(b) Upward rising

The above-mentioned statement was given by

(3) Marginal Utility Curve

(c)  L shaped curve

Komal, identify the objective of Government budget was she talking about?

(4)  Total Utility Curve

(d) Y shaped curve

(1) 1–(a)

(2) 2–(b)

(3) 3–(c)

(4) 4–(d) (2) Narrow

(3) Both (1) and (2)

(4) None of these

13. Identify the correctly matched statements from the Column I to that of Column II: Column I

Column II

(1) Oil Cartels

(a) Monopoly

(2) ISRO

(b) Perfect Competition

(3) DRDO

(c) Monopolistic Competition

(4) Perfume Industry

(c) Monopolistic Competition

(3) 3–(c)

(4) 4–(d)

(2) Activities to secure a reallocation of resources (4) None of these

(1) Broad

(2) 2–(b)

(1) Redistributive activities (4) Stabilising activities

12. M3 and M4 are known as _________ money.

(1) 1–(a)

a country directs the allocation of resources in a manner such that there is a balance between the goals of profit maximisation and social welfare”.

14. Which of the following statement is true about perfect competition? (1) There are large number of buyers and sellers (2) The firms sell differentiated products (3) There is no free entry and exit. (4) Each firms possesses imperfect knowledge

15. Identify the correct pair of items from the following Columns I and II:

17. In case of an under-employment equilibrium, which of the following alternatives is not true? (1) A ggregate Demand is equal to Aggregate Supply. (2) T  here exists excess production capacity in the economy. (3) Resources are not fully and efficiently utilised. (4) Resources are fully and efficiently utilised.

18. Which of the following influence price elasticity of demand?

(1) Nature of the commodity. (2) Income level. (3) vail ability of substitutes.

(4) All of these.

19. A straight line supply curve cuts the Y-axis in its negative range. What is the elasticity of supply? (1) Highly elastic

(2) Unitary elastic

(3) Less elastic

(4) Perfectly inelastic

20. Read the following statements carefully and choose the correct alternatives given below:

Statement 1: Fiscal Deficit = Total Budget Expenditure – Total Budget Receipts (Net of borrowing)

Statement 2: Primary Deficit = Fiscal Deficit + Interest Payments

Alternatives:

Column I

Column II

(1) Perfectly elastic supply

(a) Es > 1

(2) Perfectly inelastic supply

(b) Es < 1

(3) Unitary elastic supply

(c) Es = 1

(4) Relatively elastic supply

(d) Es = 0

(1) 1–(a)

(2) 2–(b)

(3) 3–(c)

(4) 4–(d)

(1) Both the statements are true. (2) Both the statements are false. (3) Statement 1 is true and Statement 2 is false. (4) Statement 2 is true and Statement 1 is false.

21. Identify the correct pair of items from the following Columns I and II: Column I

Column II

(1)  Price Floor

(a) Government imposes below the equilibrium price.

34

OSWAAL CUET (UG) Sample Question Papers, ECONOMICS (2)  Price Ceiling

(b) It is the maximum price, the producers of goods or services are allowed to charge.

(3)  Price Floor

(c) Government does it in the interest of consumers

(4)  Price Ceiling

(d) Government imposes lower limit on the price, which is higher than the equilibrium price.

2. Import of Apache Helicopter from USA

(b) Capital Account of Balance of Payments

3. Remittances from relative from UAE

(c) Debit side of Current Account of Balance of Payments

4. Investment by Tesla Incorporation in India

(d) Credit side of Current Account of Balance of Payments

(1) 1–(a)

(2) 2–(b)

(1) 1 – (a)

(2) 2 – (b)

(3) 3–(c)

(4) 4–(d)

(3) 3 – (c)

(4) 4 – (d)

22. Payment for import of goods is recorded in the _________ side of the Current account of the Balance of Payments. (1) Debit

(2) Credit

(3) Both (1) and (2)

(4) None of these

23. What can lead to change in budget set? (1) When the level of income changes (2) When price of one good changes

29. The market supply of a commodity is affected by: (1) State of technology (2) Number of firms (3) Government policy (4) All of the above

30. Which of the following is an example of complementary goods?

(3) When price of both the goods change

(1) Tea and coffee

(2) Coke and Pepsi

(4) All of the above

(3) Rice and wheat

(4) None of these

24. The value of multiplier is: (1)

1 MPC

1 (3) 1 − MPS

31. Assertion (A): The cement purchased by the (2)

1    MPS

1 (4) MPC − 1

25. Purchase of shares is related to: (1) revenue receipt

(2) revenue expenditure

(3) capital receipt

(4) capital expenditure

26. Choose the correct statement. (1) A technique of production which would maximize output or minimize cost should be used. (2) Human wants are limited. (3) Resources are abundant. (4) Resources do not have alternative uses.

27. In case of underemployment equilibrium: (1) AS < AD (2) there is excess capacity in the economy (3) resources are not fully utilised (4) both (2) and (3)

28. Identify the correct matched pair from Column A to Column B and choose the correct alternative: Column A

Column B

1. Export of weapons to Vietnam

(a) Credit side of Current Account

constructor is considered an intermediate good.

Reason (R): The cement is used for the construction of the building to be sold to the consumer. (1) Both Assertion (A) and Reason (R) are true, and Reason (R) is the correct explanation of the Assertion (A). (2) Both Assertion (A) and Reason (R) are true, but Reason (R) is not the correct explanation of the Assertion (A). (3) Assertion (A) is true, but Reason (R) is false. (4) Assertion (A) is false, but Reason (R) is true.

32. Assertion (A): GDP is the correct measure of the improvement of welfare of the people.

Reason (R): Many activities in an economy are not evaluated in monetary terms, they are not included in GDP due to non-availability of data. (1) Both Assertion (A) and Reason (R) are true, and Reason (R) is the correct explanation of the Assertion (A). (2) Both Assertion (A) and Reason (R) are true, but Reason (R) is not the correct explanation of the Assertion (A). (3) Assertion (A) is true, but Reason (R) is false. (4) Assertion (A) is false, but Reason (R) is true.

33. The government uses _________ as proxy for income of households to identify the poor. (1) Monthly Per Capita Expenditure

Sample Question Papers (2) Monetary Per Capita Expenditure (3) Minimum Per Capita Expenditure (4) None of these

34. M4 = _________+ Total deposits with the Post

Office Saving Organization (excluding National Saving Certificate). (Fill up the blank with correct alternative) (1) M1

(2) M2

(3) M3

(4) None of these

35. If in an oligopoly market, firms produce homogenous product it is called ................. oligopoly. (1) Perfect

36. Identify the correct pair of items from the following Columns I and II:

(3) Budget set

(4) Consumer Budget

38. ____________ is the best known example of a price floor. (1) Minimum wage

(2) Maximum wage

(3) Both (1) and (2)

(4) All of the above

39. Which of the following is not the component of Balance of Payment?

(3) Capital Account

(4) Non-Collusive

(2) Consumer’s Bundle

(4) A  utonomous investment refers to investment which is dependent of the level of consumption in the economy.

(2) Credit Account

(3) Collusive

(1) Marginal Rate of Substitution (MRS)

(3) A  utonomous investment refers to investment which is independent of the level of income in the economy.

(1) Current Account

(2) Imperfect

Column I

35

(4) Official International Reserve Account

40. Under the condition of profit maximisation, MR must be equal to .....................

Column II (a) It states the real employment of the consumer from which he can purchase certain qualitative bundles of three goods at given price (b) It refers to the number of units of good Y which the consumer is willing to gain for an additional unit of good X. (c) It is quantitative combination of those bundles which a consumer can purchase from his given income at prevailing market prices. (d) It is a qualitative combination of three goods which can be purchased by a consumer from his given expenses at given prices..

(1) 1–(a)

(2) 2–(b)

(3) 3–(c)

(4) 4–(d)

37. Which of the following statements is correct? (1) A  utonomous investment refers to investment which is dependent on the level of income in the economy. (2) A  utonomous investment refers to investment which is dependent on the level of consumption in the economy.

(1) AR

(2) MC

(3) AC

(4) TC

41. What is the closest example of a centrally planned economy? (1) United States of America (2) India (3) Soviet Union (4) United Kingdom

42. If reserve ratio is 10% and primary deposit is `1,250 crores, the total deposit created by commercial banks will be _________. (1) `13,500 crores

(2) `10,500 crores

(3) `12,500 crores

(4) `11,500 crores

43. Under long run production function, all the factors of production are______________. (1) Fixed

(2) Variable

(3) Both (A) and (B)

(4) None of these

44. Increase in price of substitute goods leads to: (1) Expansion in demand (2) Increase in demand (3) Decrease in demand (d) Contraction in demand

45. Identify the correctly matched statements from Column I to that of Column II: Column I

Column II

1. Total revenue

(a) Sum of marginal revenues

2. MC > ATC

(b) Stage III

3. MP is negative

(c) Qx = f(L,K)

4. Production Function

(d) ATC must rise

36

OSWAAL CUET (UG) Sample Question Papers, ECONOMICS (1) 1 – a

(2) 2 – b

(3) 3 – c

(4) 4 – d

factor payments in the form of wages, rent, interest and profits, shown by the arrow direction.

46. Identify the correct pair of from the following Columns I and II: Column A

Column B

1. Total Product increases (a) Second at an increasing rate and Stage Marginal Product rises till it reaches its maximum point. 2. Total product increases at a decreasing rate and reaches maximum, and MP becomes zero.

(b)  First Stage

3. Total product also decreases (c)  Third Stage and marginal product (MP) becomes negative. 4. Improvement in technique (d)  Fourth stage of production and discovery of fixed factor substitute can postpone the operation of law for some time. (1) 1 – (a)

(2) 2 – (b)

(3) 3 – (c)

(4) 4 – (d)

47. Money Flows from __________ to ____________ as factor payments. (1) Firms, households (2) Households, firms (3) Government, firms (4) Households, government

48. Circular flow of income refers to the flow of activities of production, income generation and expenditure involving different ___________ of the economy. (2) Aspect

Circular Income Flow in a Two Sector Economy:

In the figure given we can see that upper loop shows the resources such as land, capital and entrepreneurial ability flow from households to firms in the direction shown by the arrow direction. Labour, Land, Capital & Enterprise

(3) Type (4) None of the above

49. Which of the following is not the significance of Circular Flow of Income? (1) It reflects structure of an economy. (2) It shows interdependence among different sectors. (3) It shows injections and leakages from flow of money. (4) It does not help in estimation of national income and related aggregates. 

Wages, Rent, Interest, Profits

Factor payments

50. Which of the following is not an assumption of a Business firm

Households

Thus, we see that money flows from business firms to households as factor payments and then it flows from households to firms. Thus, there is, in fact, a circular flow of money or income. This is how the economy functions.

(1) Sector

II. Read the below case and answer the questions that follow:

The lower part of the figure shows the flow of money from households to firms in the form of consumption expenditure done by the households to purchase the goods and services produced by the firms, making the flow of goods and services from firms to households.

two sector model of Circular Flow of Income? (1) D  omestic economy comprises only 2 sectors, the producers and the households. (2)  The households spend their entire income, so that there is no saving.

Consumption Expenditure



Flow of Goods and Services

The money flows from firms to the households as

(3) D  omestic economy is an open economy (no exports and imports). (4) There is no government in the economy. 



SAMPLE

Question Paper Maximum Marks : 200

8

Time : 60 Minutes

General Instructions : (i) This paper consists of 50 MCQs, attempt any 40 out of 50 (ii) Correct answer or the most appropriate answer: Five marks (+5) (iii) Any incorrect option marked will be given minus one mark (-1) (iv) Unanswered/Marked for Review will be given no mark (0) (v) If more than one option is found to be correct then Five marks (+5) will be awarded to only those who have marked any of the correct options (vi) If all options are found to be correct then Five marks (+5) will be awarded to all those who have attempted the question. (vii) Calculator / any electronic gadgets are not permitted.

1. ______ is Domestic Income:

Column I

(1) GDPMP

(2) NDPFC

(3) NNPMP

(4) GNPFC

2. How do we get the primary deficit from the fiscal deficit?

Column II

1. Production (a) Shows the functional relation Function between physical inputs and physical output of a good. 2. Short Run (b) All the factors of production Production are variable. Function

(1) By adding interest (2) By subtracting interest (3) By adding net exports (4) By subtracting net exports

3. If the amount of LRR is 20% and the initial deposit is ` 1000 crores, then it will create the total amount of money in the economy equal to _________. (1) ` 6000 crores

(2) ` 5000 crores

(3) ` 7000 crores

(4) ` 8000 crores

4. From the set of statements given in Column A and Column B, choose the correct pair of statement:

3. Long Run (c) One factor of production is Production variable and all others are Function fixed 4. Long Run (d) Addition to utility Production Function (1) 1 – (a)

(2) 2 – (b)

(3) 3 – (c)

(4) 4 – (d)

6. If India exports goods worth `20 crores and imports

Column A

Column B

1.  Reduction of pollution

(a)  Microeconomics

(1) Surplus of `10 crores in balance of trade

2. Problems due to unemployment

(b)  Microeconomics

(2) Deficit of `10 crores in balance of trade

3. Shift in the demand curve

(c)  Microeconomics

4. Government expenditure on building of roads

(d)  Microeconomics

goods worth `30 crores, it will have a _________.

(3) Deficit of `50 crores in balance of trade (4) None of these

7. Which other monetary policy instrument the RBI cannot use? (1) Open Market Operations (2) Cash Reserve Ratio

(1) 1 – (a)

(2) 2 – (b)

(3) 3 – (c)

(4) 4 – (d)

5. Identify the correct pair of terms and definitions from the following Columns I and II:

(3) Margin Requirement (4) Deficit Financing

8. ____________ shows the functional relation between physical inputs and physical output of a good.

38

OSWAAL CUET (UG) Sample Question Papers, ECONOMICS (1) Production function (2) Supply function (3) Cost function (4) None of the above

9. What are the four factors of production?

2. Perfectly inelastic supply

(b) Es < 1

3. Unitary elastic supply

(c) Es = 1

4. Relatively elastic supply

(d) Es = 0

(1) Land, Labour, Capital, Income

(1) 1 – (a)

(2) 2 – (b)

(2) Land, Labour, Capital and Entrepreneurship

(3) 3 – (c)

(4) 4 – (d)

(3) Choice, Growth, Opportunity cost, Land (4) Allocation of Resources, Opportunity Cost, Capital

Utilisation,

10. If the percentage increase in the quantity demanded of a commodity is less than the percentage fall in its price, then elasticity of demand is:

17. There is a.....................type of competition in monopolistic competition. (1) Non-Price

(2) Price

(3) Cut throat

(4) Situational

18. The rate of increase in __________________ due

(1) > 1

(2) = 1

to a unit increment in income is called marginal propensity to consume.

(3) < 1

(4) = 0

(1) Ex-post consumption

11. In a/an ..................... market of television, the brand ‘R.G.’ was leading the market share. Its nearest competitor ‘Digi’ suddenly changed the strategy by bringing in a new model of mobile phone at a relatively lesser price. In response ‘R.G.’ too slashed its price. (1) Monopoly (2) Monopolistic Competition

(2) Ex-ante consumption (3) Both (1) and (2) (4) None of the above

19. __________ is the difference between Domestic Income and National Income. (1) NFIA

(2) Net Indirect Tax

(3) Depreciation

(4) All of the above

20. Identify the incorrect statement from the following.

(3) Oligopoly (4) Perfect Competition

12. _________ is the difference between gross and net: (1) Depreciation

(2) NFIA

(3) Net Indirect Tax

(4) Subsidies

13. When there are infinitely small changes in price and demand, then the ..................... method is used. (1) Proportionate

(2) Geometric

(3) Percentage

(4) All of the above

14. Cut in Repo rate by RBI is likely to ____________ the demand for goods and services in the economy. (1) Increase

(2) Decrease

(3) Either (1) or (2)

(4) Neither (1) nor (2)

15. The government fixes the exchange rate in case of

(1) A government budget is an annual financial statement showing item wise estimates of expected revenue and anticipated expenditure during a fiscal year. (2) A government uses monetary instruments of taxation and subsidies with a view of improving the distribution of income and wealth in the economy. (3) A government undertakes commercial activities that are of the nature of natural monopolies; and which are established and managed for social welfare of the public. (4) A government reduces the inequality in the distribution of income and wealth by imposing taxes on the rich and giving subsidies to the poor, or spending more on welfare of the poor.

21. Condition los equilibrium in monopoly

fixed exchange rate regime. (1) The statement is true.

(i) MC is equal to MR

(2) The statement is false as the government fixes the exchange rate in case of flexible exchange rate regime.

(ii) MC curve cuts the MR from below.

(3) The statement is false as the government fixes the exchange rate in case of managed exchange rate. (4) None of the above.

16. Identify the correct pair of items from the following Columns I and II: Column I

Column II

1. Perfectly elastic supply

(a) Es > 1

(1) Option (i) is correct (2) Option (ii) is correct (3) Both options are correct (4) None of the above.

22. A ________ is the lowest legal price that can be paid in a market for goods and services, labour, or financial market. (1) Price ceiling

(2) Price floor

(3) Both (1) and (2)

(4) None of the above

Sample Question Papers 23. The money supply in Indian economy is generally

(1) Decrease

measured in which of the following forms.

(2) Increase

(1) Currency (notes and coins) with the public + Demand deposits + Other deposits held with the Reserve Bank of India.

(3) Remain same

(2) Currency (notes and coins) with the public + Demand deposits + Other deposits held with the Reserve Bank of India + Post Office saving deposits (3) Currency (notes and coins) with the public + Demand deposits + Other deposits held with the Reserve Bank of India + Time deposits of all commercial banks and co-operative banks (excluding interbank time deposits) (4) All of the above

24. Decrease in Cash Reserve Ratio will lead to _______. (1) fall in aggregate demand. (2) rise in aggregate demand. (3) no change in aggregate demand. (4) fall in general price level.

25. Identify the correct pair of items from the following Columns I and II: Column I

Column II

(1)  Utility

(a) Bread and butter

(2)  Normal Goods

(b) Rise in price

(3)  Contraction in demand

(c) Capacity of a commodity to satisfy human wants

(4)  Complementary goods

(d) Positively related

(1) 1 – (a)

(2) 2 – (b)

(3) 3 – (c)

(4) 4 – (d)

26. Which of the following is a component of capital account? (1) Loans

39

(4) Cannot be determined

30. Demand deposits include (1) Saving account deposits and fixed deposits (2) Saving account deposits and current account deposits (3) Current account deposits and fixed deposits (4) All type of deposits

31. When does the government put price ceiling? (1) Government believes that prices are too high. (2) Government believes that prices are too low. (3) Government wants to show who is the boss. (4) None of the above

32. The term “balance of trade” denotes the difference between the exports and imports of _________ in a country. (1) Services

(2) Invisible

(3) Goods

(4) None of these

33. Which of the following is the determinant of market demand? (1) Income of the consumers (2) Season and weather (3) Price of related goods (4) All of the above

34. Read the following statements carefully and choose the correct alternatives given below:

Statement 1: Revenue Expenditure is expenditure incurred for purposes other than the creation of physical or financial assets of the central government.

Statement 2: Revenue Expenditure relates to those expenses incurred for the normal functioning of the government departments.

Alternatives:

(2) Foreign investments (3) Change in Foreign Exchange Reserves (4) All of these

27. The marginal revenue curve shows the relationship between _______and _______. (1) Marginal revenue, quantity produced (2) Marginal Revenue, quantity sold (3) Marginal cost, quantity produced (4) Marginal cost, quantity sold

28. Factor Cost is the: (1) Market Value

(2) Money Value

(3) Transfer Value

(4) None of the above.

29. If increase in demand is greater than the increase in supply, then equilibrium price will:

(1) Both the statements are true. (2) Both the statements are false. (3) Statement 1 is true and Statement 2 is false. (4) Statement 2 is true and Statement 1 is false.

35. Assertion (A): When Average Cost is constant, AC curve is at its minimum point.

Reason (R): At this point, MC curve cuts AC curve, which implies MC = AC. (1) Both Assertion (A) and Reason (R) are true, and Reason (R) is the correct explanation of the Assertion (A). (2) Both Assertion (A) and Reason (R) are true, but Reason (R) is not the correct explanation of the Assertion (A).

40

OSWAAL CUET (UG) Sample Question Papers, ECONOMICS (3) Assertion (A) is true, but Reason (R) is false. (4) Assertion (A) is false, but Reason (R) is true.

36. Assertion (A): Elasticity of supply of gold is unitary elastic.

Reason (R): The unitary elastic supply is equal to

(3) It cannot explain unemployment, poverty, illiteracy, and other problems prevailing in the country. (4) Both (2) and (3)

44. Aggregate Demand is not determined by which of

one. R

the following:

(1) Both Assertion (A) and Reason (R) are true, and Reason (R) is the correct explanation of the Assertion (A).

(1) Consumption Expenditure (2) Investment Expenditure

(2) Both Assertion (A) and Reason (R) are true, but Reason (R) is not the correct explanation of the Assertion (A).

(4) Government Policies

(3) Assertion (A) is true, but Reason (R) is false. (4) Assertion (A) is false, but Reason (R) is true.

37. What value can the depreciation never have? (1) Zero

(2) Positive

(3) Negative

(4) None of the above

38. Break-even point occurs when : (1) Y = S

(2) S = O

(3) Y = C

(4) both (b) and (c)

39. High Powered Money includes: (1) C + DD + OD

(2) C + R + ER

(3) C + R + TD

(4) C + DD + TD

40. The other name for law of production is _________. (1) Law of Producers (2) Law of Variable Proportions (3) Law of Producer’s Variable (4) None of the above

41. Statement 1: Marginal revenue curve is parallel to x-axis.

Statement 2: The slope of marginal revenue curve is upward sloping. (1) Both the statements are true. (2) Both the statements are false. (3) Statement 1 is true but statement 2 is false. (4) Statement 1 is false but statement 2 is true.

42. Which of the following will be excluded when one is calculating National Income through the Value Added Method? (1) Intermediate Good (2) Services for self Consumption (3) Second Hand Goods (4) All of the above.

43. Which of the following is a limitation of microeconomics? (1) It helps in formulating government policies. (2) Microeconomics fails to explain the functioning of an economy as a whole.

(3) Net Exports

45. Identify the correctly matched pair of items in Column A to that in Column B: Column A

Column B

1. Income tax

(a) Revenue receipts

2. Corporation tax

(b) Capital receipts

3. Machinery

(c) Revenue expenditure

4. Borrowing

(d) Revenue receipts

(1) 1 – (a)

(2) 2 – (b)

(3) 3 – (c)

(4) 4 – (d)

46. ___________ is the part of Profit. (1) Corporate Tax

(2) Dividend

(3) Retained Earnings

(4) All of the above

I. Read the following passage and answer the question that follows: The ordinal list revolution originates in the criticism of the psychological foundations of the theory of demand, namely, the principle of decreasing marginal utility as Alfred Marshall ([1890] 1898) used it. The rejection of hedonist hypotheses led Irving Fisher (1892) and Pareto (1896 -97, 1900, 1909) to favour an objective or “positive” approach to economic concepts. The “ordinal list revolution” (Omarzabal 1995, 116) is grounded in a methodological transformation of economics that put the facts of objective experience as a foundation of economics and provided a research program for the ensuing years (Green and Moss 1993; Lewin 1996). Mathematically, ordinalism is entirely based upon the idea that one can dispense with the use of a specific utility function and that no meaning shall be attached to utility measurement, except as an ordinal principle. Clearly, the development of ordinalism must be separated from the introduction of the concept of the indifference curve. Ordinalism was first advocated in Fisher’s “Mathematics Investigations” (1892) and Pareto’s Sunto (1900) and Manual ([1909] 1971), while the indifference curve had appeared in F. Y. Edge worth’s Mathematical Psychics (1881). It was thus only through Fisher’s and Pareto’s recasting that

Sample Question Papers the concept of the indifference curve became irreversibly associated with the promotion of ordinalism. Along the way, the recasting of the theory of choice along ordinal list lines raised a number of issues (about integrability, measurability, and complementarity) that would be progressively settled. A reasonable closing date for the ordinalist revolution is 1950, after Houthakker’s (1950) and Samuelson’s (1950) contributions. From the late 1920s, the Paretian school was progressively gaining a larger audience while the use of the concept of marginal utility and other derivative concepts was challenged. Consequently, demand theory was recast along the principles of individual preferences and ordinal utility functions. Nevertheless, English authors proved very silent about the meaning of indifference curves. Most if not all of the reflections after 1920 about the nature of indifference curves took place in America, mainly under the impulse of Henry Schultz at Chicago. This is an American story. -- Source: hal.archives-ouvertes.fr/hal-01771855/document

47. Which of the following is not the way of studying Utility?

41

(1) Cardinal Approach (2) Ordinal Approach (3) Both (1) and (2)

(4) Neither (1) nor (2)

48. How is utility measured in Ordinal utility theory? (1) In utility (2) Through Indifference curves (3) In rupees (4) Through Accounting

49. Consider the given statement: Under Cardinal Utility approach, utility is measured in utility. (1) The statement is true (2) The statement is false, as in Ordinal Utility the utility is measured in utility (3) The statement is false, as in Cardinal Utility the utility is measured with indifference curves (4) The statement is false as in Cardinal Utility, utility cannot be measured.

50. .....................

is a curve showing different combination of two goods, each combination offering the same level of satisfaction to the consumer. (1) Indifference Curve (2) Marginal Utility Curve (3) Both (1) and (2) (4) None of the above



SAMPLE

9

Question Paper Maximum Marks : 200

Time : 60 Minutes

General Instructions : (i) This paper consists of 50 MCQs, attempt any 40 out of 50 (ii) Correct answer or the most appropriate answer: Five marks (+5) (iii) Any incorrect option marked will be given minus one mark (-1) (iv) Unanswered/Marked for Review will be given no mark (0) (v) If more than one option is found to be correct then Five marks (+5) will be awarded to only those who have marked any of the correct options (vi) If all options are found to be correct then Five marks (+5) will be awarded to all those who have attempted the question. (vii) Calculator / any electronic gadgets are not permitted.

1. Which of the following is excluded while calculating National Income as per expenditure method? (1) Purchase of Second Hand goods (2) Purchase of financial Assets (3) Intermediate goods (4) All of the above

2. Construction of railway line is a type of _________ expenditure. (1) Capital

(2) Revenue

(3) Primary

(4) All of these

3. Identify the correctly matched items from Column I to that of Column II:

(3) Total deposits organizations (4) None of these

with

Post

Office

savings

5. ___________ can be referred to as functional relationship between supply of a commodity and the factors determining it. (1) Supply schedule

(2) Supply curve

(3) Supply function

(4) Supply

6. Which is a component of Budget? (1) Budget Receipts

(2) Budget Expenditure

(3) Both (1) and (2)

(4) None of the above

7. What happens when the Investment is lesser than Savings?

Column I

Column II

(1) Demand Curve of Perfect Competition

(a) V-shaped Curve

(1) A  ggregate Demand is more than the Aggregate Supply.

(2) Demand Curve of Monopoly

(b) U-shaped Curve

(2) A  ggregate Demand is less than the Aggregate Supply

(3) Demand Curve of Monopolistic Competition

(c) Upward rising

(3) A  ggregate Demand is equal to the Aggregate Supply.

(4) Demand Curve of Oligopoly

(d) In-determinant

(4) A  ggregate Demand is independent of Aggregate Supply.

(1) 1 – (a)

(2) 2 – (b)

(3) 3 – (c)

(4) 4 – (d)

4. The word ‘net’ implies that _________ are to be included in money supply. (1) Only deposits of the public held by the banks (2) Savings deposits with Post Office savings banks

8. The simplest consumption function assumes ______. (1) Consumption changes at a constant rate as income changes (2) Consumption changes faster with more income (3) Consumption does not change with income. (4) Consumption changes with more supply.

9. ___________ states that, other things being equal, quantity supplied increases with increase in price and decrease with decrease in price of a commodity.

Sample Question Papers (1) Law of supply

(4) Price Elasticity of Demand is defined as the measurement of percentage in quantity produced in response to a given percentage change in own price of the commodity.

(2) Law of variable proportion (3) Cost function (4) Law of returns to scale

10. __________ is the output at base year prices. (1) Nominal GDP

(2) Real GDP

(3) National GDP

(4) None of the Above

11. Oligopoly is a form of market in which there are ..................... big sellers of a commodity and ..................... number of buyers. (1) few, large

15. The supply curve of a good shift to the right when prices of other goods ..................... (1) Rise

(2) Fall

(3) Remains the same

(4) Increases

16. Marginal revenue equals ________ price. (1) Market

(2) Industry

(3) Consumer

(4) None of the above

17. Consider the following statements:

(2) large, few

I.

(3) large, large

M1 is a broad concept of money.

II. M2 is a narrow concept of money.

(4) one, many he above

12. Identify the correctly matched pair of items of Column A to that of Column B: Column A

Column B

III. M3 includes both M1 and lime deposit of public with bank

Which of the following is true with respect to the concept of supply of money?

1. Lending to Kenya

(a) Capital Account

(1) I and II

(2) I only

2. Export of Banking services

(b) Visible item

(3) II and III

(4) II only

3. Import of aircraft

(c) Invisible item

4. Foreign aid

(d) Visible item

18. Identify the correctly matched pair of Column A to

(1) 1 – (a)

(2) 2 – (b)

(3) 3 – (c)

(4) 4 – (d)

13. Growth of GDP and Major Sectors (in %) Column I

Column II

(1) Feature of Perfect Competition

(a) Only one buyer.

(2) Feature of Monopoly Market

(b) Only few sellers

(3) Feature of Monopolistic (c) Large Number Competition of Buyers and Sellers (4) Feature of Monopoly Market

43

(d) Only few buyers

(1) 1 – (a) (2) 2 – (b) (3) 3 – (c) (4) 4 – (d) 14. Which of the following statement is true? (1) Price Elasticity of Demand is defined as the measurement of percentage in quantity demanded in response to a given percentage change in own price of the commodity. (2) Price Elasticity of Demand is defined as the measurement of percentage in quantity supplied in response to a given percentage change in own price of the commodity. (3) Price Elasticity of Demand is defined as the measurement of percentage in quantity demanded in response to a given percentage change in own production of the commodity.

that of Column B: Column A 1. Foreign Trade

Column B (a) Export Processing Zone 2. Adverse Balance (b) Problem of India’s of Trade Foreign Trade 3. Import (c) Benefits to Substitution participating countries 4. Export Promotion (d) Scarcity of Foreign Trade (1) 1 – (a)

(2) 2 – (b)

(3) 3 – (c)

(4) 4 – (d)

19. What does Real GDP show? (1) Change in price only (2) Change in output only (3) Change in both price and output (4) None of the Above.

20. Identify the correct pair of items from the following Columns I and II: Column I

Column II

(1)  Budget Line

(a)  Normal goods

(2) Bajra

(b) Inferior goods

(3)  Consumer equilibrium (c) Luxurious goods (4)  Elastic Demand

(d) M = Px•x + Py•y

(1) 1 – (a)

(2) 2 – (b)

(3) 3 – (c)

(4) 4 – (d)

44

OSWAAL CUET (UG) Sample Question Papers, ECONOMICS

21. What is saving per Income called?

(3) Assertion (A) is true, but Reason (R) is false.

(1) APC

(2) APS

(4) Assertion (A) is false, but Reason (R) is true.

(3) MPC

(4) MPS

24. _________ refers to situation of excess of imports of goods over exports of goods.

22. Assertion (A): The demand curve is downward

(1) Balance of Payment Deficit

sloping.

(2) Fiscal Deficit

Reason (R): Income effect means with fall in the price of a good, consumer’s real income or purchasing power rises and he demands more units of the good. (1) Both Assertion (A) and Reason (R) are true, and Reason (R) is the correct explanation of Assertion (A). (2) Both Assertion (A) and Reason (R) are true, but Reason (R) is not the correct explanation of Assertion (A).

(3) Balance of Trade Deficit (4) All of these

25. The slope of the Total Revenue equals________. (1) Average Revenue

(2) Marginal Revenue

(3) Average cost

(4) Marginal Cost

26 Savings and Investment affect the _____________. (1) Aggregate demand and supply (2) Government policies (3) Government budget

(3) Assertion (A) is true, but Reason (R) is false.

(4) None of the above

(4) Assertion (A) is false, but Reason (R) is true.

23. Assertion (A): A lower indifference curve represents

27. Statement 1: A rational firm aims to operate in second phase of law of variable proportion.

a higher level of satisfaction. Reason (R): According to the Indifference Curve Approach, utility is an ordinal concept, that is, it can be ranked and not measured.

Statement 2: In diminishing returns to a factor, i.e., second phase of law of variable proportion, MP falls.

(1) Both Assertion (A) and Reason (R) are true, and Reason (R) is the correct explanation of Assertion (A).

(1) Both the statements are true. (2) Both the statements are false. (3) Statement 1 is true but statement 2 is false.

(2) Both Assertion (A) and Reason (R) are true, but Reason (R) is not the correct explanation of Assertion (A).

(4) Statement 1 is false but statement 2 is true.



(` in lakh crore)

NET RECEIPT OF THE CENTRE 25

1.9

20 0.7

15 10

0.4 0.2

0.4 1.4

0.4 2.0

0.5 2.0

0.6

2.7

1.2 1.9

1.1 2.4

0.7 3.3

0.5 2.1

2.4

2.5

5 0

2011-12

2012-13

2013-14

2014-15

2015-16

Net-Centre's Tax Revenue

2016-17

2017-18

2018-19

2019-20

RE 20-21 BE 21-22

Non Tax Revenue

Non Debt Capital Receipt

Source: www.indiabudget.gov.in On the basis of the above-mentioned information answer the following question: 78. The value of non-tax revenue has _________ crores 29. Consider the following statements: between 2016-17 and 2017-18.

(1) Use of public parks increases welfare

(1) Fallen by 0.8

(2) Risen by 0.8

(2) Distribution of GDP increases welfare

(3) Fallen by 0.6

(4) Risen by 0.6

(3) Higher GDP always cause higher welfare

Sample Question Papers Which of the above statements are false? (1) Only 1 (2) Only 2 (3) Only 3 (4) 1 and 3 30. ..................... products are sold under perfect competition. (1) Differentiated (3) Different

(2) Homogeneous (4) All of the above.

31. Investment Multiplier is the ratio of change in ___________ and _____________. (1) Income, Investment (2) Savings, Income (3) Investment, Consumption (4) Savings, Investment

32. Identify the correct pair of items from the following Columns I and II: Column I 1. Total Product

Column II (a) Total input produced by a firm during a given period of time with given number of outputs. 2. Total Physical (b) Total output produced Product by a firm during a given period of time with given number of inputs. 3. Average Product (c) The change in Total Product resulting from the use of one more (or one less) unit of the variable input, keeping all other inputs constant. 4. Marginal Product (d) Output per unit of a variable input. (1) 1 – (a) (3) 3 – (c)

(2) 2 – (b) (4) 4 – (d)

33. Read the following statements carefully and choose the correct alternatives given below:

Statement 1: Current account records exports and imports of visible and invisible items only.

Statement 2: Balance of current account is the difference between sum of credit items and sum of debit items on current account.

Alternatives: (1) (2) (3) (4)

Both the statements are true. Both the statements are false. Statement 1 is true and Statement 2 is false. Statement 2 is true and Statement 1 is false.

34. Identify the correct pair of items from the following Columns I and II:

45

(3)  Vegetable Market (c) Perfect Competition Market (d) Perfect Competition Market

(4)  Railways (1) 1 – (a)

(2) 2 – (b)

(3) 3 – (c)

(4) 4 – (d)

35. How is the interest earned by normal resident treated? (1) Included in National Income (2) Included in Domestic income (3) Excluded in National Income (4) Excluded in Domestic Income

36. Identify the correct pair of items from the following Columns I and II: Column I 1. Price taker firm

2. Homogeneous product 3. Price Maker

4. Heterogeneous Product

Column II (a) One who has an option to accept the price by their competitors. (b) The products are same or similar. (c) One who has no option but to accept the price determined by the industry. (d) The products are different.

(1) 1 – (a)

(2) 2 – (b)

(3) 3 – (c)

(4) 4 – (d)

37. The production function exhibits technological relationship between physical inputs and output. (1) True

(2) False

(3) Partially true

(4) None of the above

38. The main principle underlying the cost classification is the main element of the cost in ________ and ________ cost basis. (1) fixed, opportunity

(2) fixed, implicit

(3) implicit, explicit

(4) fixed, variable

39. Kalpana said that Commercial Bank is the controller of credit. Geeta said that the Central Bank is the controller of credit. Bobby said that both the commercial bank and the Central Bank are the controller of credit. Who among them are not correct? (1) Kalpana and Geeta

(2) Kalpana and Bobby

(3) Geeta and Bobby

(4) Only Geeta

Column I

Column II

(1)  Tea Shops

(a) Perfect Competition Market

example of:

(2)  Cafeteria

(b) Perfect Competition Market

(3) Revenue receipts

40. Subsidies and expenditure on scholarship are (1) Revenue Expenditure (2) Capital Expenditure (4) Capital Receipts

46

OSWAAL CUET (UG) Sample Question Papers, ECONOMICS

41. Which of the following have elastic demand? (1) Match box

(2) Water

(3) Medicines

(4) Air conditioners

42. What does the Factor Cost represent? (1) Net Indirect Tax of all final goods and services. (2) Depreciation of all final goods and services. (3) Money Value of all final goods and services. (4) None of the above.

43. As we move along a downward sloping straight line demand curve from left to right, price elasticity of demand ...................... (1) Goes on falling (2) Goes on rising (3) Remains the same (4) None of the above

44. Aggregate Demand is not determined by which of the following: (1) Consumption Expenditure (2) Investment Expenditure (3) Net Exports (4) Government Policies

(1) Centrally planned economy (2) Socialist economy (3) Mixed economy

48. A professor can do three jobs --teaching, tuition

house a part of? (1) Mixed Income of self-employed (2) Domestic Income (3) National Income (4) All of the Above

46. Identify the correctly matched items from Column I to that of Column II:



47. The problem of choice is faced by:

(4) All of the above

45. What is the income of a teacher teaching at his own

Column I

Economic problem is the problem of choice. The problem of choice has to be faced by every economy of the world, whether developed or developing. Human beings have wants which are unlimited. Economic problems are reflected in the form of Central or Basic Problems of an economy. Any economy --whether market, centrally planned, or mixed --has to face these problems. In economic analysis, the concept of opportunity cost is widely used. Opportunity cost is defined as the cost of alternative opportunity given up or surrendered.

Column II

work and writing books. He gets ` 2 lakhs from teaching, ` 1.5 lakhs from tuition work and ` 2.5 lakhs from the royalty of books. He is presently teaching. What is the opportunity cost of writing books? (1) Tuition work (2) Teaching (3) Switching profession (4) Writing books

49. Consider the following statements: Statement 1: Resources or factors of production can

(1) Feature of Monopoly Market

(a) There is only one seller.

(2) Advantage of Monopoly Market

(b) Government policies and restriction.

(3) Limitation of Monopoly Market

(c) Free entry and exit of the firm.

(2) Both Statements are false

(d) Feature of Perfect Competition

(4) More than one seller and only one buyer.

(4) Statement (2) is only true

be natural, human, capital, and entrepreneurial.

Statement 2: An economy must make a choice of the wants which are important for the economy. (1) Both Statements are true (3) Statement (1) is only true

50. Which system of economy does India follow?

(1) 1 – (a)

(2) 2 – (b)

(1) Mixed

(3) 3 – (c)

(4) 4 – (d)

(2) Centrally planned

I. Refer to the passage below and answer the following questions.

(3) Market (4) None of these



SAMPLE

Question Paper Maximum Marks : 200

10

Time : 60 Minutes

General Instructions : (i) This paper consists of 50 MCQs, attempt any 40 out of 50 (ii) Correct answer or the most appropriate answer: Five marks (+5) (iii) Any incorrect option marked will be given minus one mark (-1) (iv) Unanswered/Marked for Review will be given no mark (0) (v) If more than one option is found to be correct then Five marks (+5) will be awarded to only those who have marked any of the correct options (vi) If all options are found to be correct then Five marks (+5) will be awarded to all those who have attempted the question. (vii) Calculator / any electronic gadgets are not permitted.

1. Economists like Adam Smith follow which school of economics? (1) Positive

(2) Normative

(3) Both

(4) None of these

2. Which of the following statements is true?  (1) W  hen MPS = 1, the value of investment multiplier is also 1. (2) W  hen MPS = 1, the value of investment multiplier is 0.

7. Identify the objective of budget: (1) Activities to secure a reallocation of resources (2) Redistributive activities (3) Stabilizing activities (4) All of the above

8. Which of the following does the Market Price include? (1) Net Factor Income From Abroad

(3) W  hen MPS = 0.5, the value of investment multiplier is 1.

(2) Depreciation

(4) W  hen MPS = 1, the value of investment multiplier is 0.5.

(4) All of the above

3. If the total deposits created by commercial banks is ` 10,000 crores and legal reserve requirements is 10%, the amount of initial deposits will be _________. (1) ` 1,000 crores

(2) ` 2,000 crores

(3) ` 5,000 crores

(4) ` 9,000 crores

4. Which of the following is not a revenue receipts ? (1) Recovery of Loans (2) Foreign Grants (3) Profits of Public expenditure (4) Wealth Tax

5. If a good takes up significant share of consumers’ budget, it will be: (1) Less elastic

(2) Highly elastic

(3) Unitary elastic

(4) Perfectly elastic

6. Market demand curve is obtained by ..................... summation of the individual demand curves. (1) Vertical

(2) Horizontal

(3) Both (1) and (2)

(4) None of these

(3) Net Indirect Tax

9. Identify the correctly matched items from Column I to that of Column II: Column A 1.  Variable factor 2. TC

Column B (a) Machinery (b)  FC + VC

3. AR (c) MR/Q 4. Increasing returns to a (d)  TP decreases factor (1) 1 – (a) (2) 2 – (b) (3) 3 – (c) (4) 4 – (d)

10. If MPS = 0, the value of multiplier will be: (1) 2 (3) 0

(2) 1 (4) ∞

11. The _________ is usually instituted via law and is typically applied to necessary goods like food, rent, and energy sources in order to ensure that everyone has access to them. (1) Price floor

(2) Price ceiling

(3) Both (1) and (2)

(4) None of the above

OSWAAL CUET (UG) Sample Question Papers, ECONOMICS

48

12. Saving deposit in the Post office is a part of: (1) M1

(2) M2

(3) Both (1) and (2)

(4) None of these

(1) 1 – (a) (2) 2 – (b) (3) 3 – (c) (4) 4 – (d)

18. Which of the following is not the source for capital

13. What concept are all domestic variants?

receipt of the government?

(1) National (2) Territorial (3) Economical (4) All of the above 14. What all is covered under the allocation of resources?

(1) Dividend from public sector undertakings

(1) (2) (3) (4)

What goods to produce and how much? How to produce? For whom to produce? All of the above

15. The fixed cost curve is parallel to _______ and variable cost curve is _________ shaped. (1) x-axis, U-shaped (2) x-axis, inverse S-shaped

(2) Recovery of loans (3) Sale of shares of public sector undertaking (4) Market borrowings

19. In case of Gross Domestic Capital Formation, which one will be the part of it? (1) Gross Fixed Capital Formation (2) Inventory Investment (3) Both (1) and (2) (4) Neither (1) nor (2)

20. _________ are recorded as negative (debit) items.

(3) y-axis, U-shaped

(1) Imports

(2) Exports

(4) y-axis, Inverse S-shaped

(3) Both (1) and (2)

(4) None of these

16. What is the other name for Income Method? (1) Factor Payment Method

21. It is seen that the private consumption expenditure,

(2) Factor Service Method

private investment expenditure and ex-ante savings has reduced the ________________ in the economy.

(3) Factor Output Method

(1) Aggregate demand (2) Aggregate supply

(4) None of the Above.

(3) Investment

17. Identify the correct pair of items from the following Columns I and II: Column II

(1) Monotonic Preferences

(a) Consumer’s preferences are called monotonic when between any three bundles, consumer always choose a bundle having more of one good and no less of other goods.

(3) Indifference Curve

(4) Indifference Map

22. How do the producers respond to the situation of price ceiling?

Column I

(2) Indifference Set

(4) None of the above

(b) It is a set of those division of two goods which offer the consumer the same level of satisfaction, so that the consumer is indifferent across any number of combinations in his indifference set. (c) It is a curve showing different combination of two goods, each combination offering the same level of satisfaction to the consumer. (d) It refers to a set of indifference curves placed in different diagrams for the same type of goods.

(1) Rationing supplies (2) Decreasing production level (3) Lowering the quality of production (4) All of the above

23. ..................... states that as more and more units of a commodity are consumed, marginal utility derived from additional unit must decline. (1) Law of Diminishing Marginal Utility (2) Law of Supply (3) Law of Variable Proportions (4) Law of Equi-marginal utility.

24. Machine is a _________ item. (1) Invisible

(2) Visible

(3) Unilateral

(4) None of these

25. By lowering costs, price ceilings also have the beneficial effect of helping to stimulate _______. (1) Demand

(2) Supply

(3) Cost

(4) All of the above

26 __________ is shown by Nominal GDP. (1) Change in price only (2) Change in Output only (3) Both (1) and (2) (4) Neither (1) nor (2)

27. Commercial Bank was facing some financial crisis due to the crisis of non-performing assets. It went

Sample Question Papers to the RBI for help. What role will RBI play in such a case? (1) It acted as a regulator of the Commercial Banks. (2) It acted as a banker to the Commercial Banks. (3) It acted as a supervisor to the commercial bank. (4) It acted as a lender of last resort.

28. Which of the following statement is false? (1) PPC slopes downwards

Column I

Column II

(1) Railways

(a) Monopoly Market.

(2) TV seller

(b) Perfect Competition Market.

(3) Microwave oven

(c) Monopsony Market.

(4) Fruits Market

(d) Monopoly Market.

49

(2) PPC is concave (3) PPC slopes upwards

(1) 1 – (a) (2) 2 – (b)

(4) None of these

(3) 3 – (c) (4) 4 – (d)

29. What is the circumstance when aggregate output is

36. ..................... market has the characteristic of both

determined solely by the level of aggregate demand called?

monopoly and perfect competition.

(1) Investment Multiplier

(2) Monopsony

(2) Effective Demand

(3) Duo-poly

(3) Propensity to Consume

(4) Oligopoly

(1) Monopolistic Competition

37. Balance of payments is an accounting statement that

(4) Propensity to Save

30. Disinvestment is a _________ receipt. (1) Revenue

(2) Capital

(3) Primary

(4) None of the above

31. Identify the correct pair of items from the following Columns I and II: Column I 1.  MC cuts AC 2.  Derived from TC

Column II (a) MC is greater than AC (b)  Opportunity cost

3.  When AC falls 4.  When AC rises

(c) MC is lower than AC (d) At its minimum point.

(1) 1 – (a) (2) 2 – (b) (3) 3 – (c) (4) 4 – (d)

32. In a system of flexible exchange rates, the exchange rate is determined by the forces of:

provides a systematic record of all the _________ transactions between the residents of a country and the rest of the world during a given period of time. (Fill up the blank with correct alternative) (1) Non-economic

(2) Economic

(3) Social

(4) None of these

38. When calculating the national Income which of the following will not be considered? (1) Windfall gains (2) National Debt Interest (3) Purchase of second hand goods (4) All of the above.

39. Assertion (A): TU curve starts from the origin, increase at a decreasing rate, reaches a maximum and then starts falling.

(1) Market demand

(2) Market supply

Reason (R): The rising MU curve shows the law of

(3) Both (1) and (2)

(4) None of these

diminishing marginal utility. (1) Both Assertion (A) and Reason (R) are true, and Reason (R) is the correct explanation of Assertion (A). (2) Both Assertion (A) and Reason (R) are true, but Reason (R) is not the correct explanation of Assertion (A). (3) Assertion (A) is true, but Reason (R) is false. (4) Assertion (A) is false, but Reason (R) is true.

33. The impact of ‘Excess Demand’ under Keynesian theory of income and employment, in an economy are : (1) D  ecrease in income, output, employment and general price level. (2) D  ecrease in nominal income, but no change in real output. (3) Increase in income, output, employment and general price level. (4) N  o change in output/employment but increase in general price level.

34. Total utility is ................... at the point of satiety. (1) Minimum

(2) Maximum

(3) Zero

(4) None of these

35. Identify the correctly matched items from Column I to that of Column II:

40. Assertion (A): MRS is defined as the amount of good Y given up in exchange for good X such that total utility is constant.

Reason (R): Diminishing MRS means the consumer wants to give up lesser units of Y in exchange for good X. (1) Both Assertion (A) and Reason (R) are true, and Reason (R) is the correct explanation of Assertion (A).

50

OSWAAL CUET (UG) Sample Question Papers, ECONOMICS (2) Both Assertion (A) and Reason (R) are true, but Reason (R) is not the correct explanation of Assertion (A).

(3)  Implicit Cost

(d)  Hidden Cost (d) The expenses incurred by the producer when the inputs are purchased or hired from the black market.

(3) Assertion (A) is true, but Reason (R) is false. (4) Assertion (A) is false, but Reason (R) is true.

41. Read the following statements and choose the correct alternative from the following:

Statement 1: Price floor is imposed by the government on agriculture goods, medicine, and education, so as to make the needed goods or services more affordable.

Statement 2: Price ceiling is imposed by the government on agriculture goods, medicine, and education, so as to make the needed goods or services more affordable. (1) Both the statements are true. (2) Both the statements are false. (3) Statement 1 is true but statement 2 is false. (4) Statement 1 is false but statement 2 is true.

42. Considering operating surplus which one of the following is not a part of it? (1) Rent and Royalty (2) Wages and Salaries (3) Interest and Profits (4) None of the above

43. Which of the following statement is false? (a) When MU is negative, TU will be increasing. (b) When MU is positive, TU will be decreasing. (c) When MU is zero, TU is maximum. (d) None of these

44. _________ are known as narrow money. (1) M3 and M2

(2) M4 and M2

(3) M1 and M2

(4) M1 and M4

45. The difference by which actual Aggregate Demand exceeds the Aggregate Demand, required to establish full employment equilibrium is known as ______________. (1) Inflationary gap

(2) Deflationary gap

(3) Either (1) or (2)

(4) Neither (1) nor (2)

46. Identify the correct pair of items from the following Columns I and II:

(c) The value of a factor in its next best alternative use.

(1) 1 – (a) (2) 2 – (b) (3) 3 – (c) (4) 4 – (d) I. Read the following passage and answer the questions that follows:

Agriculture provides livelihood to almost threefourth of population of India. Indian agriculture is highly dependent on spatial and temporal distribution of rainfall. Climate extremes such as drought and flood affect agriculture severely. An account of impact of climate extreme viz. drought and flood, on Indian food-grain production has been presented in this chapter. There are temporal fluctuations in food grain production and area under the food-grain. In secular terms, both of them increased up to mid-eighties. After mideighties there is decline in the area of food grain while maintaining an increase in production of food-grain suggesting the improvement in agricultural technology and policy. There is more temporal fluctuation in the production of food grain than the area under food grain. The analysis reveals that impact of drought on Indian agriculture is more than that of flood. Rabi food grain production depicts better adaptability to drought than Kharif food grain production mostly due to better access to irrigation infrastructure. Among the various food crops analysed all except jowar can effectively face flood events. Wheat and jowar perform relatively better during drought events. Rice is most sensitive crop to the extreme climate events. Since rice is staple food in the subcontinent, management of rice productions against climate extremes needs special attention for food security and sustainability.

47. What has caused the increase in supply of food grains.

Column I

Column II

(1) Improvement in technology

(1) Opportunity Cost

(a) The value of a factor in its next worst alternative use.

(3) Better agricultural policy

(2)  Explicit Cost

(b) The expenses incurred by the producer when the inputs are purchased or hired from the market.

(2) Increase in production (4) All of the above

48. The contribution of agriculture towards generating employment opportunities is : (1) 3/4th

(2) 2/3rd

rd

(4) 1/4th

(3) 1/3

Sample Question Papers 49. Statement 1: Till the mid-eighties in secular terms, there was increase in production of food grain and area under the food-grain. Statement 2: After mid-eighties, area under food grain increased. (1) Both the statements are true. (2) Both the statements are false. (3) Statement 1 is true but statement 2 is false.

51

(4) Statement 1 is false but statement 2 is true.

50. Which among the following factors affect the supply of food-grain production? (1) Cost of fertilizers (2) Better irrigation facilities (3) Use of agricultural tools or instruments (4) All of the above



SOLUTIONS OF Question Paper 1. Option (1) is correct. Explanation: Inventory is a stock concept as it can be measured at a point of time and change in inventory is a flow concept because it can be measured over a time period.

2. Option (1) is correct. Explanation:

1

8. Option (1) is correct. Explanation: Perfect competition is a situation in which there are large number of buyers and sellers. Each seller has a very small proportion in the market. And since the share of each seller in the market is less. Market forces of demand and supply decide the equilibrium price and equilibrium quantity in the industry and that price is taken by each firm in the perfect competition. Hence, firm is a price taker.

9. Option (2) is correct.

3. Option (2) is correct. Explanation: Flows of Government Budget (1) Minimize fiscal deficit (2) Reallocation of resources (3) Economic growth and Stability.

4. Option (1) is correct. Explanation: It is an inflow of money and it is recurring in nature.

5. Option (1) is correct. Explanation: Money Supply is the total stock of money circulating in the economy at a particular point of time including currency and other liquid instruments.

6. Option (1) is correct. Explanation: The study of population in India involves the study of the economy as a whole, thus it is an example of study of macroeconomics.

7. Option (2) is correct. Explanation : Properties or Characteristics of Indifference Curves : (i) It slopes downwards from left to right. (ii) Indifference Curves are convex to the origin. (iii) Indifference Curves will never intersect each other. (iv) A higher Indifference Curve represents higher level of satisfaction. (v) Indifference Curve neither touch to each x-axis nor y-axis.

Explanation: S APS = Y S 0.8 = 4 ,000 S = ` 800 crores C= Y − S = ` 4,000 − ` 800 = ` 3,200 crores

10. Option (3) is correct. Explanation: Selling cost is the cost incurred on advertisement of goods. Selling cost is zero in case of perfect competition because of perfect knowledge of buyers and sellers. Because products are homogeneous and they are perfect substitutes of each other.

11. Option (2) is correct. Explanation: A direct tax is a tax, the burden of which is on that very person who is liable to pay it to the government.

12. Option (2) is correct. Explanation: Normal goods are those goods whose demand rises with rise in income and falls with fall in income. When income increases in case of normal goods, then demand rises and hence demand curve shifts to the right. Hence, equilibrium price and equilibrium quantity both rise.

13 Option (4) is correct. Explanation: Full employment equilibrium refers to the situation where aggregate demand is equal to aggregate supply and all those who are equal to work and willing to work (at existing wage rate) are getting work.

Solutions 14 Option (2) is correct. Explanation: Under monopoly, the firm faces a demand curve, which slopes downwards from left to right. It implies monopoly firm can sell more only at lower price.

15. Option (4) is correct. Explanation: There are certain limitations of GDP as an index of welfare: (1) GDP does not take into account changes in inequalities in the distribution of income. (2)  GDP does not include non-monetary exchanges. (3) GDP does not consider the changes in the population of a country. (4) GDP does not account for valuation of externalities.

16. Option (1) is correct. Explanation: Import of goods and services are recorded in current account. Import is recorded on the debit side as it leads to an outflow of foreign exchange in the country.

17. Option (1) is correct. Explanation: Money supply is the total stock of money in circulation held by public at a given point of time.

18. Option (3) is correct. Explanation: There is a difference when we talk about in-elasticity and perfect in-elasticity. When a commodity has few uses, it’s demand is inelastic because the prices are less, demand increases but since the commodity has a few uses only there will be a slight increase.

19. Option (1) is correct. Explanation: Area which lies below the MC curve is the total variable cost because MC is calculated by subtracting TVC as follows:MC = TVCn - TVCn-1 Hence, area covered under MC is TVC.

20. Option (2) is correct. Explanation: Negative externalities refer to negative impact of an economic activity on the others without involving any penalty.

21. Option (1) is correct. Explanation: In case of direct tax, the burden of tax and the liability to pay it falls on the same person.

22. Option (2) is correct. Explanation: Coins in India are issued by the Government of India. It is the sole responsibility of the Government of India to mint coins of all denominations.

53

23. Option (1) is correct. Explanation: The central problems under `problem of allocation of resources’ are : (i) What to produce ? (ii) How to produce ? (iii) For whom to produce ?

24. Option (2) is correct. Explanation: Under monopoly, the firm faces a demand curve, which slopes downwards from left to right. It implies monopoly firm can sell more only at lower price.

25. Option (2) is correct. Explanation: Macroeconomics is the study of behaviour of an economy as a whole. National income is the value of total final goods produced by an economy. Thus, the study of National Income is related to macroeconomics.

26. Option (3) is correct. Explanation: The utility approach refers to a consumer’s realised happiness when he is willing to spend money on a stock of commodity that can satisfy his need. The indifference curve approach gives a tabular presentation of different combinations of two goods that yield the same level of satisfaction to the consumer. Both of them help a consumer in realising the amount of commodity they require to avail satisfaction.

27. Option (1) is correct. Explanation: Devaluation refers to fall in the value of domestic currency due to deliberate increase in foreign exchange rate by the government which follows fixed exchange rate system.

28. Option (3) is correct. Explanation: Contraction in supply can be defined as due to fall in price of the commodity, there is fall in quantity supplied of a commodity, i.e., there is downward movement along the same supply curve.

29. Option (3) is correct. Explanation: Increase in the price of substitute goods leads to increase in the demand for other goods. Inferior goods are the opposite of normal goods, its demand increases with the rise in income and decreases with fall in income.

54

OSWAAL CUET (UG) Sample Question Papers, ECONOMICS

30. Option (1) is correct. 31. Option (1) is correct. Explanation: Features of Monopolistic market (1) Highly Differentiated Product (2) Many buyer and Seller (3) Low barrier to entry and exit (4) Non price competition

32. Option (3) is correct. Explanation: Full employment equilibrium refers to a situation when equilibrium is attained i.e., aggregate demand is equal to aggregate supply at full employment level.

33. Option (4) is correct. Explanation: When there is excess of imports over exports, it is called an unfavourable balance of trade.

34. Option (3) is correct. Explanation: The two cost curves which start from the same point on the Y-axis are total fixed cost and total cost. TC is sum of TFC and TVC. Since at zero level of output, TVC is zero and starts from origin. Thus, at zero level of output, TC = TFC and start from the same point on the y-axis.

35. Option (1) is correct. Explanation: In case of deficient demand, there is a need to liberalize credit. It can be done by reducing bank rate so that the commercial banks also reduce their lending rate, thereby increasing the availability of credit in the economy.

36. Option (2) is correct. Explanation: Features of Monopolistic market (1) Consumption based tax . (2) One tax rate across the country. (3) No tax on tax. (4) Simplify taxation process.

37. Option (1) is correct. Explanation: Production Possibility Curve refers to the graphical representation of possible combinations of two goods that can be produced with given resources and technology. PPC slopes downwards from left to right. It is because in a situation of fuller utilisation of given resources, production of both the goods cannot be increased. More of Goods X can be produced only with less of Goods Y.

38. Option (3) is correct. Explanation: When there are no goods or services exchanged for the payment of money, it is known as transfer payment.

39. Option (2) is correct. Explanation: RBI should release money as it will increase money supply in the economy, and thus in turn will increase production activities.

40. Option (1) is correct. Explanation: Depreciation of home currency implies fall in the price of domestic goods for the foreign buyers.

41. Option (3) is correct. Explanation: Oligopoly is a form of market in which there are few big sellers of a commodity and a large number of buyers. Each seller has a significant share of the market.

42. Option (4) is correct. Explanation: Elasticity is the responsiveness of demand for the change to the price of the commodity. With the price of the commodity being low, demand increases but since the commodity has less uses the demand becomes inelastic.

43. Option (3) is correct. Explanation: Net Factor Income from Abroad = Factor Income from Abroad – Factor Income to Abroad `200 = Factor Income from Abroad – `40 Factor Income from Abroad = `200 + `40 = `240 crores

44. Option (1) is correct. Explanation: TU = EMU Total utility is the aggregate amount of satisfaction or fulfilment that a consumer receives through the consumption of a specific goods or service.

45. Option (2) is correct. Explanation: Decrease in Government spending will reduce the level of aggregate demand in the economy and will help to correct inflationary pressures in the economy.

46. Option (3) is correct. Explanation: Minimum support price or price floor is fixed by the government in the interest of producers and it refers to the minimum price which is decided by the government so that producers are not exploited and it is fixed above the equilibrium price.

Solutions 47. Option (1) is correct. Explanation: Agricultural Development bank of Pakistan uses the production function approach to measure bank output and costs.

48. Option (2) is correct. Explanation: Economies of scale is larger for branches at a smaller scale of production

55

49. Option (1) is correct. Explanation: When the banks grow in size, they move closer in achieving increasing returns to a factor.

50. Option (3) is correct. Explanation: The increasing returns to scale in larger branches is due to both increase in loans and increase in deposits.



SOLUTIONS OF Question Paper 1. Option (3) is correct. Explanation: Final goods are referred to as those goods that do not require further processing. These goods are produced for the direct consumption either by a consumer for final consumption or by a producer for investment or capital formation.

2. Option (1) is correct. Explanation: Money supply refers to the total stock of money of all types held by the people of a country at a given point of time.

3. Option (3) is correct. Explanation: APC =

C

Y C 0.8 = ` 4,000 C = ` 3,200 crores S

=Y − C = ` 4,000 − ` 3,200 = ` 800 crores

4. Option (1) is correct. Explanation: The word `Economics’ was derived from two Greek words oikou (a house) and nomos (to manage).

5. Option (4) is correct. Explanation: Primary deficit indicates borrowing requirements of the government to meet fiscal deficit net of interest payments.

6. Option (2) is correct. Explanation: Since, the MU curve is a strictly downward sloping graph and is also the slope for TU curve, after reaching the saturation point i.e., MU=0, moving further on TU graph gives us negative value of MU.

7. Option (1) is correct. Explanation: When the price of factors of production like land, labour rises, then it leads to rise in cost of production and hence supply falls. Thus, supply curve shifts to the left.

2

8. Option (3) is correct. Explanation: Depreciation indicates the changes in the value of an asset over a period of time.

9. Option (1) is correct. Explanation: The average propensity to consume measures the percentage of income that is spent rather than save. It defines the amount of consumption in every `1 of income par all level of income.

10. Option (3) is correct. Explanation: Autonomous transactions are independent of the state of BOP Account. For example, if a foreign company is making investments in India with the aim of earning profit, then such a transaction is independent of the country’s BOP situation.

11. Option (3) is correct. Explanation: Under perfect competition, price is same or remains constant. Hence, total revenue increases at a constant rate and hence average revenue or marginal revenue is same or constant and AR or MR curve is parallel to x-axis.

12. Option (1) is correct. Explanation: When price falls with rise in output, i.e., under imperfect competition, when TR reaches its maximum point, then MR reaches zero. And after that TR starts falling and MR becomes negative.

13. Option (3) is correct. Explanation: Supply of the commodity is said to be perfectly elastic when supply curve is parallel to x-axis and when there is increase or decrease in demand for a commodity, then equilibrium quantity changes, i.e., increases or decreases but equilibrium price remains the same.

14. Option (2) is correct. Explanation: A legal tender is a type of money that the court of law need to see as a satisfactory payment of any financial debt. In India, the authentic legal tender of the Reserve Bank of India consists of coins and notes.

Solutions 15. Option (2) is correct. Explanation: Macroeconomics is the study of behaviour of an economy as a whole. National Income is the value of total final goods produced by an economy. Thus, the study of National Income is related to macroeconomics.

16. Option (2) is correct. Explanation: Primary deficit indicates borrowing requirements of the government to meet fiscal deficit net of interest payments.

17. Option (3) is correct. Explanation: Managed floating exchange rate system is the amalgamation of the flexible exchange rate system and the fixed exchange rate system. Under this system, central banks intervene to buy and sell foreign currencies in an attempt to moderate exchange rate movements.

18. Option (4) is correct. Explanation: All other factors except `Number of firms’ is not a determinant of individual supply. All other factors like price of the given commodity, taxation policy, state of technology are the determinants of individual supply.

19. Option (3) is correct. Explanation: In an Oligopoly form of market no single firm can predict its prospective sales with perfection. This is because any given change in the price/output decision by a rival firm would initiate a series of actions, reactions and counter actions by others. Therefore, there is no certain nature and position of demand curve under this form of market for a firm.

20. Option (1) is correct. Explanation: The price elasticity of demand varies directly with the time period. If the time period required to find the substitute commodity is more, the price elasticity will increase and vice versa. This means the elasticity for a shorter time period is always low or it can be even inelastic.

21. Option (3) is correct. Explanation: Central Bank regulates the volume and use of credit by using quantitative and qualitative tools, i.e., monetary policy. Instruments of Monetary Policy are Bank Rate, Repo Rate, Reverse Repo Rate, Cash Reserve Ratio.

57

22. Option (3) is correct. Explanation: Average propensity to save refers to the proportion of income that is saved rather than spent on current goods and services.

23. Option (2) is correct. Explanation: More sophisticated and heavy capital goods that raise the ability of a labourer to produce goods includes heavy machineries. These heavy and sophisticated machineries reduce the time taken by labourer.

24. Option (3) is correct. Explanation: In the third statement, the first part of the statement is positive giving facts, and the second part is normative based on value judgements.

25. Option (1) is correct. Explanation: In the first stage, when total product increases at an increasing rate, then marginal product also rises till the point where it reaches maximum point.

26. Option (3) is correct. Explanation: Under monopoly market there is only one firm which constitutes the industry.

27. Option (1) is correct. Explanation: Indirect tax is a tax on goods and services.

28. Option (1) is correct. Explanation: MPC = 0.6 MPS = 0.4 K =1/MPS = 1/1-MPC = 1/0.4 = 2.5

29. Option (3) is correct. Explanation: When total physical product starts falling, then marginal physical product of a factor becomes negative.

30. Option (3) is correct. Explanation: NNPFC= NNPMP - Indirect Taxes + Subsidies

31. Option (1) is correct.

58

OSWAAL CUET (UG) Sample Question Papers, ECONOMICS

32. Option (3) is correct. Explanation: The elastic demand curve for luxuries is flatter than normal because the coefficient of elasticity is always greater than one. Hence, the reason is wrong as it says, the coefficient of elasticity ranges between 0 and 1. The assertion is correct.

33. Option (3) is correct. Explanation: The functional relationship between output and cost of production is given by cost function. C = F(Q) Because cost is dependent on output. As output changes, cost also changes.

34. Option (4) is correct. Explanation: It embodies the highest amount of skilled and unskilled labor that con be employed within an economy at any given time.

35. Option (3) is correct. Explanation: A customer purchases each commodity until the MU per rupee spent on it is equal to the MU per rupee spent on another good. A customer cannot shift a rupee of expenditure from one commodity to another and raise his utility while this condition is met. This law is the law of equi-marginal utility.

36. Option (3) is correct. Explanation: The economy entered a period of stagnation in 1970 and after Mao zedong death, the communist party leadership chose to shift to market - oriented reforms to revive the economy.

37. Option (2) is correct. Explanation: People’s sense of material well being is referred to as welfare elis is dependent on the availability of products and services for consumption per person. When GDP rises, the flow of good and services increases.

38. Option (3) is correct. Explanation: Fiscal deficit is defined as the amount of borrowing the government has to resort to meet its expenses.

39. Option (2) is correct. Explanation: Depreciation of domestic currency refers to fall in the value of domestic currency in terms of foreign currency caused

by rise in foreign exchange rate in the foreign exchange market.

40. Option (2) is correct. Explanation: Invisible items refer to those items which cannot be seen, felt, touched or measured. For example, services of shipping, banking, insurance, etc.

41. Option (2) is correct. Explanation: As the railways in India is operated solely by the government of India, making only one seller for it, Railways is the example of Monopoly Market.

42. Option (2) is correct. Explanation: The Central Bank undertakes this function of being a monopolist in issuing currency to maintain uniformity of currency across nation and to control money supply process at a centralised level.

43. Option (1) is correct. Explanation: Under monopolistic competition firm is a price maker and due to close substitutes available in the market, the demand curve is more elastic than monopoly but not perfectly elastic demand curve.

44. Option (3) is correct. Explanation: Supply can be referred to as total quantity of a commodity that a producer is willing and able to offer for sale at a given price during a given period of time. Stock refers to total quantity available with the seller to be sold.

45. Option (3) is correct. 46. Option (4) is correct. Explanation: Economic resources are of four kinds: (i) Natural (air, water) (ii) Capital (machines) (iii) Human (labour) (iv) Entrepreneurial (a person who bears the risk and integrates all other resources to produce output.) Hence, we can say that all of the abovementioned resources are kinds of economic resources.

Solutions 47. Option (2) is correct. Explanation: The income of the consumers remains unchanged. This is one of the assumptions required for the law of demand to hold true. Law of demand follows the assumption that the commodity should be a normal good. Therefore, the reason is the correct statement, but the assertion is the wrong statement.

48. Option (2) is correct. Explanation: A leftward shift in the demand curve indicates a decrease in demand because consumers are purchasing fewer products for the same price. Therefore, when the prices of complementary goods increase, demand decreases.

59

49. Option (1) is correct. Explanation: Contraction of demand means movement along a demand curve. In this, the consumer moves to the left or upwards on the same demand curve.

50. Option (3) is correct. Explanation: The price elasticity of demand for a good depends on the nature of the good and the availability of close substitutes of the good. Consider, for example, necessities like food. Such goods are essential for life and the demands for such goods do not change much in response to changes in their prices.



SOLUTIONS OF Question Paper 1. Option (4) is correct. Explanation: Factor Payments are made to the factors of production, thus scholarship is not a factor payment, instead it is a transfer payment.

2. Option (4) is correct. Explanation: Before 1930, there was only one `economics’. Ragnar Frisch coined the words `micro’ and `macro’ in 1933 to denote the two branches of economic theory, namely, microeconomics and macroeconomics.

3. Option (3) is correct. Explanation: Main source of money supply in India is in the form of bank deposit and cash. RBI monitors the money supply in the economy and has the power to print and issue currency.

4. Option (3) is correct. Explanation: It is an inflow of money in Indian economy.

5. Option (3) is correct. Explanation: Features of oligopoly : (a) Few fir or with large market share. (b) High Barrier to entry.

6. Option (3) is correct. Explanation: When AD=AS, all the producers wish to produce during the year is exactly equal to what the buyers wish to spend on the purchase of goods and services during the year, so, there is no excess capacity.

7. Option (3) is correct. Explanation: The supply curve of a good shift to the right when there is fall in price of other goods. In that case, given good becomes more profitable to produce and hence its supply rises.

8. Option (2) is correct. Explanation: The average propensity to consume (APC) measures the percentage of income that is spent rather than saved.

3

9. Option (2) is correct. Explanation: Capital loss is non- recurring in nature. Unforeseen obsolescence of fixed capital assets during production is a capital loss that may happen due to natural calamities, war, unexpected decrease in demand, etc.

10. Option (2) is correct. Explanation: Primary deficit indicates borrowing requirements of the govt. to meet fiscal deficit net of interest payments.

11. Option (3) is correct. Explanation: Foreign exchange refers to exchanging the currency of one country for another at prevailing exchange rate. Forex is the trading of one currency for another.

12. Option (2) is correct. Explanation: The important feature of perfect competition is that firms sell homogeneous products. Homogeneous products which are same in quality, size, colour etc. They are not different from each other and hence they are perfect substitutes of each other.

13. Option (3) is correct. Explanation: When price remains same but supply falls due to factors other than the own price of the commodity. It is called as decrease in supply. It can be due to various reasons like rise in price of factor of production, outdated technique of production, etc.

14. Option (2) is correct. Explanation: When there is increase in TR at a constant rate and rate of TR is denoted by MR. Hence, MR also becomes constant. MR is a horizontal straight line parallel to x-axis.

15. Option (1) is correct. Explanation: M1 includes all forms of assets that are easily exchangeable as payment for goods and services. It consists of coin and currency in circulation and demand deposits, as payment for goods and services.

Solutions 16. Option (3) is correct. Explanation: AR = TR/Units sold = 1,00,000/ 20,000 = 5

17. Option (3) is correct. Explanation: Net Factor Income from Abroad = Factor Income from Abroad – Factor Income to Abroad `200 = Factor Income from Abroad – `40 Factor Income from Abroad = `200 + `40 = `240 crores

18. Option (1) is correct. Explanation: ∆C MPC = ∆Y 8,000 = 10 ,000 = 0.8

19. Option (4) is correct. Explanation: The main objective of microeconomics is to determine the price of a commodity or factors of production whereas, the main aim of macroeconomics is to determine the income and employment level of the economy. It studies the behaviour of aggregates of the economy as a whole.

20. Option (4) is correct. Explanation: Primary deficit refers to the difference between the current year fiscal deficit and interest payment on previous borrowings. It indicates the borrowing requirement of the government excluding interest.

21. Option (3) is correct. Explanation: In the short-run production function, there are two factors, fixed and variable. This law of variable proportion operates in the short-run and not in the long-run.

22. Option (3) is correct. Explanation: Buying of machinery from Japan is an autonomous transaction. Autonomous items refers to those Balance of Payment (BOP) transactions which are undertaken for profit.

23. Option (2) is correct. Explanation: MP3 = TP3 - TP2 = 450 - 350 = 100 units Hence, marginal product of the third unit of input is 100 units.

61

24. Option (2) is correct. Explanation: Fiscal deficit is the condition when the expenditure of the government exceeds its revenue in a year.

25. Option (3) is correct. Explanation: National income is the sum total of factor incomes earned by normal residents of a country in the form of rent, wages, interest and profit in an accounting year.

26. Option (2) is correct. Explanation: With reducing the prices the firm in the monopoly market will be able to sell more as they experience an elastic demand curve.

27. Option (1) is correct. Explanation: Components of Aggregate Demand: (i) Private consumption expenditure (ii) Government consumption expenditure (iii) Investment expenditure (iv) Net exports

28. Option (1) is correct. Explanation: Law of variable proportion states that as more and more units of a variable factor are combined with the factor, then stage must ultimately come when marginal product of the variable factor starts declining.

29. Option (3) is correct. Explanation: An indifference curve is convex to the origin because of diminishing marginal rate of substitution. It is downward sloping because if the quantity of one good is reduced then the quantity of the other good is increased. Therefore, an indifference curve is downward sloping and convex to the origin.

30. Option (3) is correct. Explanation: Non-Tax Revenue is the recurring income earned by the government from sources other than taxes.

31. Option (4) is correct. Explanation: GDP is a flawed measure of human welfare, e.g., if increase in the level of GDP is associated with higher level of income inequality, social welfare may not increase.

32. Option (2) is correct. Explanation: In the perfect competition, there is no restriction on entry and exit of firms. Firms enter the market by getting attracted towards profit and they leave the market when the existing firms bear losses.

62

OSWAAL CUET (UG) Sample Question Papers, ECONOMICS Hence, in the long run firms earn normal profits. Since there are large number of firms which form a industry. Hence, price is decided by the market forces of demand and supply in the industry and firm has to accept that price. Hence, firm is a price taker and not a price maker.

33. Option (2) is correct. Explanation: Cost suffered by unrelated third party due to result of an economic transaction is known as negative externalities. Pollution created by factories/vehicles has negative effect on the society.

34. Option (1) is correct. Explanation: Money Supply is the total stock of money circulating in the economy at a particular point of time including currency and other liquid instruments. It includes all types of deposits in bank including demand deposits time deposit.

35. Option (4) is correct. Explanation: Toothpaste is an example of Monopolistic Competition Market as there are a lot of sellers that sell it with a little differentiation in their product.

36. Option (3) is correct. Explanation: It is the rate at which one currency can be converted into another currency.

37. Option (1) is correct. Explanation: India had favourable balance of trade in 1972-73 and 1976-77.

38. Option (2) is correct. Explanation: When the price of a commodity decreases new buyers enter the market and start purchasing it this is because they were unable to purchase it when the prices were high but now they can afford it thus, as price falls. The demand rises and the demand curve becomes downward sloping.

39. Option (3) is correct. Explanation: Policy adopted by the Central Bank of an economy in the direction of credit control or money supply is known as Monetary Policy.

40. Option (2) is correct. Explanation: The utility approach assumes that the prices of commodities and consumer’s income are given and remain constant and utility is a measurable concept. Therefore, the only assumption true among the given is (2).

41. Option (2) is correct. Explanation: Non-monetary exchanges refers to goods and services that are produced but not exchanged for cash. The value of these exchanges is often difficult to estimate and therefore excluded from the national income estimates.

42. Option (4) is correct. Explanation: Coal is classified as a nonrenewable energy source because it takes millions of years to form.

43. Option (2) is correct. Explanation: According to Samuelson, an economic organization’s three fundamental and interconnected problems: what, how, and for whom, are grouped under allocation of resources. The term “allocation of resources” refers to how much of each resource is committed to a specific task.

44. Option (4) is correct. Explanation: Full employment equilibrium refers to the situation when people who are able and willing to work at the prevailing wage rate get employment.

45. Option (1) is correct. Explanation: Utility is a cardinal concept as it can be measured.

46. Option (2) is correct. Explanation: Commercial Banks deals with public directly by accepting and lending deposits.

47. Option (1) is correct. Explanation: With the increase in the price the demand for coke will respond with the equal intensity or more, making the demand is elastic.

48. Option (2) is correct. Explanation: Because as more and more customer become health conscious and move away from high calorie sugary drink.

49. Option (2) is correct. Explanation: Demand refers to the quantity of a commodity that a consumer is willing and able to buy at each possible price during a given period of time.

50. Option (1) is correct. 

SOLUTIONS OF Question Paper 1. Option (4) is correct.

10. Option (1) is correct.

Explanation: Capital is measured at a particular point of time. (When government spend more than its total income.)

2. Option (1) is correct. Explanation: To do one economics assignment, it takes 30 minutes. To do two economics assignments, it takes 1 hour which is the same time as needed to do one finance assignment. Therefore, the opportunity cost of doing two economics assignments is one finance assignment.

3. Option (4) is correct. Explanation: Aggregate Demand refers to the value of final goods and services which all sectors of an economy are planning to buy during a year.

4. Option (2) is correct. Explanation: Demand deposits include saving account deposits and current account deposits because these are very liquid and can be used immediately to purchase goods and services.

5. Option (4) is correct. Explanation: Income of the consumer affects the individual demand since more will the person earn more he will be able to afford, hence an increase in demand for normal goods.

6. Option (2) is correct. Explanation: Fiscal Deficit refers to the excess of total expenditure over total receipts excluding borrowings. (When government spend more than its total income.)

7. Option (1) is correct. Explanation: Money flows payments for the services.

4

refer

to

the

8. Option (4) is correct. Explanation: It is an outflow of money from Indian economy.

9. Option (2) is correct. Explanation: Marginal Propensity to Consume is the ratio of change in consumption due to change in income.

Explanation: (a) Increase in demand for a goods Leftward shift in the demand curve (b) Decrease in demand - Rightward shift in the demand curve (c) Ed = ∞ - Perfectly Inelastic Demand (d) Downward Sloping- Normal Demand Curve

11. Option (1) is correct. Explanation: When the actual price of a commodity is less than the equilibrium price, then demand starts rising. Due to excess demand, consumers are willing to buy but sellers are not willing to sell. Hence, price will rise.

12. Option (3) is correct. Explanation: Fixed cost curve is parallel to x-axis because it remains constant at all levels of output. Hence, fixed cost is the cost incurred on fixed factors of production like land and machinery.

13. Option (2) is correct. Explanation: According to United Nation, “Economic territory is the geographical territory administered by a government within which persons, goods and capital circulate freely.” Since, Foreign Embassies can't be administered by Indian Government, these are a part of India's geographical territory.

14. Option (3) is correct. Explanation : (a) Tea Industry - Monopolistic Competition (b) Ship Building industry - Monopsony (c) Oil Producing Countries - Oligopoly (d) Food Market - Perfect Competition

15. Option (2) is correct. Explanation: ΔY = ΔC MPC = ΔC/ΔY MPC = ΔC/ΔC = 1 MPS = 1– MPC = 0 K = 1/MPS = 1/0 = ∞

64

OSWAAL CUET (UG) Sample Question Papers, ECONOMICS

16. Option (3) is correct.

25. Option (4) is correct.

Explanation: Managed floating exchange rate system is the amalgamation of the flexible exchange rate system and the fixed exchange rate system. Under this system, central bank intervenes to buy and sell foreign currencies in an attempt to moderate exchange rate movements.

26. Option (2) is correct.

17. Option (2) is correct. Explanation: RBI is the top monetary authority in the country who print currency and regulates money supply through its monetary policy.

18. Option (2) is correct. Explanation: Supply curve is upward sloping curve from left to right. The slope of supply curve is measured by: ΔY/ΔX, where in on Y-axis price is measured and on x-axis quantity supplied is measured. Hence, slope of supply curve = ΔP/ΔQ.

Explanation: Due to installation of a machine with latest technology, there is fall in cost of production which has led to increase in supply of the commodity and hence supply curve shifts to the right.

27. Option (2) is correct. 28. Option (1) is correct. Explanation: For the Indifference Curve Approach the consumers are assumed to be rational, who aim at maximizing their benefits.

29. Option (4) is correct.

19. Option (4) is correct. Explanation: In case of direct tax, the burden of tax and the liability to pay it falls on the same person.

20. Option (3) is correct. Explanation: When MP is more than AP, then AP curve rises. When MP=AP, then AP is at its maximum point. And when MP is less than AP, AP falls.

Explanation: Indifference curves are graphs that represent various combinations of two commodities that an individual considers equally valuable.

30. Option (1) is correct. Explanation: Stock refers to the total quantity of a commodity available with the seller at any given time.

31. Option (4) is correct.

21. Option (4) is correct. Explanation: Aggregate demand can increased by : (a) Decreasing bank rate. (b) Purchasing government securities.

Explanation: MU curve is the slope of TU curve. Since the slope of TU curve at that point is zero, therefore, MU is zero and that point is known as the saturation point. Units of the good are consumed till the saturation point.

be

22. Option (4) is correct. Explanation: Among the following, “wages to employees” is the variable cost for a firm. All other options are the examples of fixed cost because they do not change with the change in the level of output.

23. Option (1) is correct. Explanation: In case of monopoly there is only one seller implying that the price structure does not depend on any other firm in the market.

24. Option (4) is correct. Explanation: In an Oligopoly form of market no single firm can predict its prospective sales with perfection. This is because any given change in the price/output decision by a rival firm would initiate a series of actions, reactions and counter actions by others. Therefore, there is no certain nature and position of demand curve under this form of market for a firm.

Explanation: Export of goods and services from India to US would mean inflow of foreign exchange to India.

32. Option (3) is correct. Explanation: Legal tender money is that form of money that has a legal sanction by the government behind it. Cheque able demand deposits are not legal tenders because a person can legally refuse to accept payment through cheque able demand deposits.

33. Option (4) is correct. Explanation: Normative economics focuses on the value of economic fairness or what the economy should be or based ‘to be’. It is subjective and value based ought.

34. Option (4) is correct. Explanation: It refers to the excess of total expenditure over total receipts excluding borrowings.

35. Option (2) is correct.

Solutions Explanation: Monopoly is a market situation in which there is a single seller, there are no close substitutes for commodity it produces and there are barriers to entry of new firms. Monopolist himself fixes the price. Accordingly, a monopolist faces a downward sloping demand curve which shows that more can be sold only at a lower price.

36 Option (4) is correct. Explanation: Characteristics of Monopolistic Competition : (i) Large number of firms, (ii) Product differentiation, (iii) Free entry and exit of firms, (iv) Selling Cost, (v) Non-price competition, (vi) Sales techniques, (vii) Absence of collective action, (viii) Consumer’s attachment, (ix) Price policy of a firm, (x) Lack of perfect knowledge.

37. Option (1) is correct. Explanation: There are three phases in returns to a factor. In the first phase, i.e., increasing returns to a factor where in TP increases at an increasing rate and MP also increases.

38. Option (3) is correct. Explanation: By controlling credit, the central bank can exercise an effective control over economic activities and mobilise it in the desired direction. Central Bank regulates the volume and use of credit by using quantitative and qualitative tools.

39. Option (2) is correct. Explanation: A debit to capital a/c means the business doesn’t owe so much to its owners (i.e. reduces the business capital).

40. Option (3) is correct. Explanation: Average product can be defined as total product divided by number of units of variable factor. Since, total product can only be positive and cannot take negative values, hence, AP can have only positive values.

41. Option (4) is correct. Explanation: In macroeconomics, the money supply (or money stock) refers to the total volume of money held by the public at a particular point in time in an economy.

65

42. Option (2) is correct. Explanation: Demand refers to effective demand, which is the demand for a commodity should be backed by a willingness to spend and purchasing power.

43. Option (3) is correct. Explanation: (a) Relatively Inelastic Demand - ed < 1 (b) Relatively Elastic Demand - ed > 1 (c) Perfectly Inelastic Demand - ed = 0 (d) Perfectly Elastic Demand- ed = infinity

44. Option (1) is correct. Explanation: If aggregate demand is greater than aggregate supply, flow of goods and services in the economy tends to be less than their demand, which means the existing stock of producers would be sold out.

45. Option (3) is correct. Explanation: Uniformity of Taxes: Benefit of GST At the State Level: SGST One Point Single Tax: Objective of GST Brought an end to black money: Effect of Demonetisation.

46. Option (3) is correct. Explanation: Disinvestment is a capital receipt as it reduces assets.

47. Option (2) is correct. Explanation: Positive externalities refer to positive impact of an economic activity on the others without involving any price.

48. Option (2) is correct. Explanation: Welfare means material wellbeing of the people as it is the state of having enough housing, clothing, food, and possessions, etc. to live comfortably.

49. Option (1) is correct. Explanation: Real GDP and Welfare are related with each other as rise in the real GDP leads to rise in the welfare.

50. Option (1) is correct. Explanation: According to give case, NCAP is implemented in order to ensure clear air in an economy.



SOLUTIONS OF Question Paper 1. Option (1) is correct. Explanation: M3 = M1 + Time deposits of all commercial banks and co-operative banks (excluding interbank time deposits).

2. Option (2) is correct. Explanation: Real flow refers to the flow of factor services and goods and services among different sectors of the economy.

3. Option (1) is correct. Explanation: Law of variable proportion states that as more and more units of a variable factor are combined with the factor, then stage must ultimately come when marginal product of the variable factor starts declining.

4. Option (4) is correct. Explanation: ∆C MPC = ∆Y 600 = 1,000 = 0.6

5. Option (4) is correct. Explanation: Deficient demand refers to the situation when aggregate demand (AD) is short of aggregate supply (AS) corresponding to full employment in an economy.

6. Option (1) is correct. Explanation: In positive economics, human decisions can be studied as facts and can be backed by actual data. In 1, 2 and 4, the decisions taken by the policy makers can be verified using some statistics and can be proven. On the contrary, normative economics gives perspectives of individuals without proper authentication. Therefore, the third statement falls under the normative category as it just gives a piece of advice.

7. Option (3) is correct. Explanation: Variable cost is the cost which is incurred on variable factors of production like

5

labour, raw material etc. variable cost changes with the change in the level of output

8. Option (3) is correct. Explanation: When demand of a good changes due to change in its own price it is represented at different points on the same demand curve. It is called, movement along demand curve. It shows `Extension and Contraction’ of demand. Demand curve does not change in both these conditions. (i) Extension of Demand : Other things being equal, when demand of a good increases due to decline in price of that good, then it is called Extension of Demand. (ii) Contraction of Demand : Other things being equal, when demand of a good decreases due to increase in price of that good, it is called Contraction of Demand.

9. Option (1) is correct. Explanation: Loan advanced by World Bank is a capital receipt as it raises liability or reduces assets.

10. Option (3) is correct. Explanation: Autonomous investment refers to that portion of total investment which is independent of the change in the level of income, interest rate and rate of profit. Investment in public utility services are considered autonomous investment because these type of investments made by the government does not rely on the decisional profit or loss.

11. Option (3) is correct. Explanation: Flexible exchange rate is determined by the forces of demand and supply i.e. where demand equals supply eliminating the undervaluation or over valuation of currencies.

12. Option (4) is correct. Explanation: Accommodating transactions are those that are undertaken as a consequence of the autonomous transactions.

Solutions 13. Option (2) is correct. Explanation: When total product starts falling, then marginal product becomes negative because total product is summation of marginal product. TP after reaching its maximum point starts falling

14. Option (3) is correct. Explanation: Under perfect competition, in the long run, firm earns normal profit because of freedom of entry and exit of firms. In this, when firm enters the market, its supply rises and price falls and hence abnormal profits reaches to normal profits and when firms leave the market, then, abnormal loss becomes equal to normal profits. Hence, there are neither abnormal profits nor abnormal losses in the long run.

15. Option (2) is correct. Explanation: Budget line displays all the combinations of goods a consumer can buy with the given income and price of a commodity. Assumptions of budget line are income and prices of consumer is given and remains unchanged.

16. Option (1) is correct. Explanation: Monopoly is a market situation in which there is a single seller, there are no close substitutes for commodity it produces and there are barriers to entry of new firms. Monopolist himself fixes the price. Accordingly, a monopolist faces a downward sloping demand curve which shows that more can be sold only at a lower price.

17. Option (1) is correct. Explanation: Revenue expenditure do not create any liability for the government. For example, taxes received by the government, unlike borrowings, do not create any liabilities for it.

18. Option (1) is correct. Explanation: Under perfect competition, the price is determined by the industry. It is due to the fact that there are large number of buyers and sellers of homogeneous products under perfect competition. No single seller by changing his supply can influence the price. A monopolist is the only seller and himself determines price of his product. He is a price maker. There is no challenge to his price decisions as there are no competitive firms in

67

the market and there are no close substitutes of his product. Barriers to the entry of new firms further strengthens his position as a price maker.

19. Option (2) is correct. Explanation: The main reason why the number of firms is small is that there are barriers which prevent entry of the firms into industry. Patents, large capital requirement, control over the crucial raw materials, etc. prevent new firms from entering the industry. Only those who are able to cross these barriers enter in the market.

20 Option (1) is correct. Explanation: Equilibrium output refers to a situation when in an economy: AD=AS So that, all the producers who wish to produce during the year is exactly equal to what the buyers wish to spend on the purchase of goods and services during the year.

21. Option (2) is correct. Explanation: Net National Product (NNP) at Market Price = NNP at Factor Cost + Net Indirect Taxes

22. Option (3) is correct. Explanation: The guiding principle of the problem `how to produce?’ is to adopt such technique of production which has least cost to produce per unit of the commodity. At macro level the most efficient technique is the one which uses least quantity of scarce resources. Hence, producers must always produce efficiently by using the most efficient technology. Thus, every economy must choose the most efficient technique of producing a commodity.

23. Option (3) is correct. Explanation: Fixed factors of production like land, machinery etc. and money spent on these is a fixed cost because it does not change with the level of output. Variable factors of production like casual labour, raw material etc. Money spent on these is a variable cost because it changes with the change in the level of output.

68

OSWAAL CUET (UG) Sample Question Papers, ECONOMICS

24. Option (1) is correct. Explanation: When a customer optimizes his happiness given his income and commodity prices, he is said to be in equilibrium. This means that the marginal utility of the good equals the price of the product.

25. Option (2) is correct. Explanation: In case of deficient demand, there is a need to liberalize credit. It can be done by reducing bank rate so that the commercial banks also reduce their lending rate, thereby increasing the availability of credit in the economy.

26. Option (2) is correct. Explanation: Supply curve is horizontal line parallel to x-axis it refers to goods that have a price elasticity of supply value equal to infinity.

27. Option (3) is correct. Explanation: Non-Tax Revenue is the recurring income earned by the government from sources other than taxes.

28. Option (2) is correct. Explanation: When supply is perfectly inelastic, and demand for a commodity increases or decreases which causes a change in equilibrium price of the commodity but equilibrium quantity remains constant.

29. Option (2) is correct. Explanation: Shift in the demand Curve Increase/Decrease in the demand with respect to other factors other than price. Movement along the demand curve - Increase/ Decrease in Price

30. Option (4) is correct. Explanation: K=1/1 – MPC K=1/0 K=∞

31. Option (1) is correct. Explanation: Current a/c balance of payment is a record of a country’s international transaction with the rest of the world. It includes all the transactions that involve economic values and occur between resident and non-resident entities.

32. Option (1) is correct. Explanation: Goods used in the production of final goods are known as intermediated goods.

33. Option (3) is correct. Explanation: Marginal Utility : The addition to total utility on consuming an additional unit of a commodity. Cardinal measure of utility : It is that measurement of utility, which is measured in terms of units. Total Utility : It is the sum total of utility derived from the consumption of all units of a commodity. Utility : The power or capacity of a commodity to satisfy human wants.

34. Option (2) is correct. Explanation: Accommodating items are termed as ‘below the line items’, as these items of BoP are not determined by considerations of profit but to restore identity of BoP i.e. to balance the BoP.

35. Option (4) is correct. Explanation: Microeconomics investigates the behaviour of particular economic units such as commodity price determination, consumer, producer, or company behaviour. Price theory is another name for microeconomics. Therefore, individual income is a part of microeconomics.

36. Option (1) is correct. Explanation: The market demand curve in the summation of all the individual demand curve in a market.

37. Option (3) is correct. Explanation: The Central Bank plays the role of the sole note issuing authority in an economy. The Central Bank undertakes this function of being a monopolist in issuing currency to maintain uniformity of currency across nation and to control money supply process at a centralised level. This function also builds faith in the currency system of the economy.

38. Option (4) is correct.

Solutions Explanation: As there are a large number of sellers selling differentiated products in the monopolistic competition market, there are a variety of options for the consumers.

39. Option (1) is correct. Explanation: Demand has an inverse relation with the price of the commodity, hence whenever the price increases demand decreases and vice versa.

40. Option (1) is correct. Explanation: Recovery of loan is a capital receipt because it reduces assets of the government.

41. Option (1) is correct. Explanation: Collusive Oligopoly is one in which, the firms cooperate with each other in deciding price and output. Whereas, Noncollusive Oligopoly is one in which the firms compete with each other.

42. Option (2) is correct. Explanation: K = 1/MPS K = 1/1-MPC K = 1/1-1 K = 1/0 K=∞

43. Option (4) is correct. Explanation: M1 and M2 are known as narrow money. M1 = Currency (notes and coins) with the public + Demand deposits + Other deposits held with the Reserve Bank of India. M2 = M1 + Post Office Saving Deposits

69

Explanation: Increase in demand raises and decrease in demand lowers the equilibrium price. Also, equilibrium quantity will increase when demand increases and will decrease when demand decreases.

45. Option (2) is correct. Explanation: Goods and Services Tax is an indirect tax used in India on the supply of goods and services.

46. Option (2) is correct. Explanation: When change in price produces no change in demand, then such a demand is called perfectly inelastic demand. In this situation, demand curve is a straight line parallel to the Y axis.

47. Option (1) is correct. Explanation: Real GDP is GDP evaluated at the market prices of some base year.

48. Option (2) is correct. Explanation: The country's real gross domestic product is likely to expand due to a faster economic recovery following the relaxation of the lock down.

48. Option (4) is correct. 49. Option (1) is correct. Explanation: According to first advance estimates of national income released by the NSO, the country GDP is estimated to contract by a record 77% during the current financial year.

50. Option (1) is correct.

44. Option (1) is correct.



SOLUTIONS OF Question Paper 1. Option (1) is correct. Explanation: Wealth is the accumulated value of economic goods or past savings measured at a given point of time.

2. Option (3) is correct. Explanation: Borrowings is a capital receipt as it creates a liability.

3. Option (2) is correct. Explanation: The opportunity cost of an activity is what you have given up to do in the time when you have done some other activity. Therefore, while you chose to study you had given up the idea of watching a movie.

4. Option (1) is correct. Explanation: C = f (Y)

5. Option (2) is correct. Explanation: In case of perfect competition, AR curve is horizontal straight line. Hence, TR curve rises at a constant rate, and, thus, AR and MR curve is horizontal straight line.

6. Option (1) is correct. Explanation: In the perfect competition, there are very large number of buyers and sellers. They are so large that the share of each buyer or seller in the market is insignificant. Hence, they cannot influence the price in the market.

7. Option (1) is correct. Explanation: Law of diminishing marginal utility states that as more and more units of a commodity are consumed, marginal utility derived from every additional unit must decline. The law of equi-marginal utility states that the consumer will distribute his money income between the goods in such a way that the utility derived from the last rupee spent on each good is equal.

8. Option (2) is correct. Explanation: Supply curve is upward sloping which shows that as the price of the commodity rises, quantity supplied also rises and as the price falls, quantity supplied also falls.

6

9. Option (3) is correct. Explanation: When a firm is able to sell any quantity of a good at a given price. This happens under perfect competition when price of a good remains constant. Then, firm’s marginal revenue is equal to average revenue.

10. Option (4) is correct. Explanation: When aggregate demand is equal to aggregate supply at less than full employment, it is a situation of under employment equilibrium.

11. Option (3) is correct. Explanation: When decrease in demand is less than increase in supply, then there is fall in equilibrium price but rise in equilibrium quantity.

12. Option (1) is correct. Explanation: Price floor by the government creates excess supply. This excess supply creates buffer stock. Since the producers are not able to sell all they want to sell, they sell illegally below the minimum price.

13. Option (3) is correct. Explanation: When supply is perfectly inelastic, then supply curve is a vertical straight line parallel to Y-axis.

14. Option (3) is correct. Explanation: Net Factor Income from Abroad = Factor Income from Abroad – Factor Income to Abroad `200 = Factor Income from Abroad – `40 Factor Income from Abroad = `200 + `40 = `240 crores

15. Option (1) is correct. Explanation: Following are some assumptions of the law of diminishing marginal utility: (i) It is assumed that utility can be measured and a consumer can express his satisfaction in quantitative terms such as 1, 2, 3... etc. (ii) Consumption of reasonable quantity. (iii) Continuous consumption. (iv) Monetary measurement of utility.

Solutions (v) No change in quantity. (vi) Rational consumer. (vii) MU of money remains constant. (viii) Independent utilities. (ix) Fixed income and price of the goods. (x) Commodites should be Nomogencous in nature.

16. Option (2) is correct. Explanation: Es Percentage change in quantity 60 supplied/Percentage change in price = 50 = 1.2

17. Option (1) is correct. Explanation: Sale of machinery to abroad is a part of Current accounts. Current account records imports and exports of goods and services and unilateral transfers. Sale of machinery to abroad leads to inflow of foreign currency and receipt from exports is shown on the positive side (credit items).

18. Option (2) is correct. Explanation: Fiscal deficit is defined as excess of total budget expenditure over total budget receipts excluding borrowings during a fiscal year

19. Option (4) is correct. Explanation: Excess demand arises when demand for exports increases due to comparatively lower prices of domestic goods or due to decrease in the exchange rate for domestic currency.

20. Option (2) is correct. Explanation: Balance of Trade = – ` 5,000 crores Value of Imports = ` 9,000 crores Balance of Trade (Deficit) = Value of Exports – Imports Value of Exports = Balance of Trade (Deficit) + Imports = – ` 5,000 crores + ` 9,000 crores = ` 4,000 crores

21. Option (2) is correct. Explanation: An Indifference Curve will ordinarily be convex to the origin. This is because of Diminishing Marginal Rate of Substitution.

22. Option (1) is correct. Explanation: Income tax is a forced transfer as tax payer cannot avoid it.

23. Option (1) is correct. Explanation: When the goods can be used instead of the other they are termed as substitutes. That is the increase in the price of one will lead to a increase in the demand of another. For eg : Tea and Coffee.

71

24. Option (3) is correct. Explanation: Total product can be defined as total output produced from a given number of inputs. Total product is equal to average product multiplied by number of units of variable factor.

25. Option (1) is correct. Explanation: The Y-axis of the demand curve represents the price of the commodity, whenever there is a decrease in demand the curve shifts and the quantity demanded is decreased.

26. Option (1) is correct. Explanation: Price Elasticity of Demand is defined as the measurement of percentage in quantity demanded in response to a given percentage change in own price of the commodity.

27. Option (3) is correct. Explanation: The paradox states that an increase in autonomous saving leads to a decrease in aggregate demand and thus a decrease in gross output which will in turn lower total saving.

28. Option (1) is correct. Explanation: The export of goods, services and unilateral receipts is entered on the credit side of the Current Account. Therefore, being categorised as such, the export of weapons to Vietnam will be credited to the current account as well.

29. Option (1) is correct. Explanation: With a rise in real national income, welfare of the people rises as people have more money to spend on the needs and wants.

30. Option (4) is correct. Explanation: Lionel Robbins defines economics as a science of scarcity. Prof. Robbins in his book Nature and Significance of Economic Science states, “Economics is the science which studies human behaviour as a relationship between ends and scarce means which have alternative uses”.

31. Option (1) is correct. Explanation: Fiscal Deficit refers to the excess of total expenditure over total receipts excluding borrowings. It indicates borrowing requirements of the government.

32. Option (1) is correct. Explanation: Capital goods includes those fixed or tangible assets that are purchased by producers or businesses to produce goods and services.

72

OSWAAL CUET (UG) Sample Question Papers, ECONOMICS

33. Option (3) is correct. Explanation: Errors and Omissions constitute the third element in the BoP (apart from the current and capital accounts) which is the ‘balancing item’ reflecting our inability to record all international transactions accurately.

34. Option (1) is correct. Explanation: When AD=AS, all the producers who wish to produce during the year is exactly equal to what the buyers wish to spend on the purchase of goods and services during the year.

35. Option (1) is correct. Explanation: Current account is that account of BoP, which records exports and imports of visible and invisible items and unilateral transfers. A major part of transactions in foreign trade is in the form of export and import of goods (visible items). Payment for import of goods is written on the negative side (debit items) and receipt from exports is shown on the positive side (credit items). Balance of these visible exports and imports is known as balance of trade (or trade balance). Export of cation textile leads to inflow of foreign exchange thus, recorded as credit item.

36. Option (2) is correct. Explanation: When the producer starts the business in the building owned by him then instead of hiring building from outside he is renting out his own land. Hence, imputed rent of own building is the implicit cost.

37. Option (3) is correct. Explanation: (a) Perfectly Elastic Demand (Ed = ∞) : When percentage change in quantity demanded is infinite with a slight rise in the price, then demand for such a commodity is said to be perfectly elastic. In such a situation, demand curve is parallel to X-axis. (b) Unitary Elastic Demand (Ed = 1) : When percentage change in quantity demanded is equal to percentage change in price, then demand for such a commodity is said to be unitary elastic. Shape of demand curve is rectangular hyperbola and elasticity at every point on this curve is unity. (c) Relatively Elastic Demand (Ed > 1) : When percentage change in price of a commodity causes greater percentage change in quantity demanded then demand is said to be highly elastic.

38. Option (3) is correct. Explanation: Externalities refer to positive and negative impact of an economic activity on the others without involving any price or penalty.

39. Option (1) is correct. Explanation: Price elasticity of demand is bss than one, under monopoly market.

40. Option (1) is correct. Explanation: Inflationary gap measures the difference between the current level of real GDP and the GDP that would exist if an economy was operating at full employment.

41. Option (2) is correct. Explanation: In the second phase, total product increases at a decreasing rate and then reaches maximum. When TP is maximum, MP becomes zero.

42. Option (3) is correct. Explanation: Import Substitution industrialization (ISI) is a trade and economic policy that advocates replacing foreign imports with domestic production to protect domestic industries. Thus, there was no requirement of foreign aid.

43. Option (4) is correct. Explanation: Purchase of shares is an investment for the government and increases assets.

44. Option (3) is correct. Explanation: All resources that are available to the people at any point in time for satisfying their wants are scarce and limited.

45. Option (1) is correct. Explanation: Social welfare is an aggregate of the utilities or satisfaction of all the individuals in the society.

46. Option (3) is correct. Explanation: Shift in the Demand Curve is the situation when the demand changes with the change in other factors affecting the demand other than price.

47. Option (1) is correct. Explanation: The Central Bank plays the role of the sole note issuing authority in the economy.

48. Option (4) is correct. 49. Option (1) is correct. Explanation: As per the above paragraph, Narendra Modi announced the demonetisation of existing notes of `500 and `1,000.

50. Option (1) is correct. Explanation: The one rupee note and coins are issued by ministry of finance and it bears the signature of Finance Secretary.



SOLUTIONS OF Question Paper 1. Option (4) is correct. Explanation: Microeconomics is the study of the behaviour of individuals and firms in making decisions based on the allocation of scarce resources.

2. Option (2) is correct. Explanation: Co-operative marketing is a measure to ensure a fair price to farmers. Member farmers sell their surplus to the cooperative society which substitutes collective bargaining in place of individual bargaining. It links rural credit farming marketing processes to the best advantage of the farmers.

3. Option (1) is correct. Explanation: Balance of Trade = – `300 crores Value of exports = `500 crores Balance of trade (Deficit) = Value of Exports – Imports Imports = Exports – Balance of trade (deficit) = `500 crores – (–`300 crores) = `800 crores

4. Option (3) is correct. Explanation: Marginal propensity to save (MPS) is an economic measure of how savings change, given a change in income.

5. Option (4) is correct. Explanation: In case of direct tax, the burden of tax and the liability to pay it falls on the same person. The liability to pay the tax cannot be shift on other person.

6. Option (3) is correct. Explanation: Cross - Price Effect refers to effects of a change in price of commodity-X on demand for commodity-Y when X and Y are related goods.

7. Option (1) is correct. Explanation: The Law of Demand states that when the price of the commodity increases the quantity demanded of the commodity decreases and vice versa. Hence it shows the inverse relationship between the price and quantity.

7

8. Option (1) is correct. Explanation: Production can be defined as transforming inputs into output for selling their output. Production adds utility to the product which is ultimately consumed by the consumers.

9. Option (1) is correct. Explanation: Features of monopoly market : (a) Single seller of the product (b) Entry restrictions (c) No close substitute (d) Price makes.

10. Option (2) is correct. Explanation: Fiscal policy is the use of government spending and taxation to influence the economy.

11. Option (1) is correct. Explanation: Indifference Curve is Downward Sloping, Marginal Utility and total utility are inverted U - shaped

12. Option (1) is correct. Explanation: M3 and M4 are known as broad money. M3 = M1 + Time deposits of all commercial banks and co-operative banks (excluding interbank time deposits). M4 = M3 + Total deposits with the Post Office Saving Organisation (excluding National Saving Certificate). These measures of money supply are called broad money as they includes less liquid assets.

13. Option (4) is correct. Explanation: Oil Cartels are oligopoly; ISRO is Monopsony; DRDO is also Monopsony

14. Option (1) is correct. Explanation: In the perfect competition, there are very large number of buyers and sellers. They are so large that the share of each buyer or seller in the market is insignificant. Hence, they cannot influence the price in the market.

74

OSWAAL CUET (UG) Sample Question Papers, ECONOMICS

15. Option (3) is correct. Explanation: In case of unitary elastic supply, percentage change in quantity supplied is equal to percentage change in price. Hence, elasticity of supply is equal to one.

16. Option (2) is correct. Explanation: Private enterprises always desire to allocate resources to those areas of production where profits are high. However, it is possible that such areas of production (like production of alcohol) may not promote social welfare. Production of goods which are injurious to health (like cigarettes and alcohol) is discouraged through heavy taxation. On the other hand, production of “socially useful goods” (like electricity, ‘Khadi’) is encouraged through subsidies. So, finally government has to reallocate resources in accordance with social and economic considerations in case the free market fails to do or does so inefficiently.

17. Option (4) is correct. Explanation: When aggregate demand is equal to aggregate supply at less than full employment, it is a situation of under employment equilibrium.

18. Option (4) is correct. Explanation: Factors Determining the Elasticity of Demand (i) Objective Factors : (a) Nature of commodity, (b) Existence of substitutes, (c) Alternative uses of a commodity, (d) Postponement of consumption, and (e) Joint demand. (ii) Subjective Factors : (a) Habits of consumers, (b) Change in income of consumers, (c) Standard of living of people, (d) Share in total expenditure, and (e) Class of buyers. (iii) Social Factors : (a) Distribution of National Income, and (b) Rationing System. (iv) Price Factors : (a) General price level, and (b) Effect of time element.

19. Option (3) is correct. Explanation: As the supply curve has a negative range meaning it is steep thus it is relatively inelastic or less elastic.

20. Option (3) is correct. Explanation: Fiscal deficit is defined as excess of total expenditure over total receipts (revenue and capital receipts) excluding borrowing. Fiscal Deficit = Total Budget Expenditure – Total Budget Receipts (Net of borrowing). Primary deficit refers to the difference between fiscal deficit of the current year and interest payments on the previous borrowings. Primary Deficit = Fiscal Deficit – Interest Payments.

21. Option (2) is correct. Explanation: Price ceiling is imposed by the government at a level lower than the equilibrium price. When the equilibrium price is too high for the consumers to afford basic goods like education, agricultural goods, then government fixes the maximum price in the interest of consumers.

22. Option (1) is correct. Explanation: A major part of transactions in foreign trade is in the form of export and import of goods (visible items). Payment for import of goods is written on the negative side (debit items) and receipt from exports is shown on the positive side (credit items). Balance of these visible exports and imports is known as balance of trade (or trade balance).

23. Option (4) is correct. Explanation: Situation for change in budget set : (a) When the level of income changes (b) When price of one good changes (c) When price of both the good change

24. Option (2) is correct. Explanation:            

M=

1 MPS

M=

1 1 − MPC

Multiplier is a amount of new income that is generated from an addition of extra income.

25. Option (4) is correct. Explanation: Purchase of shares is an investment for the government and increases assets.

Solutions 26. Option (1) is correct. Explanation: Since resources are scarce, an inefficient technique of production, which would lead to wastage and high cost, cannot be applied. A technique of production which would maximise output or minimise cost should be used. We generally consider two types of techniques of production: labourintensive and capital-intensive techniques. In labour-intensive technique, more labour and less capital is used. In capital-intensive technique, more capital and less labour is used.

27. Option (4) is correct. Explanation: It is a state in an economy where unemployment is persistently higher than usual.

33. Option (1) is correct. Explanation: Monthly per capita consumption expenditure of `972 in rural areas and `1407 in urban areas is recommended as the poverty line at the all India level.

34. Option (3) is correct. Explanation: M4 is the widest measure of money supply that the RBI uses. It includes all the aspects of M3 and also includes the savings of the post office banks of the country. It is the least liquid measure of all of them. M4 = M3 + Post office savings

35. Option (1) is correct. Explanation: Collusive Oligopoly : It is that form of oligopoly in which all the firms determine price and quantity of output on the basis of co-operative behaviour. Non-collusive Oligopoly : It is that form of oligopoly in which all the firms determine the price and quantity of output according to the action and reaction of the firms. Perfect Oligopoly : If firms produce homogeneous product then it is called Perfect Oligopoly. Imperfect Oligopoly : If firms produce heterogeneous product it is called Imperfect Oligopoly.

28. Option (1) is correct. Explanation: The export of goods, services and unilateral receipts is entered on the credit side of the Current Account. Therefore, being categorised as such, the export of weapons to Vietnam will be credited to the current account as well.

29. Option (4) is correct. Explanation: The market supply of a commodity is affected by the different factors like state of technology, number of firms, government policy etc.

30. Option (4) is correct. Explanation: Tea and Coffee are used instead of each other. So is rice and wheat and coke and pepsi. Thus all are substitutes.

31. Option (1) is correct. Explanation: Goods used in the production of final goods are known as intermediate goods.

32. Option (4) is correct. Explanation: There are certain limitations of GDP as an index of welfare: (1) GDP does not take into account changes in inequalities in the distribution of income. (2)  GDP does not include non-monetary exchanges. (3) GDP does not consider the changes in the population of a country. (4) GDP does not account for valuation of externalities.

75



36. Option (3) is correct. Explanation: (a) Marginal Rate of Substitution (MRS): It refers to the number of units of good Y which the consumer is willing to gain for an additional unit of good X. (b) Consumer’s Bundle: It is a qualitative combination of three goods which can be purchased by a consumer from his given expenses at given prices. (c) Budget set: It is quantitative combination of those bundles which a consumer can purchase from his given income at prevailing market prices. (d) Consumer Budget: It states the real employment of the consumer from which he can purchase certain qualitative bundles of three goods at given price.

37. Option (3) is correct. Explanation: Autonomous investment is the portion of the total investment made by a government or other institution independent of economic considerations.

76

OSWAAL CUET (UG) Sample Question Papers, ECONOMICS

38. Option (1) is correct. Explanation: Minimum wage is fixed by the government in the interest of the producers above the equilibrium wage when the equilibrium price is too low.

39. Option (2) is correct. Explanation: Components of BoP Account are Current account, Capital account and Official international reserve account. Current account is that account of BoP, which records exports and imports of visible and invisible items and unilateral transfers. Capital Account of BoP records capital transfer such as loans and investments between one country and the rest of the world, which causes a change in the assets or liability status of the residents of the domestic country or its government. Official international reserve account includes the foreign exchange reserves, gold reserves and Special Depository Receipts (SDRs).

40. Option (2) is correct. Explanation: According to this approach, the producer is at equilibrium, when the Marginal Revenue (MR) is equal to the Marginal Cost (MC) and marginal cost curve cuts the marginal revenue curve from below.

41. Option (3) is correct. Explanation: A centrally planned economy, often known as a command economy, is an economic system in which economic decisions are made by a central authority, such as the government, affecting product creation and distribution. The Soviet Union is the closest example of a centrally planned economy.

42. Option (3) is correct. Explanation: Given, Cash reserve ratio = 10% Primary deposit = `1,250 crores Total deposits are the overall money including the primary deposits created by the commercial banks as a process of credit creation through its primary deposit. Total deposit = (1/ Cash reserve ratio) × Primary deposit = (1/10%) × `1,250 crores = (100/10) × `1,250 crores = 10 × `1,250 crores = `12,500 crores

43. Option (2) is correct. Explanation: Under long-run production function wherein it does not distinguish between fixed factors and variable factors, all the factors of production are variable.

44. Option (2) is correct. Explanation: As the price of substitute is increasing it leads to the increase in the demand and thus the rightward shift in the demand curve.

45. Option (1) is correct. Explanation: Total revenue can be defined as sum total of marginal revenues, i.e., TR = ΣMR. In other words, MR is calculated as follows: MR = TRn - TRn -1

46. Option (3) is correct. Explanation: In the third stage of production, total product starts decreasing and then marginal product falls below x-axis and hence becomes negative.

47. Option (1) is correct. Explanation: Money flow from firm to the household as factor payment in the form of wages, rent, interest and profits.

48. Option (1) is correct. Explanation: Two sectors of the economy: (a) Households (b) Firms

49. Option (4) is correct. Explanation: Significance of two sector economy : (a) It reflects structure of an economy. (b) It shows inter interdeputer among different sectors. (c) It shows injections and leakages from flow of money.

50. Option (3) is correct. Explanation: The two sector economy has the following assumptions: (1) There are only two sectors in the economy; household sector and business sector. (2)  No government interventions over the economic activities. (3)  Business sectors do not carry out any import or export activities, creating a closed economy.



SOLUTIONS OF Question Paper 1. Option (2) is correct. Explanation: Domestic income (NDPFC) is the net money value of all the final goods and services produced within the domestic territory of a country during a period of one year.

2. Option (2) is correct. Explanation: Primary deficit refers to the difference between fiscal deficit of the current year and interest payments on the previous borrowings. By deducting interest payment from fiscal deficit, we get primary deficit. Primary Deficit = Fiscal Deficit – Interest Payments.

3. Option (2) is correct. Explanation: If the amount of LRR is 20% then the initial deposit of `1000 crores would create the total amount of money in the economy worth ` 1000 × 1/0.2 = ` 5000 crores.

4. Option (3) is correct. Explanation: In microeconomics, shift in demand curve occur when a determinant of demand other than price changes.

5. Option (1) is correct. Explanation: Production function can be defined as the functional relationship between physical inputs and physical output of a good.

6. Option (2) is correct. Explanation: The term “balance of trade” denotes the difference between the exports and imports of goods in a country. Balance of trade refers to the visible items only. It is the difference between the value of merchandise (goods) exports and imports. Balance of Trade = Export of visible goods – Import of visible goods. Here, exports of goods worth `20 crore is less than the imports of goods worth `30 crore. Therefore, there is a deficit of `10 crores in balance of trade.

7. Option (4) is correct. Explanation: Deficit financing is an instrument of fiscal policy.

8

8. Option (1) is correct. Explanation: Production function shows the functional relationship between inputs and physical output of a good. Output = f(Land, Labour)

9. Option (2) is correct. Explanation: The four factors of production are defined as goods and services needed to carry out production i.e., land, labour, capital and entrepreneurship. They are available in limited quantities in relation to the demand.

10. Option (3) is correct. Explanation: As the percentage change in quantity demanded is less than the percentage change in the price, it is elasticity is less than 1 and is relatively inelastic.

11. Option (3) is correct. Explanation: In oligopoly, there are only few firms. If they compete on the basis of price, there is likely to be price war and the firm may loose. So, the firm adopts measures other than price for competing like customer care, after sale service, free gifts, etc. This is non-price competition.

12. Option (1) is correct. Explanation: NNPFC = GNPFC – Depreciation

13. Option (2) is correct. Explanation: Geometric method was suggested by Prof. Marshall and is used to measure the elasticity at a point on the demand curve. (i) When there are infinitely small changes in price and demand, then the ‘Geometric Method’ is used. (ii) This method is also known as ‘Graphic Method’ or ‘Point Method’ or ‘Arc Method’. Elasticity of demand (Ed) is different at different points on the same straight line demand curve.

78

OSWAAL CUET (UG) Sample Question Papers, ECONOMICS (iii) In order to measure Ed at any particular point, lower portion of the curve from that point is divided by the upper portion of the curve from the same point.

14. Option (1) is correct Explanation: To correct the situation of deficient demand RBI will reduce repo rate. Banks in turn will reduce lending rate of interest, so there will be more demand for loans. This will increase money supply and correct the situation of deficient demand.

15. Option (1) is correct. Explanation: The system of exchange rate in which exchange rate is officially declared and fixed by the government is called fixed exchange rate system. Fixed exchange rate is not determined by the forces of demand and supply in the market. Such a rate of exchange has been associated with Gold Standard System during 1880-1914.

16. Option (3) is correct. Explanation: In case of unitary elastic supply, percentage change in quantity supplied is equal to percentage change in price. Hence, elasticity of supply is equal to one.

17. Option (1) is correct. Explanation: Non-price competition like selling cost and advertisement is present in monopolistic competition.

18. Option (2) is correct. Explanation: MPC is the ratio between change in consumption and change in income.

19. Option (1) is correct. Explanation: Net Factor Income from Abroad is the difference between the income earned from abroad for rendering factor services by the normal residents of the country to the rest of the world and the income paid for the factor services rendered by non-residents in the domestic territory of a country.

20. Option (2) is correct. Explanation: Budget of a government shows its comprehensive exercise on the taxation and subsidies. A government uses fiscal instruments of taxation and subsidies with a view of improving the distribution of income and wealth in the economy. A government

reduces the inequality in the distribution of income and wealth by imposing taxes on the rich and giving subsidies to the poor, or spending more on welfare of the poor. It reduces income of the rich and raises the living standard of the poor, thus, leads to equitable distribution of income.

21. Option (3) is correct. Explanation: Condition for equilibrium : (a) Marginal cost is equal to marginal revenue. (b) Marginal cost curve cuts the marginal revenue curve from below.

22. Option (2) is correct. Explanation: Price floor is the minimum price which is decided by the government and fixed above the equilibrium price to be paid in a market for goods and services, labour, or financial market.

23. Option (4) is correct. Explanation: The money supply in Indian economy is generally measured in following forms: (i) M1 = Currency (notes and coins) with the public + Demand deposits + other deposits held with the Reserve Bank of India (ii) M2 = M1 + Post Office saving deposits (iii) M3 = M1 + Time deposits of all commercial banks and co-operative banks (excluding interbank time deposits) (iv) M4 = M3 + Total deposits with the Post Office Saving Organisation (excluding National Saving Certificate)

24. Option (3) is correct. Explanation: A decrease in CRR would all other things being equal, increase aggregate demand.

25. Option (4) is correct. Explanation: Utility : Capacity of a commodity to satisfy human wants. Normal Goods : When price increases, demand decreases Contraction in Demand : Rise in Price Complementary Goods : Positively related

Solutions 26. Option (4) is correct. Explanation: Capital account is that account which records all such transactions between residents of a country and rest of the world which cause a change in the asset or liability status of the residents of a country or its government. The main components of capital account are Loans, Foreign investments and change in Foreign Exchange Reserves.

27. Option (3) is correct. Explanation: The marginal revenue shows the relationship between marginal cost and quantity produced.

28. Option (2) is correct. Explanation: Market Value if the Market Cost and Transfer Value is the cost of Transfer payments.

29. Option (2) is correct. Explanation: When demand and supply both increase, but increase in demand is greater than the increase in supply, then equilibrium price will rise and equilibrium quantity will also rise.

30. Option (2) is correct. Explanation: A demand deposit is money deposited into a bank account with funds that can be withdrawn on-demand at any time.

31. Option (1) is correct. Explanation: When the government thinks that the prices are too high for the essential commodities like food, medicine etc, then government sets the maximum price that producers can charge, this price lies below the equilibrium price so as the price ceiling can be effective.

32. Option (3) is correct. Explanation: Balance of trade is the net difference of import and export of all visible items between the normal residents of a country and rest of the world.

33. Option (4) is correct. Explanation: Determinants of Demand : Important determinants of demand are: (a) Price of commodity, (b) Price of related commodities, (c) Money income of the consumers, (d) Tastes and preferences of consumers, (e) Changes in weather conditions,



(f) (g) (h) (i) (j) (k) (l)

79

Changes in population, Distribution of income, Changes in structure of population, Changes in quantity of money, Distribution of National Wealth, Phases of business cycles, Change in saving habits

34. Option (1) is correct. Explanation: Revenue Expenditure is expenditure incurred for purposes other than the creation of physical or financial assets of the central government. It relates to those expenses incurred for the normal functioning of the government departments and various services, interest payments on debt incurred by the government, and grants given to state governments and other parties, even though some of the grants may be meant for creation of assets.

35. Option (1) is correct. Explanation: MC curve cuts AC curve which means MC =AC. And in this stage, Average cost is constant and is at its minimum point.

36. Option (4) is correct. Explanation: When the unitary elastic supply is said to be one when the percentage change in quantity supplied is equal to percentage change in price. Elasticity of supply of gold is more elastic as percentage change in quantity supplied is greater than percentage change in price.

37. Option (3) is correct. Explanation: There can be either no depreciation or some depreciation, but negative depreciation is never possible.

38. Option (4) is correct. 39. Option (2) is correct. Explanation: C - Currency R - Required reserve ER - Excess reserve

40. Option (2) is correct. Explanation: The other name for law of production is law of variable proportion or returns to a factor or returns to a variable factor.

41. Option (3) is correct. Explanation: Marginal revenue curve is equal to price axis in a perfectly competitive market. Hence, it is parallel to x-axis.

80

OSWAAL CUET (UG) Sample Question Papers, ECONOMICS

42. Option (4) is correct. Explanation: Intermediate goods and second hand goods will lead to the problem of double counting. As no value is added for self consumption, thus it is not included in the calculation of National Income.

43. Option (4) is correct. Explanation: Microeconomics helps in formulating economic policies which enhance productive efficiency and results in greater social welfare. But it has a limitation. Microeconomics fails to explain the functioning of an economy as a whole. It cannot explain unemployment, poverty, illiteracy and other problems prevailing in the country.

44. Option (4) is correct. Explanation: AD=C+I+G+X-M Where, C = Household Consumption Expenditure I = Private Investment Expenditure G = Government Expenditure X-M = Net Exports

45. Option (1) is correct. Explanation: Revenue Receipts are the receipts that neither create any liability nor reduction

in assets of the government. It includes tax revenues like income tax, corporation tax and non-tax revenue like fines and penalties, special assessment, escheat, etc.

46. Option (3) is correct. Explanation: As Retained Earnings are kept aside from the profit, they are actually a part of profit.

47. Option (4) is correct. Explanation: Utility can be studied both cardinally and ordinally.

48. Option (2) is correct Explanation: In Ordinal utility theory, the utility is measured through indifference curve.

49. Option (1) is correct Explanation: In case of cardinal utility approach the utility is measured in terms of utils and so is comparable.

50. Option (1) is correct. Explanation: The Marginal utility Curve is the curve that represents the marginal utility that is the change in utility as the consumption increases by 1 unit. Indifference Curve shows the combination of two goods that gives the same amount of satisfaction.



SOLUTIONS OF Question Paper 1. Option (4) is correct. Explanation: Precautions while using Expenditure Method: (i) Only final expenditure is to be taken into account to avoid error of double counting. (ii) Expenditure on second hand goods is not to be included. (iii) Expenditure on transfer payments by the government is not to be included. (iv) Imputed value of expenditure on goods produced for self consumption should be taken into account. (v) Expenditure on shares and bonds is not to be included in total expenditure.

2. Option (1) is correct. Explanation: Building of railway tracks is a capital expenditure as it is creating an asset. An expenditure that either create assets for the government or cause reduction in liabilities of the government is a capital expenditure. In capital expenditure any one of the above conditions must be satisfied. Thus, it refers to expenditure that leads to creation of assets and reduction in liabilities. Such expenditure is incurred on long period development.

3. Option (4) is correct. Explanation: (a) Demand Curve of Perfect Competition: Horizontal to X- axis (b) Demand Curve of Mono poly : Downward sloping (c) Demand Curve of Monopolistic Competition : Downward Sloping (d) Demand Curve of Oligopoly : Indeterminant

4. Option (1) is correct. Explanation: In money supply, CU is currency (notes plus coins) held by the public and DD is net demand deposits held by commercial banks. The word ‘net’ implies that only deposits of the public held by the banks are to be included in money supply. The interbank

9

deposits, which a commercial bank holds in other commercial banks, are not to be regarded as part of money supply.

5. Option (3) is correct. Explanation: Supply function shows the functional relationship between supply of a commodity and the various factors like price of related goods, price of factors of production, goals of the firm etc.

6. Option (3) is correct. Explanation: Budget is a financial statement showing the expected receipt and expenditure of Government for the coming fiscal or financial year.

7. Option (2) is correct. Explanation: When the investment is less than savings, the expenditure in the economy is less than what producers had expected, resulting in undesired building-up of unsold stock. Consequently, AD falls short of AS.

8. Option (1) is correct. Explanation: Simplest consumption function assumes that the consumption changes constantly as per the change in the income.

9. Option (1) is correct. Explanation: Law of supply shows that the direct relationship between price and quantity supplied. As price rises, quantity supplied also rises and as the price falls, quantity supplied also falls.

10. Option (2) is correct. Explanation: When the goods and services are produced by all producing units in the domestic territory of a country during an accounting year and valued at base year’s prices or constant price, it is called real GDP or GDP at constant prices.

11. Option (1) is correct. Explanation: Oligopoly is a form of market in which there are few big sellers of a

82

OSWAAL CUET (UG) Sample Question Papers, ECONOMICS commodity and a large number of buyers. Each seller has a significant share of the market.

12. Option (1) is correct. Explanation: Lending to Kenya will result in increase in assets.

13. Option (3) is correct. Explanation: (a) Feature of Perfect Competition : Large Number of buyers and sellers (b) Feature of Monopoly Market : Only one seller (c) Feature of Monopolistic Competition : Large Number of Buyers and Sellers (d) Feature of Monopoly Market : Large number of buyers

14. Option (1) is correct. Explanation: Price Elasticity of Demand is defined as the measurement of percentage in quantity demanded in response to a given percentage change in own price of the commodity.

15. Option (2) is correct. Explanation: When the price of related goods fall, the good becomes relatively more profitable to produce, shifting the supply curve to the right.

16. Option (1) is correct. Explanation: Marginal revenue equals the market price because market price is not affected by the output choice of a single firm.

17. Option (3) is correct. Explanation: M1 and M2 are narrow money as they include highly liquid assets which are easily acceptable for payments M3 = M1 + Time Deposits of Public with Banks.

18. Option (2) is correct. Explanation: Adverse bot is a situation in which the value of the goods a country import is more than the value of goods it exports.

19. Option (2) is correct. Explanation: Real GDP changes only by change in physical output not by change in price level. It is called a true indicator of economic development.

20. Option (2) is correct.

Explanation: (a) Budget Line : . M = Px·x + Py·y (b) Bajra : Inferior goods (c) Consumer equilibrium: Price ceiling (d) Elastic Demand : Luxurious goods

21. Option (2) is correct. Explanation: APS is the ratio of saving to income.

22. Option (1) is correct. Explanation: Income effect means with fall in the price of a good, consumer’s real income or purchasing power rises and he demands more units of the good. This shows that when the price falls, demand increases and so the demand curve is downward sloping.

23. Option (4) is correct. Explanation: A higher indifference curve represents a higher level of satisfaction. It is because of the assumption that preferences are monotonic. Therefore, the assertion is incorrect and the reason is true.

24. Option (3) is correct. Explanation: The difference between export and import of goods, i.e. only the visible items of economic transactions is termed as Balance of Trade. A deficit in Balance of Trade indicates that the value of export of goods is less than the imports of goods for a country.

25. Option (2) is correct. Explanation: Marginal revenue can be defined as change in total revenue divided by increase in one unit of output. A TR curve is a straight line coming out of the origin the slope of a TR is equal to MR, it equals the market price and AR in perfect competition.

26. Option (1) is correct Explanation: Aggregate demand and supply affected by saving and investment.

27. Option (1) is correct. Explanation: A rational firm aims to operate in second phase of law of variable proportion which is decreasing returns to a factor because in this phase, total product reaches its maximum point and marginal product starts to fall but is positive.

Solutions 28. Option (2) is correct. Explanation: In 2016-17 the value of non-tax revenue is 2.7 and in 2017-18 the value of nontax revenue is 1.9. The difference between the two is 2.7-1.9 = 0.8. So, the value of non-tax revenue has risen by 0.8.

29. Option (3) is correct. Explanation: In general, Real GDP and Welfare are directly related with each other. A higher GDP implies more production of goods and services. It means more availability of goods and services. But more goods and services may not necessarily indicate that the people were better off during the year. In other words, a higher GDP may not necessarily mean higher welfare of the people.

30. Option (2) is correct. Explanation: Perfect Competition is a type of market where there are a large number of buyers and sellers selling homogeneous goods.

31. Option (1) is correct. Explanation: The term investment multiplier refers to the concept that any increase in public or private investment spending has a more than proportionate positive impact on aggregate income and the general economy.

32. Option (2) is correct. Explanation: Total physical product can be defined as total output produced by a firm during a given period of time with given number of inputs. In other words, total product is total production made by a firm.

33. Option (4) is correct. Explanation: Current account is that account of BoP, which records exports and imports of visible and invisible items and unilateral transfers. Balance of current account is the difference between sum of credit items and sum of debit items entered in the current account.

34. Option (3) is correct. Explanation: Tea Shops are Monopolistic Competition; Cafeteria are Monopolistic Competition; Railways are Monopoly.

35. Option (1) is correct. Explanation: Interest Earned by normal resident is included in the National Income as it is a type of transfer payment.

83

36. Option (2) is correct. Explanation: Homogeneous products are those products which are similar in size, colour, brand, design etc. They are perfect substitutes of each other.

37. Option (1) is correct. Explanation: The above statement is true that production function exhibits technological relationship between physical inputs and output. For example, output is a function of Land and Labour.

38. Option (4) is correct. Explanation: The company does not consider any other basis like direct and indirect costs or revenue and capital cost or functional classification for cost classification.

39. Option (2) is correct. Explanation: Only Geeta is correct as the Central Bank is only the controller of credit and not the Commercial bank. The commercial banks only create credit. Credit control and control of money supply is probably the most important function of a Central Bank. Through various methods/instruments of credit control the Central Bank aims to achieve growth with stability in an economy.

40. Option (1) is correct. Explanation: As the expenditure on subsidies and on scholarships do not create an asset nor does it reduce a liability, so it is an example of revenue expenditure.

41. Option (4) is correct. Explanation: Air conditioners are luxury goods and luxury goods have an elastic demand curve.

42. Option (3) is correct. Explanation: Factor Cost is the sum total of the money values.

43. Option (1) is correct. Explanation: As the movement is from left to right and the demand curve shows a negative relation, the price elasticity of demand will keep on falling.

44. Option (4) is correct. Explanation: AD=C+I+G+X-M Where, C=Household Consumption Expenditure

84

OSWAAL CUET (UG) Sample Question Papers, ECONOMICS I=Private Investment Expenditure G=Government Expenditure X-M=Net Exports

45. Option (4) is correct. Explanation: Income of the teacher teaching at his own house is a mixed income which will form a part of Domestic Income, which in turn will be added to the National Income.

46. Option (1) is correct. Explanation: (a) Feature of Monopoly Market - There is only one seller. (b) Advantage of Monopoly Market - Firm is a price maker (c) Limitation of Monopoly Market Government policies and restriction (d) Feature of Perfect Competition - Free entry and exit of the firm

47. Option (4) is correct.

48. Option (4) is correct. Explanation: Opportunity cost is the next best alternative you can choose among the list of available options. Here the next best alternative is writing books, in which he can earn 2.5 lakhs rupees.

49. Option (1) is correct. Explanation: Resources are scarce and can be of four types; natural (land), human (labour), capital (machines), and entrepreneurship. For answering the question `how to produce?’, an economy must make a choice of the wants which are important for the economy. Even though the two statements are not linked together, both are true individually.

50. Option (1) is correct. Explanation: India has a mixed economy, with both the private and state sectors coexisting, and the country taking advantage of international trade.

Explanation: As per the above paragraph, problem of choice has to be laced by every economy of the world.



SOLUTIONS OF Question Paper 1. Option (1) is correct. Explanation: Assam Smith follow positive school of economics.

2. Option (1) is correct. Explanation: K = 1/MPS K = 1/1 K=1

that there is a balance between the goals of profit maximisation and social welfare. A government uses fiscal instruments of taxation and subsidies with a view of improving the distribution of income and wealth in the economy. Budget is used as an important policy instrument to combat the situations of deflation and inflation.

8. Option C is correct.

3. Option (1) is correct. Explanation: Legal Reserve Requirement (LRR) is 10% or 0.1 and Money Multiplier

1 = LRR Money Multiplier =

10

1 = 10 0.1

If total deposits created is of `10,000 crores Initial Deposits = Total Deposits/Money

10 , 000 Multiplier = 10 Initial Deposits = `1,000 crore

4. Option (1) is correct. Explanation: Recovery of loan is capital receipt as it reduces assets.

5. Option (2) is correct. Explanation: If there is a good which takes up significant share of consumer budget then it implies it cann’t be any necessaries comfort or jointly demanded goods.

6. Option (2) is correct Explanation: Sum of the quantities that all consumers of a commodity are willing to buy at a given price during a period of time. Market demand curve is derived as a horizontal summation of individual demand curves.

7. Option (4) is correct. Explanation: Through its budgetary policy the government of a country directs the allocation of resources in a manner such

Explanation: Net Indirect Tax is subtracted from Market Price to get to Factor Cost.

9. Option (2) is correct. Explanation: Total cost can be defined as the sum of fixed cost and variable cost. The cost incurred on fixed and variable factors of production is called as total cost.

10. Option (4) is correct. Explanation: K=1/MPS K=1/0 K=∞

11. Option (2) is correct. Explanation: Price ceiling is imposed on essential commodities like food, rent etc. so that consumers are not exploited by the higher prices.

12. Option (2) is correct. Explanation: M2 = M1 + Post Office saving deposits

13. Option B is correct. Explanation: The domestic Value is related to the goods produced within the territory of a country, so it is a territorial concept.

14. Option (4) is correct. Explanation: Allocation of resources means how much of each resource is devoted to the production of goods and services. As we know that resources are scarce, it is important to find out the answers of what, how and for whom, for the smooth functioning of the economy.

86

OSWAAL CUET (UG) Sample Question Papers, ECONOMICS

15. Option (2) is correct. Explanation: Since fixed cost curve is same at all levels of output, hence fixed cost curve is parallel to x-axis. Variable cost first increase at decreasing rate and then increases at increasing rate. Hence, TVC is inverse S-shaped curve.

16. Option (1) is correct. Explanation: As Incomes are the payments given to various factors of production, it is also called the factor payment method.

17. Option (3) is correct. Explanation: Monotonic Preferences : Consumer’s preferences are called monotonic when between any two bundles, consumer always choose a bundle having more of one good and no less of other goods. Indifference Set : It is a set of those combinations of two goods which offer the consumer the same level of satisfaction, so that the consumer is indifferent across any number of combinations in his indifference set. Indifference Curve : It is a curve showing different combination of two goods, each combination offering the same level of satisfaction to the consumer. Indifference Curve : A curve which is graphical presentation of an indifference set showing different combinations of two commodities between which a consumer is indifferent. Indifference Map : It refers to a set of indifference curves placed together in a diagram.

18. Option (1) is correct. Explanation: Government receipts, which neither create any liabilities for the government; and not cause any reduction in assets of the government, are called revenue receipts. In revenue receipts both the conditions should be satisfied. Revenue receipts include items which are repetitive and routine in nature. Dividend from public sector undertaking is a source of revenue receipt for the government as it is recurring in nature.

19. Option (3) is correct. Explanation: Capital Formation includes fixed capital, variable capital and also the inventory.

20. Option (1) is correct. Explanation: Imports lead to an outflow of foreign exchange in the country. Thus, they are recorded as negative (debit) items.

21. Option (1) is correct. Explanation: Aggregate supply is the total supply of goods and services produced within an economy at a given overall price in a given period.

22. Option (4) is correct. Explanation: When the situation of price ceiling arises, then it leads to black marketing by the producers wherein they sell commodity at a higher price than the price fixed by the government. This leads to fall in production level and hence producers also decrease the quality of productions.

23. Option (1) is correct. Explanation: Law of diminishing marginal utility states that as more and more units of a commodity are consumed, marginal utility derived from every additional unit must decline.

24. Option (2) is correct. Explanation: Visible items include material goods (such as sugar, cloth, machines etc.) which can be seen or touched, counted, measured and weighted and which are duly recorded at the custom barriers.

25. Option (1) is correct. Explanation: Price ceilings benefit the society, as they make sure that essential goods are financially accessible to the average person, at least in the short run. This leads to increase in demand which contributes to the health of an economy.

26. Option (3) is correct. Explanation: When the goods and services are produced by all producing units in the domestic territory of a country during an accounting year and valued at current year’s prices or current prices, it is called Nominal GDP or GDP at current prices. It is influenced by change in both physical output and price level. It is not considered a true indicator of economic development.

Solutions 27. Option (4) is correct. Explanation: The RBI acts as a lender of last resort for the commercial banks. The commercial banks, when facing a financial crisis, can go to the Central Bank. The Central Bank lends money to the commercial bank, so that it can come out of the crisis and help the commercial bank.

28. Option (3) is correct. Explanation: Properties of PPC : The two basic characteristics or properties of PPC are : PPC slopes downwards : It slopes downwards from left to right because more of one good can be produced only by taking resources away from the production of another good. PPC is concave shaped : PPC is concave shaped because of increasing MRT, that is, more and more units of a commodity are sacrificed to gain one additional unit of another commodity.

29. Option (2) is correct. Explanation: It is the demand for a product or service which occurs when purchaser are constrained in a different market.

30. Option (2) is correct. Explanation: Dis-investments are capital receipts because it reduces assets. The receipts which create corresponding liability for the government or which lead to reduction in assets of the government are termed as capital receipts, e.g., loans taken by the government, disinvestment of any PSUs, etc.

31. Option (3) is correct. Explanation: When MC is less than AC, then average cost curve falls. AC is per unit cost of output. MC is the addition to the total cost when an additional unit of output is produced.

32. Option (3) is correct. Explanation: In a system of flexible exchange rates (also known as floating exchange rates), the exchange rate is determined by the forces of market demand and supply.

33. Option (4) is correct. Explanation: Under keynes theory, excess demand has the following effect on output, employment and general price level effect on : (a) Output/employment - no change. (b) General price level - excess demand leads to rise in price level.

87

34. Option (2) is correct. Explanation: When the person is satisfied completely the Total Utility derived from that commodity is maximum as it starts to fall after that.

35. Option (1) is correct. Explanation: TV seller are Monopolistic Competition Market. Microwave Oven are Monopolistic Competition Market. Fruits Market is Perfect Competition Market.

36. Option (1) is correct. Explanation: Monopolistic competition : (i) Market has the characteristic . (ii) Characteristic of both perfect and monopoly competition.

37. Option (2) is correct. Explanation: Balance of payments is an accounting statement that provides a systematic record of all the economic transactions between the residents of a country and the rest of the world during a given period of time.

38. Option (4) is correct. Explanation: Things not included in National Income (i) Income from illegal activities like smuggling, theft, gambling, etc., should not be included. (ii) Imputed rent of owner occupied structure and value of production for self-consumption is included but value of self-consumed services like those of housewife is not included. (iii) Brokerage on the sale/purchase of shares and bonds is to be included. (iv) Income in terms of windfall gains should not be included. (v) Transfer earning like old age pensions, unemployment allowances, scholarships, pocket expenses, etc., should not be included.

39. Option (3) is correct. Explanation: Assertion is true as it explains the shape of the TU curve but the reason is wrong. The correct statement should be that the falling MU curve shows the law of diminishing marginal utility.

88

OSWAAL CUET (UG) Sample Question Papers, ECONOMICS

40. Option (2) is correct. Explanation: MRS is the rate at which the consumer is willing to give up commodity `Y’ for one more unit of commodity `X’. He tries to maintain the same level of satisfactions diminishes when one proceeds down a standard convex-shaped curve, such as the indifference curve, according to the law of diminishing marginal rates of substitution. Both the statements are true but the reason is not the correct explanation of the assertion.

46. Option (2) is correct. Explanation: Opportunity Cost

Cost of next best alternative forgone

Implicit Cost

Any cost that has already occurred but not necessarily shows or reported as a separate expense.

Explicit cost

These cost are normal business cast that appear in the general ledger and directly affect a company profitability.

Hidden cost

Are unforeseen expense added on to purchase.

41. Option (4) is correct. Explanation: Price ceiling is imposed by the government when the price decided by the forces of demand and supply is too high for the poor people and hence government imposes this on essential commodities like medicine and agricultural goods.

42. Option (2) is correct. Explanation: Wages and Salaries form the part of Compensation to employees.

43. Option (3) is correct. Explanation: Relationship between Total Utility and Marginal Utility : (i) When MU is positive, TU will be increasing. (ii) When MU is zero, TU is maximum. (iii) When MU is negative, TU will be decreasing.

44. Option (3) is correct. Explanation: M1 and M2 are known as narrow money as they can be converted into cash quickly than M3 and M4.

45. Option (1) is correct. Explanation: It measures the difference between the current level of real GDP and the GDP that would exist if an economy was operating at full employment.

47. Option (4) is correct. Explanation: The increase in the supply of food grains can be due to various reasons like improvement in technology, fall in the price of factor inputs, rise in production and due to favourable agricultural policies etc.

48. Option (1) is correct. Explanation: Agriculture provides livelihood to approximately three-fourth of the population in India because majority of population lives in rural areas.

49. Option (3) is correct. Explanation: Statement 1 is true as it says that till the mid-eighties , there was increase in production of food-grains and area under the food-grain also increased but after mideighties, there was decline in the area of food grain.

50. Option (4) is correct. Explanation: The supply of food grain production is affected by all the above factors like cost of fertilizers, favourable irrigation facilities etc.



Attention ambitious achievers! Prepare to conquer the CUET(UG) exam with Oswaal360! Unlock the path to success with double the value of your purchase and gain access to: Captivating Video Lectures

Challenging Practice Tests

Realistic Mock Tests

Don't miss out on our upcoming series of Exclusive Mock Tests, designed to boost your confidence and ensure success. Prepare for other competitive exams like NEET, JEE, and more, and pave the way to a bright future.

Scan now and Unlock the door to your dream college with Oswaal360 by your side!

CUETECO

Share your review of the books and help your fellow students & juniors.

Economics

Scan to Review

Attention ambitious achievers! Prepare to conquer the CUET(UG) exam with Oswaal360! Unlock the path to success with double the value of your purchase and gain access to: Captivating Video Lectures

Challenging Practice Tests

Realistic Mock Tests

Don't miss out on our upcoming series of Exclusive Mock Tests, designed to boost your confidence and ensure success. Prepare for other competitive exams like NEET, JEE, and more, and pave the way to a bright future.

Scan now and Unlock the door to your dream college with Oswaal360 by your side!

CUETECO

Share your review of the books and help your fellow students & juniors.

Economics

Scan to Review